Вы находитесь на странице: 1из 91

Table of Contents

FORMATION OF THE AGREEMENT: OFFER AND ACCEPTANCE...................3


1. Distinction between and invitation to treat (bargain negotiations) and an
offer.......................................................................................................... 3
Canadian Dyers Assn. Ltd. v. Burton (1920), H.C:........................................................4
Pharmaceutical Society of GB v. Boots Cash Chemists (1953), C.A:.............................5
Unilateral Offers: Offers to the General Public..............................................5
Carlill v. Carbolic Smoke Ball (1893, H.L)- UK...............................................................5
Goldthorpe v Logan (1943) C.A.................................................................................... 6
Invitation to Bid..........................................................................................6
Harvela Investments Ltd. V Royal Trust Co. of Canada (1986) A.C...............................7
Tendering...................................................................................................7
R v Ron Engineering and Construction (1981)SCC.......................................................7
MJB Enterprises v Defense Construction Ltd. (1999) SCC.............................................9
2. Communication of Offer...........................................................................9
Blair v Western Mutual Benefit Assn [1972] BC CA.....................................................11
Williams v Carwardine [1833] U.K.............................................................................. 11
R v Clark [1927] Australia HC..................................................................................... 12
3. Acceptance...........................................................................................12
(A) Acceptance by Conduct........................................................................12
Livingstone v Evans [1925] Alta SC............................................................................13
Battle of the Forms...................................................................................13
Butler Machine Tool Co. v Ex-Cell-O Corp. [1979] CA..................................................13
Tywood Industries LTD. v St. Anne-Nackawic Pulp & Paper Co. LTD. [1979] Ont HC. . .14
E-Commerce.............................................................................................15
ProCD v Matthew Zeidenberg and Silken Mountain Web Services, INC. [1996] US C.A
................................................................................................................................... 15
(B) Silence................................................................................................16
Dawson v Helicopter Exploration CO. [1955] SCC.....................................................16
Felthouse v. Bindley (1862) (English Case).................................................................16
St. John tug boat co. v Irving refinery ltd. (1964) SCC................................................16
Eliason v. Henshaw (1819) US.................................................................................... 19
4. Communication of Acceptance...............................................................20
A) Mailed Acceptances.............................................................................20
Household Fire & Carriage Accident Insurance Co. v. Grant (1879) CA.......................20
Holwell Securities v. Hughes (1974) CA......................................................................21
Instantaneous Communication (faxes, emails, etc).....................................22
Brinkibon Ltd. v. Stahag Stahl Und Stahl Warenhandlesgesellschaft mbH (1983)
English Case (Instantaneous Communication)...........................................................22
Rudder v. Microsoft Corp. (1999) Ont SCJ (Instantaneous communication)...............23
4. Termination of Offer.............................................................................24
a) Revocation..........................................................................................25
Dickinson v. Dodds (1876) CA.................................................................................... 25
Byrne v. Van Tienhoven (1880) CA............................................................................. 25
Errington v. Errington and Woods (1952) K.B CA (not Canadian)...............................26
b) Lapse...................................................................................................26
Barrick v. Clark (1951) SCC........................................................................................ 26

Chapter 3: FORMATION OF THE AGREEMENT: CERTAINTY OF TERMS......28


1. Vagueness............................................................................................28
R. v. Cae industries ltd. (1986) CA..............................................................................28
2. Incomplete terms..................................................................................30
1

May & Butcher Ltd. v. R. (1934) UK last years mid term...........................................30


Hillas & Co v. Arcos Ltd (1932) UK HL.........................................................................31
Foley v. Classique Coaches ltd. (1934) KB CA.............................................................32
4. Agreement to Negotiate.......................................................................33
Empress Towers Ltd. v. Bank of Nova Scotia 1991 NSCA............................................33
Mannpar Enterprises Ltd v. Canada 1999 BCCA.........................................................34
Wellington City Council v. Body Corporate 2002 Newzealand CA...............................36
5. Anticipation of Formalization.................................................................37
Bawitko Investments Ltd. v. Kernels Popcorn Ltd 1991 ONCA....................................37

Chapter 4: THE ENFORCEMENT OF PROMISES......................................39


1. Exchange and Bargains.........................................................................39
The Governors of Dalhousie College at Halifax v. The Estate of Arthur Boutilier,
Deceased (1934) SCC................................................................................................ 39
Brantford General Hospital Foundation v. Marquis Estate [2003] ON SCJ...................40
Wood v. Lucy, Lady Duff-Gordon 1917, U.S................................................................40
2. Past Consideration................................................................................40
Eastwood v. Kenyon (1840)........................................................................................ 40
Lampleigh v. Brathwait (1615)..................................................................................41
3. Consideration Must be of Value in the Eyes of the Law............................41
Thomas v. Thomas 1842, Q.B..................................................................................... 41
6. Pre-Existing Legal Duty.........................................................................42
Pao On v. Lau Yiu Long 1980 PC duty owed to a third party....................................42
Stilk v. Merrick (1809) Eng.......................................................................................... 43
Gilbert Steel Ltd. v. University Const. Ltd. 1976, C.A. Ont.........................................43
Williams v. Roffey Bros & Nicholls (Contractors) Ltd. 1990, C.A. England ...............43
Greater Fredericton Airport Authority Inc. v. Nav Canada [2008] NBCA.....................44

ACCORD AND SATISFACTION:..............................................................44


7. PROMISES TO ACCEPT LESS...................................................................45
Foakes v. Beer 1884, H.L............................................................................................ 45
Re SelectMove Ltd. 1995, C.A. England......................................................................45
Foot v. Rawlings 1963, S.C.C...................................................................................... 45

PROMISSORY ESTOPPEL AND WAIVER.................................................46


6 Basic Criteria that must be present in order to have promissory estoppel:.............46
Hughes v. Metropolitan Railway Company AC HL 1877..............................................46
Central London Property Trust Ltd. v. High Trees House Ltd. K.B. 1947......................47
a) The Nature of the Representation..........................................................47
John Burrows Ltd. v. Subsurface Surveys Ltd 1968, S.C.............................................47
B) The Equities.........................................................................................47
D. & C. Builders Ltd. v. Rees 1966, Q.B......................................................................47
Saskatchewan River Bungalows Ltd. v. Maritime Life Assurance Co. 1994, S.C........48
d) THE RELIANCE.......................................................................................48
W.J. Alan & Co. v. El Nasr Export & Import Co. 1972 QB CA........................................48
Societe Italo-belge Pour le Commerce et LIndustrie S. A. v. Palm and Vegetable Oils
(Malaysia) SDN BHD; The Post Chaser.. 1982 QB.......................................................49
E) SWORD OR SHIELD?..............................................................................49
Petrides v. Shabinsky 1982, Ont. H.C.........................................................................49
Robichaud c.Caisse Populaire De Pokemouche Ltee 1990, N.B. C.A..........................50
Combe v. Combe 1951 C.A......................................................................................... 50
Waltons Stores (Interstate) Pty. Ltd. v. Maher (1988) H.C. of Australia......................50
M.(n) v. A (A.T.) 2003, (BCCA) P254............................................................................50

INTENTION TO CREATE LEGAL RELATIONS............................................51


B) FAMILY ARRANGEMENTS........................................................................51
Balfour v. Balfour 1919, Eng.C.A................................................................................ 51
2

c) COMMERCIAL ARRANGEMENTS...............................................................51
Rose and Frank Co. v. J.R. Crompton and Bros. Ltd. 1923, K.B. CA.............................51
Toronto Dominion Bank v. Leigh Instruments Ltd. (Trustee of) Ont.CA, leave to appeal
refused (2000)............................................................................................................ 52

FORMALITY Promises under seal......................................................52


d) PROMISES UNDER SEAL.........................................................................52
Royal Bank v. Kiska 1967, Ont. C.A............................................................................52

FORMALITY: The Requirement of Writing.............................................52


Dynamic Transport Ltd v O.K Detailing Ltd 1978 SCC.................................................53
Deglman v. Guaranty Trust Co. S.C. 1954.................................................................53
Thompson v. Guaranty Trust Co. 1974, S.C...............................................................53
Lensen v. Lensen 1984, Sask. CA..............................................................................54

CHAPTER 5: PRIVITY OF A CONTRACT..................................................54


Tweddle v. Atkinson 1861 Q.B.................................................................................... 54
Dunlop Pneumatic Tyre Co. Ltd. v. Selfridge & Co. Ltd. 1915, H.L..............................55
A Third Party may Acquire the Benefit through:..........................................55
Beswick v. Beswick 1966, C.A. Eng.............................................................................55
VANDERPITTE V PREFERRED ACCIDENT INSURANCE CO. 1933 PC..............................56
Agency.....................................................................................................56
McCannell v Mabee McLaren Motors LTD 1926...........................................................56
New Zealand Shipping CO LTD v AM Satterthwaite & CO LTD 1975 PC......................56
Employment.............................................................................................57
London Drugs LTD v Kuehne & Nagel International LTD 1992 SCC.............................57
Edgeworth Construction LTD v ND Lea & Associates LTD 1993 SCC...........................58
Subrogation.............................................................................................58
Fraser River Pile & Dredge LTD v Can-Dive Services LTD 1993 SCC............................58

FORMATION OF THE AGREEMENT: OFFER AND ACCEPTANCE


1. Distinction between and invitation to treat (bargain negotiations) and
an offer
-

Simple negotiations, there is no intention yet to enter a contractto actually make an offer
(the person who makes the offer is the offeror), the offeror is prepared to enter the contract
once the offeree accepts so the power of acceptance is in the hand of the offeree.
There usually are terms with it (payment is due upon delivery, for ex.) there can only be an
offer if the offeree accepts those terms exactly the same way as they were offered by the
offeror. (mirror image rule)
If there is a change in the terms, we have a counter offerno acceptance. Then the only thing
on the table is the counter offer.
Invitation to treat is an invitation to start bargaining, this is not an offer.
The distinction between the offer and the invitation to treat is the willingness of the offeror to
be bound by the next communication of the offeree, as long as they have not withdrawn the
offer before the offeree accepts. The offeror can always withdraw the offer up until the point
the offeree accepts
the first prerequisite to contractual liability based on consent is that the parties must have
reached an agreement
1. communication of the offer
2. acceptance of the terms of the offer
3. (consideration) or communication of the acceptance (value? Usually monetary, doesnt
have to be though)
this is to assist in defining the moment of responsibility rather than a priori statements to
be blindly applied in a broad variety of circumstances
meeting of the minds or consensus ad idem - contract made on the terms and
conditions that were agreed upon.

Case and Facts

Canadian Dyers
Assn. Ltd. v. Burton
(1920), H.C:
- D requested lowest price
from B who offered $1,650
- B said willing to accept
$1650 for them only
(stresses this point)
- D sent cheque and B
received though not
deposited.
- B drew up draft deed
- B then claimed no K

Issues/Holding
- Had a contract been made
out?
- the conduct of B indicates
that there was a contract

Reasoning/Ratio
- The mere quotation of the price does
not constitute an offer to sell
- No contract unless there is offer to sell
and acceptance of that offer
- there was intention of an offer since said
willing to accept $1650, and the fact that
they went on to draft a deed, those are all
actions that they knew they made an offer
- a party must intend for it to be an offer.
- whether or not theres an offer (or
intention) will depend on the language
used and the circumstances of a
particular case...courts can also look at
subsequent actions of the parties.

Pharmaceutical
Society of GB v.
Boots Cash Chemists
(1953), C.A:
- self service drug store,
place in basket
- pharmacist is to make the
decision if customer can
buy.
- pharmaceutical company
thought she was not in
compliance with the
legislation because
customers allowed to come
into store put it in shopping
cart and thats when offer is
completed.
- seen as not an offer,
plaintiff appealed, appeal
dismissed

- is the display of goods on a


shelf an offer or invitation to
treat?
- it is an invitation to treat
contract not completed
when shopper put it in cart
(so, still in control of the
pharmacist)

-if the act of taking something off


the shelf and into cart is
acceptance, they are bound at
that point and cant put it back on
shelfcant change mindso not
an offer
- An offer to buy occurs when
customer takes goods off shelf,
into basket and brings them up to
the cash desk, offering to buy
those selected items. Ringing in
is the acceptance point. (offeree
has power of acceptance) the
consumer is offeror (to buy), and
cashier accepts the offer
- the priced item on the shelf is an
invitation to treat, so offeror is
person who makes the offer to buy
- the act of taking something off the
shelf does not amount to an acceptance
of an offer to sell because the goods on
the shelf are an invitation to treat

Unilateral Offers: Offers to the General Public


Carlill v. Carbolic
Smoke Ball (1893,
H.L)- UK
- advertisement for carbolic
smoke ball, user not to get
influenza
- plaintiff used it and got
influenza, tried to claim
award
- defendant appealed
decision, appeal dismissed

- was a contract formed


when the plaintiff performed
the conditions set out in the
advertisement
- unilateral contractyes,
there was a contract formed

- Offer can be made to general


public (not a contract with the
world - just an offer)
- they said there was no intention
to have legal relations, it was
mere puff, they say it was vague.
But the plain meaning was that
they wouldnt get sick within the
two weeks of using it. Since they
said they went to the bank to put
1000 in to show they were serious
- Offer is limited to those who
accept or perform duties outlined
in offer. So, a unilateral contract
is an offer made to the whole
world and is accepted by
performance of this act.
Inconvenience sustained by one
party is the consideration.
Notification of acceptance is not
necessary, as offeror sets out
terms in offer.
- In a unilateral K,
performance is equal to
acceptance
- Some advertisements can
constitute offers where there
is an offer to the world at
large which is accepted by an
act carried out by the offeree
in the mode required by the
offeror
- A unilateral contract is a
5

contract in which there is a


promise on one side, the
consideration for which is not
a return promise, but the
doing of some act.
- In an offer to the world,
notification or communication is
unnecessary
Problem with Unilateral Contract
1. What happens when people
partially perform the contract
there is no answer
2. If you put an ad about the
contract in the paper and its to
the whole world, then you can
revoke in the same manner that
you made the offer.
- bilateral contracts are when there is an exchange of promises
- unilateral contract is where a promise is made by the offeror to a larger mass of
people...there is no promise given in return (an invitation to perform a particular condition, and
usually they are made in the context of rewards) Acceptance is when you fulfill that condition.
So contract is between offeror and whoever fulfills that condition
- Problem with Unilateral Contract - what about partial performance? What if the contract is
revoked while performance is being enacted?: Rule: if the contract has been substantially
completed than there is contract, BUT then you need to interpret what does sustainably
competed mean.

Case/Facts

Issue/Holding

Reasoning/Ratio

Notes

Goldthorpe v
Logan (1943)
C.A.
- Logan guaranteed
hair removal, in an
advertisement,
Goldthorpe received
the treatment, but it
failed.

- Was there a
contract between
Goldthorpe and
Logan?
-

Yes.

1st Cdn case to


apply Carlill
- policy reason =
courts will protect
gullible consumers
1. Was there any
negligence on the part of
the defendant, which
caused loss of damages
to the plaintiffs?
NO.
3. If so, what is the result
in law of a breach
thereof on the part of the
defendant?

Unilateral contracts are accepted


in Canadian law and binding to
whom ever accepts them.
Policy behind this is the
protection of the consumer from
sharp practice.
An offer made to the whole world
through ad, to anyone willing to
accept
Unilateral contract no
notification of acceptance is
necessary, by fulfilling the
requirements you are accepting
Inconvenience sustained by one
party at the other parties
request constitutes consideration
Courts must protect consumers
from faulty promises of companies
Courts of this country will not permit
anyone to escape the responsibilities
arising from enforceable promisewords of Hawkins J.

- A unilateral contract
provides a technique which
ensures the enforcement of
promises made to the world (or
public) at large.

1st Canadian Case to


apply Carlill case
- The ad itself was the
offer, nurse confirming
the result was just an
addition. Had the
nurse said this doesnt
work then it could be
considered as a
counter offer and
possibly a revocation
of the offer. Going in
and getting the
treatment is the
acceptance of the offer
- foreshadows false
advertising laws and
consumer protection
- advertisers should be
aware of what they
promise

- consideration
was the
detriment and
inconvenience
sustained by the
plaintiff
- did not suggest any
exemptions
(Ontario has Consumer
Protection Program)

Invitation to Bid
Case/Facts

Issue/Holding

Reasoning/Ratio

Notes

Harvela
Investments
Ltd. V Royal
Trust Co. of
Canada (1986)
A.C
P competing to buy
shares from D, who
says he will accept
highest offer. P puts
highest offer.
Competitor puts in
referential bid says
he will go higher than
anyones highest
offer.
D accepts
competitors offer.

Was this a call for


a fixed bid or
referential bid
(auction)?
- Does Leonards
referential bid
count?
- in a fixed
bidding sale, a
bidder may not
adjust his or her
price
-highest fixed
bid wins
(plaintiff wins)

A vendor is obligated to specify the type


of offers or bids he is willing to accept in
the sale
RT intention must be deduced from the
words and terms of the invitation
To determine the intention the court
adopts a simple reading of the
investment. RT intended for a fixed
price (since their was ignorance of rival
bids)
The invitation to tender specified that
only fixed bids would be accepted

Tenders must comply with the terms of


the invitation to tender
- An invitation to bid, if specific, may be
characterized as an offer, and can be
accepted by receipt of the highest bid. A
court will construe a term (here, the term
offer) in the context in which it occurs and
give it its ordinary meaning.
- a referential bid is NOT VALID unless
the referential bid is clearly and
unambiguously authorized in the
invitation to bid.

In an auction,
when the
auctioneer
yells sold, the
contract is
made! (or
when he
bangs gravel)

- When asking for highest bidder, Vender can use AUCTION or FIXED BIDDING at
auction, each bidder can adjust their bid in reference to rival bids (winner is who will pay more
then any other person is willing to pay)
o However, in fixed bidding, bidder may not adjust his bid. Each bidder specifies a
fixed amount which he hopes will be sufficient, but not more than sufficient, to
exceed any other bid
The purchaser in a fixed bidding does not necessarily pay as much as the
purchaser was prepared to pay to secure the property

Tendering
Tender documents are often numerous and there will big large books of tendering documents.
These tendering documents tell the people who are placing the tenders what they need to do.
In general it is Crown Corporation, municipal government, and generally government agencies
that place tender document ads in the paper.
They are the party that is most likely to issue public tenders because they want to be fair in
giving the entire public chance to submit the tenders.
The tendering system was put in place to ensure that the governments were fairly and equally
choosing companies without regard to politics.

Case/Facts

R v Ron
Engineering and
Construction
(1981)SCC
- D submits tender and
deposit, but forgot to
include significant costs.
Sent message requesting
to correct tender and

Issue/Holdi
ng

Reasoning/Ratio

Was contract
entitled to
withdraw its
tender and
recover its
deposit?

Ron Engineering is bound by contract A


(call for tenders) and forfeited their
deposit in attempt to revoke tender as
the conditions in Para 13 (Information for
Tenders) were not met.
Para. 13 of the Information for Tenders
states withdrawn tender results in
losing deposit.
Need to preserve the INTEGRITY of the
tendering system.

- Contract
or can
retract
tender, but

Notes
Ron
engineering
changed the
precedent on
the law of
tender
contracts!
-Deposit is the
consideration
to form a
8

resubmit. P says too bad,


build it for what you
submitted. They declined
to do the job. So, they
chose the second lowest
bidder to do the job.
(originally, Ron
Engineering would have
had no recourse because
of the old system)

not regain
deposit.

- a tender invitation and a


tender BID are seen as being a
unilateral contract that is brought
into being almost immediately
after the submission of the BID.
- A contract B which deals with
the terms and conditions of the
building contract is entered into
subsequent to the formation of
contract A

contract?
It forces you
into the next
contract,
contract B
lose deposit if
you do not end
up wanting to
do the job

RON ENGINEERING CHANGED THE PRECEDENCE ON THE LAW OF TENDER CONTRACTS


Justice Estey said that tenders have two contracts

Contract A (bilateral contract, where there are promises and


obligations on both sides)

Snaps into effect on the deadline for the call for tenders
This contract is enforceable at this time because the bidders have accepted the
owners offer for tenders
This means that all the bidders are actually in a binding contract with the owner
for a short period of time, essentially forming an implied unilateral contract.
The non-chosen bidders are released when contract B comes into force.

Contract B

Begins when the owner chooses the bid.


It is often referred to as the construct contract
The chosen tender company is now in a binding contract with the owner.

This case is so important because it created this Contract A and Contract B rule
Up until this case courts generally allowed tenderers to make mistakes on good
faith
Which tenders cases raise this contract A and B rule? All government and quasi government and
all other private with this type of complexity to it will have to subscribe to contract A and B rule.
But simple tenders like getting someone to paint your house will not be subject to this rule.
Contract A: invitation to tender and submission of tender form a unilateral contract where the
terms and conditions are comprehensive and specific (limited to a large contracts@ that
have comprehensive and specific terms) The contract is formed immediately on submission
of the tender at the time and date specified. All bids submitted are bound by Contract A.
Contract B: After the tender period, those who were not accepted could follow certain
conditions and recover their deposit, something the contractor did not do. Those accepted
formed a new contract, the construction K to actually complete the terms of the accepted tender
(Contract B)
PROBLEM: what about the quality of the project?
- now that bidders have a contract (contract A) recourse may be taken if they are
not chosen when they were rightfully supposed to be chosen
- deposit is there to force you into the next step to enter into that construction
contract! If you breach and dont want to do the job anymore, you lose your deposit
(owner can then sue if they really want to to force the job to be done)
(The principle term of contract A is the irrevocability of the bid, and the corollary term is the
obligation of both parties to enter into a contract (contract B) upon the acceptance of the tender.
Other terms include the qualified obligations of the owner to accept the lowest tender, and the
degree of this obligation is controlled by the terms and
conditions established in the call for tenders)
9

Case/Facts

Issue/Holding

Reasoning/Ratio

Notes

10

MJB Enterprises v
Defense
Construction Ltd.
(1999) SCC
- Respondent invited
tenders, 4 received
including MJBs; Contract
awarded to Sorochan,
lowest tenderor, MJB second
lowest tenderor; Contract A
had privilege clause lowest
tender shall not necessarily
be accepted.
- Parties agree that:
Sorochans bid was not
compliant; Contract A/B
analysis from Ron
Engineering; respondent
accepts Sorochans bid in
good faith and their note as
clarification.
- MJB argues that Ss bid
should have been
disqualified
(Privilege Clause in tender
form saying lowest bid may
not be accepted.
- bid accepted was noncompliant
- MJB wants its tender
accepted, it was the next
lowest)

- Does the
respondents
inclusion of a
privilege clause in
the tender
document at issue
allow the
respondent to
disregard the
lowest bid, in
favour of any
other tender, even
a non-complaint
tender?
Crt holds no based
on presumed
intentions of the
parties and the
interpretation of
the documents.
(2) Was there an
implied term
which obligated
the respondent to

accept the lowest


compliant tender?
Crt. Holds no
because of
privilege clause.
(appellant was
awarded
$398,121.27 in
damages, because
court (after
balancing
probabilities)
found that MJB
would have been
awarded contract
putting them in
position they
would have been
had the contract
been formed)

- look to intensionsdid the


intention of the parties focus on
compliant bids

The privilege clause is


compatible with the
obligation to accept only a
complaint bid (The Privilege
clause doesnt override the
obligation to accept only a
compliant bid)

There can be implied


obligation to accept only a
complaint bid

To accept noncompliant bid,


would be contrary to the
expressed indication in the
instructions to tendersthere
was an implied term that only
compliant bids would be
acceptedthere was no basis
that the lowest bid would be
accepted, though (and had no
obligation to choose MJB, either)

- Duty on party
accepting
tender offers
and those
submitting
tends to act
fairly.
-the
clarification
may be seen
as a
referential
bid and
therefore not
complaint.

The respondents argument of


good faith in considering the
Sorochans bid (the non-compliant
one) is no defence to a claim for
breach of contractacting in good
faith or thinking that one has
interpreted the contract correctly
are not valid defences to a breach
of contract

The privilege clause is


compatible with this
obligation to accept only a
compliant bid (in this case)
and it doesnt exempt themjust allows them to select a
bid other than a noncompliant bid (The privilege
clause doesnt require the
person calling for tenders to
accept only the lowest
compliant bid.however
doesnt allow them to accept a
non-compliant bid)

- Tender documents must


express all operative terms
clearly.
-

The privilege clause does not imply that a tender could be accepted on the basis of some
undisclosed criterion simply gives discretion to take a more nuanced view of cost than the
prices quoted in the tenders
11

Ex, they may choose a certain company because they get the job done quicker,
resulting in more money made for owner

2. Communication of Offer
-

Consensus ad idem: Must be a meeting of the minds between offerer and offeree- some form of
acceptance, the offeree must know that offer is made, cant be assumed by the offeree.
to symbolize acceptance, offeree has to take steps communicated by offerer in order to show an
acceptance of the offer.
Must also be an intention to accept the offer!

Case/Facts

Issue/Ho
lding

Reasoning/Ratio

Notes

12

Blair v Western Mutual


Benefit Assn [1972] BC
CA
Corporate resolution passed
suggesting that if Mrs. Blair
retired she would receive
$8000.
In her role as stenographer,
she found out about the
resolution.
She concedes she was not told
directly.
We do not know what would
have happened if this company
did not go bankrupt, it is not
the company saying no it is the
liquidator.
Arguments: Defendant is the
liquidator (company is
winding up) people who
made the offer are no longer
there.
Liquidator argues that there is
no evidence that when she
resigned she didnt do it
because of the promise of
$8000.
Liquidator argues that there
was no intention to make this
an offer because if it were an
offer they would have offered it
in a more direct way.
Plaintiff argues she resigned
thinking she would get the
money; she relied on the fact
that it was a uni-lateral
contract (her resigning was
acceptance of an offer)

Was the
resolution
to give the
appellant
employee
at least two
years
salary upon
retirement,
an offer
accepted
upon valid
considerati
on?
HELD: No.
No
evidence of
promise or
offer
communica
ted to Blair.
No
evidence
Blair relied
upon
promise
did not
retire b/c of
it

No clear offer was present.


She did not indicate that her
retirement (performance) was a
result of the resolution.
There was no direct
communication (there was
intention, but it was not
presented to employee)
She was not relying on the
money in deciding to retire
Offer missing, acceptance is
missing, agreement is missing
(no consensus of the minds).
Court said there was no
consideration she took no
action to get the $8000 (if she
had maybe retired early then
maybe there would be
consideration)
No promise; Bare statement of
resolution; No contract
RATIO:
- a communicated offer to
one person cannot
necessarily be reasonably
understood to be an offer to
another
- if an offer is not
communicated formally or
otherwise to the party, than
it cannot on the facts alone
create or intend to create an
offer capable of acceptance.
must be clearly
communicated before it can
be accepted

Discretion
in Unilateral Contracts
Unilateral Contact can be a
tool that judges use to decide
what is and what is not a
contract.
Realistically the judge could
have decided that this was a
unilateral contract.
If her lawyer had instructed
her to say that she had retired
based on the money then she
may have won.
-When people pass a bare
resolution, unless it is
communicated to the person,
then it is simply a bare
resolution to do something,
not a promise, there was no
communication.

This case may have had


ethical stance but not
legal stance.

- courts generally
presume, based on
compliance with
conditions in the
offer, that the
person had the
intention to accept
the offerthis is not
always the case!

13

Williams v Carwardine
[1833] U.K.
Mary witnessed a murder (it
was her brother who
committed the murder);
Walters brother (the
deceased) offered a reward for
capture of arrest of people
involved in brothers murder;
Marys brother beats her up
and because she is afraid for
her life she gives the
information to police in order
to save herself;
She claims the reward should
be hers because she told, on
what she thought was her
death bead, who committed
the murder
Defendant does not want to
pay her.

Whether P
was
induced by
the reward,
if she
entitled to
recover it?
HELD:
YES
unilateral
contract.
Reasons
for coming
forward do
not matter,
she
performed
the action.
- It was
deemed
she had
knowledge
of offer, b/c
posters all
over the
town.

She knew about the offer


(knowledge of offer is
important);
She was aware about the offer
and her performance constituted
acceptance (regardless of her
motivations).
At no point did she say she did
not do it for the reward.
If the defendant wanted to
qualify the offer he would have
had to do that at the outset.
There was a contract plaintiff
filled the condition of the offer,
and she had knowledge of the
offer!

Rewards are
unilateral offers,
which become
contracts upon
successfully
performance.

A party may
satisfactorily fulfill
contractual
requirement without
being induced by the
promise or by acting
on a motivation
preferred by the
promisor.

Person is deemed to
know about the
reward

Knowingly perfoming
a condition of an
offer is acceptance
- she never said that she
didnt do it for the award

RATIO:
-the motive of an informer of
accepting a reward has
nothing to do with
recovering the reward under
the contract
- what is important is that
they must have knowledge
of the offer. i.e. the offer
must be communicated

14

R v Clark [1927]
Australia HC

ISSUE: was
there a
contract?

Proclamation leads to the


arrest and conviction of the
persons responsible for two
different deaths.
Plaintiff initially wrongfully
convicted of murder and gave
information to clear his own
name.
Had read the reward but at the
time he gave the information
he had forgotten about it.
Gave names of the two people
that committed the murders.
He claimed the rewards
because one of the Officers
suggested he claim it.

HELD: NO!
He does
not get the
reward. He
explicitly
stated
several
times that
the reward
had
nothing to
do with
him
coming
forward.

Clark argues that he fulfilled


the requirements of the
proclamation and argues the
precedent from Williams case.
Motives irrelevant, had
knowledge,

Clarke is not found to be entitled


to the reward:
1.He did not mentally assent
to the Crowns offer; there was
no moment of time at which
there was a consensus of the
minds, or meeting of the
minds
Test of assent in Canada is
objective. Assent (intention in
your mind) is vital.
He had no intention to accept
the offer! Intention is important!
2. All conditions were not
fulfilled; information did not lead
to arrest or conviction of both
but only one.
- in comparison to Williams v
Carwardine, she knew of
reward. Distinguishes
differences based on the
facts he had no intention
to accept the offer because
he says he had no intention
of accepting the offer

In reward cases, the


informant must know
about the reward prior
to giving the
information.
There cannot be
assent without
knowledge of the offer
(both parties must be
in agreement
consensus of the
minds must be
present).
Relation of Cases to Law in
Canada (could be an
example of exam
hypothetical)
- There is no reward law in
Canada YET
- What law would you follow?
- You can choose any of
these cases to support your
client
- Argue/ choose cases based
on whether you think
knowledge is essential; the
act is acceptance,
consensus of the minds, ect.

There can be no acceptance of


an offer if he did not know of the
offerignorance of the offer is
the same as never seeing it or
forgetting about it after he saw
it!!!!
Clark did not act in faith of the
proclamationas stated by him!
RATIO:
- where the offer invites
conduct on the part of the
offeree as the acceptance of
the offer, that conduct must
be undertaken not only with
knowledge of the offer but
with an intention to accept
the offer
- the motive of the offeree in
performing the required
conduct requested by the
offerer is immaterial
(irrelevant)
- This is old law and may not apply today - this case is merely persuasive, not binding.
- There is no binding Canadian case - can take into consideration either Clarke or Williams v.
Carwardine.

15

3. Acceptance
(A) Acceptance by Conduct
-

Offerer confirms the offeree the power to accept that offer


To exercise that power effectively offeree must communicate an unqualified commitment to
the terms of the offer
No meeting of the minds, unless the acceptance is a mirror image of the offer
If the acceptance varies at any respect it will be treated a s proposal of new terms
Not initial offerer stand to choose to accept new offer
If counter offer does not get acceptance, cannot go back to initial offer in existence

Questions:
- Are they accepting?
- Has the acceptance been communicated?
- Rejecting?
- Counter-offering?

Case/Facts

Livingstone v
Evans [1925] Alta
SC
Vendor wants to sell land
for $1,800; Buyer
counter-offers $1,600 and
asks what is your best
price; Vendor replies by
telegram: cannot reduce
price; arguably
reinstating the original
offer; buyer still wants
original price and accepts
the Vendors price.
The Vendor sells the land
to a third party
throughout all of this.
The Vendor argues that
the Buyers counter offer
made his (the vendors)
original over null and
void.
Buyer argues that it was
not a counter offer and
was an inquiry, still wants
the original price.

Issue/Hol
ding
ISSUE:
was Ps
counter
offer a
rejection
of Ds
offer?
Was there
a biding
offer?
HELD:
YES there
was a
binding
offer! For
P judge
says
cannot
reduce
price is
rejection
of
counteroffer, but
implies
reiteratio
n of
original
offer.

Reasoning/Ratio

Notes:

Defendants rely on hyde v rench (making of


a counter offer results in a rejection of
the original offer)
Stevenson Jaques & CO v McLean 1880,
distinguished from hyde v rench---it was
a mere inquiry
Because the Vendor said, I cannot reduce the
pricesuggests the price is still standing, and
thus the offer was not void.

Making a
counter-offer
results in a
rejection of the
original offer.

If he had replied that no I will not accept


1,600 this would change things and then
argument for null contract based on counter
offer would be stronger.
The test for determining this in the case is: the
intention of the vendor and what a reasonable
person would conclude, and the language
used.
There was a binding contract for the sale of
this land to the plaintiff of which he is entitled
specific performance.

The offeror is
the only person
who can revive
the original
offer.
Rejection of the
counter offer
nullifies the
original offer,
however,
If the rejection
includes a
renewel of the
original offer
then the right
to bind the
offeror remains

Cannot reduce price was seen to mean that


Evans could not accept the counter, but he did
not revoke the original offer and as such
Livingstone in accepting the original offer
made the contract binding. He was standing
by his original offer
Cannot reduce price was held to be a
renewal of the offer based on the language,
intention of parties, and the circumstances.
The judge then ordered specific performance.
- a mere inquiry is not a rejection of the offer!
RATIO: The making of a counter-offer is a
rejection of the original offer (Hyde v.
Wrench - 1840)
16

exception: court may decide original


offer is still opened based on the
language used and the actions of
the parties
- Rejection of the counter offer nullifies the
original offer, however,
If the rejection includes a renewal of the
original offer then the right to bind the offeror
remains
-

Battle of the Forms


Case/Facts Battle of the

Forms

Butler Machine Tool Co. v


Ex-Cell-O Corp. [1979] CA

About the sale of a big machine;


wont be delivered for 10 months
Document 1- offer to sell machine:
75535L terms and conditions on
back, says terms and conditions
should prevail and had an
acceleration clause because in
10months machine might be more
expensive so gives them a right to
sell it for the value on day of
delivery
Document 2- The Buyers place an
order for the machine but their
document had different terms and
conditions, and it had a tear off
which if sent back will mean that
the vendor is accepting their terms
and conditions, in this document
there is NO acceleration clause and
the assumption was that the price
is fixed.
Document 3- The sellers send back
the tear off and also a cover letter
accepting the buyers offer but says
as per our terms
In other words: Battle of the
forms sellers placed a list of
terms and conditions on the back
of their offer; buyers replied with
their own terms and conditions;
buyers order form had a tear-off
slip upon which the sellers were
invited to accept the order; sellers
completed and returned the tear
slip upon which the sellers were
invited to accept the order.

Issue/Hold
ing

Reasoning/Ratio

Issue: On
whos
terms was
the
contract
concluded?

Clarity of Contracts that Exists:


- The Seller- original offer
- The Buyers tear off- counter offer thus killing
original offer
- The seller sending back tear off signed- was
acceptance of the counter offer which was on terms
of the buyer
Arguments:
Vendors- They deliver it, and upon delivery claim they
are entitled to an additional 2800L based on their
acceleration clause because of cover letter.
Buyers- The buyers say that this was not the
agreement and that their tear off was a counter offer
and refuse to pay the extra money.
IN FAVOUR OF THE BUYER TAKES TRADITIONAL
APPROACH 3 approaches:

Held:
Buyer
succeeded
on appeal
because
they
brought
attention
to the
material
change.
there
was a
battle of
the forms,
the buyers
countered
original
offer,
therefore
negating it
(Hyde v.
Wrench, in
Livingstone
supra)

(1) Last Shot Rule (also called performance


doctrine) Last terms and conditions should prevail if
not objected to by the other part, TRADITIONAL!
(2) First blow if seller is explicit, buyer must
bring material change to their attention, --> must
draw attention to the change!
(3) Construe the terms of both documents
together; if you have conflicting terms, replace them
with what is reasonable (not applicable to this case
because buyers brought material change to the
sellers attention).
If a material change in the contract is brought to the
sellers attention a valid counter-offer has been made (i.e.
tear off slip).
If you have a term that you want to prevail you must
repeat it over and over again

Ratio: When there is a battle of forms two ways to


analyze:
1.TRADITIONAL OFFER AND ACCEPTANCE last
shot rule; there is a contract as soon as the last of
the forms is sent and received without objection
being taken to it. (when in favour of buyer)
2. Lord Denning: look at documents as a whole and
make a decision. (when in favour of seller)
Not firm ratio: cases will be decided on the
basis of the particular facts and circumstances.
17

note 2 International Convention- convention says rules related to contract: 1. Reply to offer that
seems like an acceptance that puts forth rules, additions, or limitations is a rejection of an offer, and
is a counter offer. 2. Reply to offer and with conditions that do not materially alter the terms is an
acceptance. 3. Alterations that deal with price, ect. Then there will be no contract.
- If you have a condition you want to prevail, you must reiterate it over and over again.
- (Lord Denning) USE WHEN IN FAVOUR OF THE SELLER! this is a tough type of case; the court
could of decided there was no contract, because there was no agreement on the terms and conditions.
He said that the buyer should have pointed out the increase-price clause because it was incompatible to
them
- Suggests that the traditional analysis is out of date- proposes new traditional, you should take the
terms and conditions
and put them together, if they can be reconciled, so be it, if they cannot be reconciled such that they
are mutually
contradicting, than the conflicting terms are to be scrapped and replaced by reasonable conditions
- Says documents should be considered as a whole

- His view was not accepted by the panel nor by current Canadian courts
Case/Facts Battle of

Issue/Holding

Reasoning/Ratio

Notes:

ISSUE: is the P
(seller) bound to
arbitration clause
even if they did not
acknowledge it?

-Application for arbitrary clause was


never accepted
-Court finds that neither party agreed
to any terms; only real agreement was
about the price and specs.
- neither party considered any other
terms other than those bound on the
face of the documents (price and
subject matter)

BATTLE
OF THE
FORMS
case
(subsecti
on of
accepta
nce)
-used
similar
approac
h as lord
denning)

the Forms Canadian

CANADIAN EXAMPLE
Tywood Industries
LTD. v St. AnneNackawic Pulp &
Paper Co. LTD.
[1979] Ont HC
Battle of Forms #2
Sellers (p) send bid which
contain no arbitration
clause. Buyers send back
form, with Arb. Clause (did
not draw attention to this
change). Sellers do not sign
form or return it, but deliver
goods.
Help to understand case:

T = Vendor, SA = Buyer
SA invitation to tender =
RFQ (contained 13 terms
without arbitration clause).

T replies with quotation


that contained 12 terms
w/out arbitration clause.

T revises quotation with 12


terms with no arbitration

SA send 2 purchase orders


including 19 terms with
arbitration clause

T doesnt sign or send back


Purchase
orders
but
delivers goods.
SA finds fault with goods and
wants arbitration.

HELD:
no
arbitration clause.
Not binding- not
pointed
by
defendant.
Last
blow wins

RATIO: Where the terms and


conditions are not mirror images,
then attention should be drawn to
the terms and conditions in order
to make them binding.
- Under the traditional analysis,
there would be a strong argument
that the defendants purchase
order constituted the last shot and
the sellers performance of the
contract constituted an acceptance
of those terms

Was the arbitration clause


valid and does it apply?

T: PO not signed, no
acknowledgment
of
SAs terms. PO was
void b/c it was never
sent back. T terms
prevail.

Crt says: look at the


conduct this is a
typical battle of the
forms situation.

Crt holds: conduct of


the parties indicated
Material change must be brought to the
that
neither
party
considered any terms attention of the other party (application of
important other than Butler if tear off returned).
the ones found on the
face of the documents
In determining the battle of forms, look to the
(being
the
conduct of the parties
specifications and the
Mirror image rule- acceptance must mirror
price) no meeting of
the terms of the offer. Forms that are
the
minds
on
the
exchanged, if there are variant term within
arbitration clause but it
them, the last document is not a true
was there on the key
acceptance, but constitutes a counter-offer.
terms.
Basically, forms exchanged cannot have

Contract formed on the


different terms and form a valid contract.
basis of no arbitration
Forms must be congruent to do so.
clause

Performance rule if performance has been


tendered and accepted

When you have two documents with


different
terms,
only
those
terms
negotiated on will be upheld.
If there are unique terms in the contract you must
draw attention to them

18

- because it so often happens that two documents do not coincide, the rule is that the major terms
must comply. This is in recognition that often times, contracts are not mirror images of each other.
- If there are unique terms in the contract, you must draw attention to them to the other
party. Arbitration clause should have been brought to the attention of the seller.

E-Commerce
Case/Facts

ProCD v Matthew
Zeidenberg and
Silken Mountain
Web Services, INC.
[1996] US C.A

ProCd the Plaintiff has


compiled phone directory
software.
Sells a version of the
software called Select Phone
(trademark) on CDRom disks
ProCd offers a comparatively
cheaper version of their
software to the general
public.
Practices price
discrimination selling its
database to the general
public for personal use at a
low price while selling
information to the trade for
a higher price.
To make price discrimination
work the seller must be able
to control arbitrage
(profiting from different
prices in different markets)
to do this they turned to the
intuition of contract.
Every ProCd box containing
its consumer product
declares that the software
comes with restriction states
in an enclosed license. This
license which is encoded on
the CDROM disks as well as
printed in the manual, and
which appears on a users
screen every time the
software runs, limits use of
the application program and
listings to non-commercial
purposes.Presses accept
to terms and conditions
The defendant bought a
consumer package of Select
Phone (trademark) but
ignored the license he formed
a company and resold the
information on the Select
Phone database.

Short FACTS: E
Commerce Shrink-wrap

Issue/Hold
ing
Issue: Is a
shrink-wrap
license a
valid
binding
contract?
HELD: for
ProCd(appe
llant)
- can put
terms
inside box,
as long as
the product
can be
returned.

Reasoning/Ratio
Arguments:
Plaintiff argues that copying the application program onto its
hard disk violates the copyright laws
Defendant argues that there must be printed terms on the
outside of a box for a contract to stand, he says he is not
bound by the traditional conception of contract law.
Says that contract was complete at cash register and that the
license happened after the contract and therefore are invalid.
Reasons:
Looking at the fact that terms and conditions can apply
afterwards.
For instance with a plane ticket, or a movie ticket there is a
notice that there are terms and conditions you must abide
by, so we are DEEMED to know that terms and conditions
exist and we should look into them.
Judge says that in Ecommerce we are DEEMED to know that
terms and conditions will be in the box, and that we must
agree to the terms and conditions and if we do not than we
can return the items.
We need to extend the traditional concept of contracts to
apply to E-Commerce, the contract does not end at the cash
register it ends when the individual accepts the terms and
conditions on the computer.
Consumer goods work the same way as something like a
warrantee.
Someone who wants to buy something pays and walks out
with a box
Inside the box is a leaflet contains some terms and the most
important of which usually is the warranty, read for the first
time in the comfort of home (not on the outside for the box)
By the defendants arguments, the warranty in the box is
irrelevant; in reality, however, every consumer gets the
standard warranty implied by the UCC in the event the
contact is silent
Unified Contract Code (American law) s.2-204(1): a contract
for sale of goods may be made in any manner sufficient to
show agreement including conduct by both parties which
recognized the existence of such a contract (equivalent to
sale of goods act Canada)
ProCd proposed a contract that a buyer would accept by
using the software after having an opportunity to read the
licence at leisure. So although the district judge was right to
19

Licenses
- Zeid. Bought ProCDs
product and formed
company.
- Z argues the terms
forbidding resale of the
product are inside the box,
and therefore constitute
adding terms after K
formed. Won at trial

say that contract can be, and often is, formed simply by
paying the price and walking out of the store, the UCC
permits contracts to be formed in other ways.
ProCd proposed such a different way and without protest the
defendant accepted.
- S.2-606 UCC, which defines, acceptance of goods,
reinforces this understanding. A buyer accepts goods under
s.2-606(10)(b) when after an opportunity to inspect, he fails to
make an effect rejection (the opportunity to return goods van
be importance; acceptance of an offer differs form acceptance
of goods after delivery)
RATIO:
- Offerer may invite acceptance by conduct and may
propose limitations on the kind of conduct that
constitutes acceptance. The offeree may accept by
performing acts that the offerer proposes to treat as
acceptance
- Computer software Shrink Rap terms and conditions
are acceptable i.e. In modern contract the seller may
sell a product that incorporates terms and conditions
not immediately visible but which the buyer accepts by
conduct
. Buyer can prevent the contract by returning the
product

NOTES:
Shows how the court extends the rules of traditional contract law to E
Commerce situation
American case, but suggests what would happen in Canada as well.

(B) Silence

- Important Supreme Court Case


- Very close call here - arguments of both sides are relevant - trial judge found for
Helicopter Exploration Co.
Case/Facts
Issue/Hold Reasoning/Ratio
ing
- Was this
Arguments: Dawson argues it was a bilateral contract
binding and
and Springer breached the contract
Dawson v Helicopter was it a
Springer argues it was a unilateral contract and that any
bilateral or
Exploration CO.
offer contained in the correspondence and in particular
unilateral
[1955] SCC
contract
the letter of March 5 called for an acceptance not by
FACTS
formed by
promise but by the performance of an act, the location
- D agreed to show H land
the series
for the claims by Dawson for Springer
with mineral deposits, in
of
return for cash (10%)
correspond Also argued that the lack of a reply and the time lapsed
- H agrees to get a pilot and
was an abandonment of the contract
ence by
that they will go before end
Dawson
Argues
that a contract must be clear and unequivocal and
of summer
and
that
these
correspondences did not clearly put forth a
- H then decides that it
Springer?
contract.
would be an impossible
- Does
Reasoning:
journey (calls deal off) to
silence
On the argument of the Appellants silence, coupled with
which D does not reply
over two
the steps he took immediately upon his return from the
- D finds out after being
tear period
away that H went and
Marshall Islands, sufficiently supports a conclusion that
constitute
staked the claims (w/ other
he, did not at any time, intend to abandon his rights
abandonme
company)(however, claims
nt of
under the contract.
he went in good faith)
contract?
Justice Rand: unilateral contract is one that is
20

- D files lawsuit for breach


of contract
Help to explain:

1931 D discovers mineral


deposits in BC, Jan 1951 D
engages in negotiations with
HE.
exchange
of
correspondence between the
two.

March 5, 1951 Respondent


proposed a mining exploration
(OFFER by HE);

April 12, 1951 (D agrees to


arrangement); and will take
leave when pilot arrangement
finalized (ACCEPTANCE)

June 7, 1951 (HE says venture


not feasible, make other
arrangements)

D doesnt reply

August 1, 1951, HE goes


ahead with exploration.

1953 HE goes ahead with


Development
1953 HE goes ahead with
exploration. D finds out and brings
action to court.

HELD:
there were
obligations
on both
sides
bilateral K.
- Heli. has
obligation
to Dawson
- no
abandonme
nt of
contract,
no
revocation
of Ss
unilateral
offer.

preformed by offeree, but in this case, there is


requirements on both sides. bilateral K
Court says it was not unilateral because promises were
exchanged and because what Dawson had to do (his
performance) could not be preformed without the
participation of Mr. Springer. (Moorcock doctrine use
of good business sense)
Also said that traditionally the majority of courts find a
Bilateral contract if preferred to a unilateral contract.
Because a unilateral contract can be pulled out and
revoked out of at the last minute before performance is
completed. This makes them problematic.
When addressing the question of whether a court must
be clear and unequivocal he said that an acceptance
does not need to be written it can be implied.
Acceptance was from the languagedoesnt need to be
expressed acceptance.
Says law has outgrown its formalist instinct with
obligations imperfectly expressed We instinctively
know that the obligations are there between each party.
RATIO:
- Silence alone does not constitute abandonment of
K must look to circumstances of each case.
- Acceptance must be clear, but need not be
expressed, and may be implied in the language and
conduct of the parties
-courts will tend to treat offers as bilateral (not
unilateral) for business efficacy.

NOTES: Important Case because Court made an incremental change to the law, the law of
contracts evolved.
IMPLIED ACCEPTANCE introduced [contracts no longer limited to] CLEAR AND
UNEQUIVOCAL ACCEPTANCE

Felthouse v. Bindley (1862) (English Case)


This was exam question last year
Facts

Issues
Remedy

Felthouse (Plantiff) to buy a horse from nephew.


If uncle hears no more he will assume that there is an agreement on the price for buying
the horse (miscommunication of the price, so uncle asked to meet down the middle).
Bindley (Defendant and Auctioneer) mistakenly auctions off the horse; admits mistake.
Nephew and Auctioneer write a letter to Felthouse apologizing for selling the horse (this
shows that both the nephew and the auctioneer thought a contract had been formed)
Felthouse sues.
Can silence constitute acceptance? Did the nephew provide enough acceptance to
constitute a contract?
Plantiff brought action against the auctioneer for the conversion of the horse

Sought
Argumen
ts

Plantiff argues that he had a contract with his nephew for the horse, thus he had property
title in it.
Defendant argues that the defendant did not vest a property title in the horse at the time
of the sale.
21

Holding
Reasons

The contract was completed after the sale of the horse when the nephew wrote a letter to
Felthouse apologizing for selling the horse.
NO CONTRACT! Nephew never accepted offer.
The uncle did not have a right to the horse; there was no contractnephew did not accept
the new price fo the horse
There was no meeting of the minds on the price;
Nephews silence does not constitute acceptance of uncles price.
Despite intentions there was no valid contract because he didnt communicate his
acceptance of what can be considered a counter offer
We need something more than a failure to reject an offer to constitute acceptance.
Offeror cannot stipulate that silence is binding acceptance
So, defendant sold nephews horse, not uncles.

Ratio/
Rule

Comment
s

Silence does not constitute acceptance. You need more than


failure to reject an offer to make a contract. You must
communicate acceptance and silence is not a mode of acceptance

In most circumstances silence does not constitute acceptance (some do)


In most cases acceptance need not be expressed but must be clear
Relates to Tug Boat and Dawson Case All about SILENCE
- have to be able to point to a specific point in time when the horse was sold, for business
efficacy.

St. John tug boat co. v Irving refinery ltd. (1964) SCC
Procedural History
At trial they find for Saint John and they award them damages
Irving Appeals and at the Court of Appeal the judge subtracted the handling charge and reduced
the amount that Irving had to pay Saint John.
They then appealed to the Supreme Court of Canada (and LOST AGAIN)
Facts
Saint John had an agreement to supply Irving with Tug Boats
Irving needed the Tug Boats to guide in the Kent Lines boats (subsidiary of Irving)
There was an agreement between Saint John and Irving to provide stand by on a daily basis for
$450/ day and when Irving doesnt need Saint John they are free to do other work and then
deduct their profits (but charge 10% handling charge) from the costs invoiced to Irving.
The agreement was initially one month and then extend to the middle of august.
No problem with paying invoices until august when Irving gets a new President.
Silence between August 15th and the end of February (boats are still standing by, and invoices
are being sent to Irving, but Irving is not paying.
Irving does not want to pay for the extra standby boat because they argue there was no
acceptance of this contract for the boats to continue. (The courts didnt accept this argument)
Issues
Did the Irving Refineries conduct during the months in question constitute a continuing
acceptance of these offers as to give rise of a binding contract to pay for the standby services of
the tug at the rate specified in the invoices
?
Arguments
Appellant argues that there was a contract
Respondent argues they did not communicate acceptance.
Holding

22

Irvings conduct showed they accepted renewal of contract after the elapsed date. There was a
contract.
Reasons
The test of whether conduct, unaccompanied by any verbal or written undertaking, can
constitute an acceptance of an offer so as to bind the acceptor to the fulfillment of the contract:
If A allows B to work for him under such circumstances that no reasonable man would suppose
that B meant to do the work for nothing. A will be liable to pay for it. The doing of the work is the
offer, the permission to do it, or the acquiescence in its being done, constitutes the acceptance.
Neither the absence of an express agreement nor the fact that the respondent did not consider
itself liable to pay for the stand-by service can however, be treated as determined the issue
raised y this appeal
The respondent must have known that the boats were being kept standing by for its use and
the respondent made no effort to either dispense with the service or complain about the charge
Respondent must have been known the boats were on standby.
And the Irving acquiesces which means their conduct overrides their silence.
The act of the tugboat services remaining on standby for the oil boats was the offer
(this act)acceptance was them saying nothing, knowing they are there waiting for
them, and doing nothing about it, and not disputing the invoice right away!
Respondents conduct was sufficient of acceptance and trial judges decision was restored! There
was a contract
Ratio/ Rule

Sometimes acceptance can be inferred by conduct of the parties as well


as words. (Irving used the boats) You do not need verbal or written
undertakings- here silence was the mode of acceptance
-Irving continued to get invoices and did not complain- KEY CONDUCT!
- apply an objective test - find out the parties intentions by what they
have done - acquiescence
Acquiescence by conduct may result in a contract being formed.
Conduct by Acquiescent = Acceptance
Acquiescent (silence) by conduct can be construed as acceptance
Comments

Builds on Felthouse case and the ratio of silence. Sets out that although silence cannot
be acceptance, silence with conduct can be acceptance.
(exception to the rule that silence does not constitute acceptance)
-Silence and conduct formed acceptance
-A way to GET AROUND THE FELTHOUSE V BINDLEY rule that says silence does not form
acceptance conduct of not complying is how this case is distinguished from a mere
conduct of silence
- SC case very authoritative!

Eliason v. Henshaw (1819) US


Facts

Important!!!!!! (E is seller, H is buyer)

H offered to buy Es flour and asked for acceptance in Harpers Ferry by return of wagon
E sent acceptance by mail to Georgetown, not by wagon
23

H acknowledged acceptance letter but withdrew offer. E never got withdrawal-delivered flour
The Defendents said that they did not receive an acceptance, and so they bought flower from
someone else, so they will not pay the Miller for his flower.
_______________

offeror wanted acceptance by return of wagon (because the offeror is the master of
the offer)

- day after received by defendant (h)write a letter accepting the offer, but sends it
to Georgetown (where the buyers were) by mail on feb 19, not by wagon.

- since he hadnt received response by wagon, he went to buy his flour elsewhere.

- sellers still send flour to buyer, and buyer refused to accept the flour, so sellers
bring an action for damages
Issues
Argumen
ts

did the sellers acceptance by the offer by mail as opposed to wagon mean that the buyer
was no longer obligated to purchase the flour?
The Miller is going to argue that the wagon was not going back to Harpers Ferry and the
Defendants knew that the Wagoner was not going to return to Harpers Ferry because the
Wagoner was an employee of the Miller.
Could argue that because they knew the wagon was not going back, what he did was not
unreasonable.
Also could have argued that the Offeror must make the act of accepting reasonable
The courts did not agree with these arguments.

Holding

Defendants argue that the Miller did not send the acceptance back.
acceptance was not sent by proper mode; no contract---buyer

not obligated

to buy.
The acceptance was received in a different place than what was specified by the Offeror, and
by the wrong method
The plaintiff should have taken the efforts to get the acceptance to Harpers Ferry
Ratio/
Rule

The offerer is the master of the offer and sets out the terms and conditions of that
offer which the offeree must accept on those terms and conditions made by the offerer and any
departures from those terms invalidates the offer unless such departure is agreed to by the
person who made the offer.

Commen
ts

IMPORTANT CASE it lies out the black letter law.

24

Two situations where silence is assessed as a mode of communicating


acceptance:
1. CONDUCT OF OFFEREE--when silence of the offerree is reasonably
understood by the offeror that silence constitutes acceptance (its
conduct that you look at)
2. OFFEROR in making his offer has waived the requirement to
communicate acceptance (more cases like carbolic smoke ball)
EXCEPTION! Postal acceptance rule: exception that it has to be communicated
when you drop your acceptance in the mailbox, it is on that date that
acceptance was communicated. Not the date that the offeror receives the mail.
has to be the most appropriate forum---if your offer came through mail or
you are told to send through mail (telegrams are treated same as mail
cases)
if offeror wants NOTICE OF ACCEPTANCE, they must have acceptance
come to their attention
4. Communication of Acceptance
A) Mailed Acceptances
General/Receipt Rule:
Acceptance occurs at the time and place where the offeror finds out that the offer was
accepted.
If acceptance is lost in the mail, it is still a contract
If offeror decides to revoke his offer and sends that notice of revocation in the mail, and
offeree gets it before he sends his acceptance, then offer is revoked
o If he does not get it and sends his acceptance in the mail, his acceptance trumps
the revocation because acceptance takes effects once it is dropped in the mail
o Revocation, however, comes into effect when offeree actually receives it!
If offeror wants NOTICE OF ACCEPTANCE, sending acceptance is not sufficientmust come to
their attention
Which juristiction is the contract formed (when in mail and has two separate jurisdictions)
- wherever the acceptance is mailed (ex if person in Ontario mails to Detroit, Ontario
laws!)
Summary of what we have learned:
1. That acceptance cannot be silent, but
2. that acceptance can be based on conduct, and
3. now we look at acceptance of mail and instantaneous communication

Household Fire & Carriage Accident Insurance Co. v. Grant (1879)


CA
Case that says postal rule exists
Facts

G applied to buy shares


- H accepted his offer and mailed his confirmation which G never received
- Company went bankrupt, and when the liquidators tried to recover the balance of Gs
shares, he denied being a shareholder
25

Issues

Was there a contract despite the fact that defendant did not receive letter of allotment?

Decision

- YES! Acceptance effected when letter dropped in the mail. Acceptor concludes the
contract by posting the letter
Thesiger L.J: Majority: The contract is complete when the acceptance is put in the
mailbox.
Bramwell L.J:Dissent says: no communication means no acceptance leads to the meeting
of the minds question.
Meeting of the minds is French law and we need to be careful with this argument
Court says to consider to facts:

Reasons

- Was the letter posted (YES)


- Was the letter received (NO)
This creates the problem of whether or not Grant owes the company money.
Post office was the agentwhen it was posted, the cotract is complete. The
offeror as the maker of the offer can always avoid the postal rule by stating a
term in his offer that he must obtain notice of acceptance, but he did not do this.
This was not a term of the offer. The offeror trusted the postal service as a
means of communication, so onus on him if he doesnt receive communication,
he can ,make inquiry to other side. RISK LIES WITH THE OFFEROR!!
(dissent: thought that when theres an offer, the acceptance must be communicated to the
offeror.doesnt like the postal acceptance rule)
Trial Court found that Grant was a shareholder, and that acceptance was concluded at time
the letter was mailed (postal rule).
- They were using the post office; they agreed to use the post office as their agent.
Even though there was no consensus it is justified by the rationale that BOTH parties have
agreed to use the post office as their agent. This is the same as giving notice to Mr. Grant
because Grant allowed the post office to be the means of communication.
Ratio

Postal Rule: If both parties agree, the post office is a common agent for both of them.
If it is not explicit, it must seem reasonable to send it by mail. Postal rule only applies
to acceptance. A contract is concluded where and when the letter of
acceptance is posted where the post office is the agent for both parties
The contract is completed as soon as the letter accepting the offer was put
into the postregardless of the fact that the letter was never received

The postal acceptance rule only applies


1.where the offeror does not prescribe a particular mode of communication,
2. does not apply where it would lead to an absurdity or inconvenience,
3. It only applies where the letter of acceptance is properly stamped and has
the right address
Comments

The postal rule (only applies to snail mail)


(1879) (C.A.), p. 91
If a certain amount of time has passed the offeror should know that something went wrong
they bare the onus to check back with the offeree. (Generally, offer is considered valid
only for a reasonable amount of time.
Generally, whatever mode the offer was made the acceptance should be the same (if no
mode of acceptance is stipulated). But, also use good business sense (i.e. verbal offer &
registered letter of acceptance).

In order to have a postal rule, not only is the K concluded when the letter is posted but you also have
to have an agreement between the parties stating that the post office will be the agent for the
parties the agreement can be expressed or implied (if offer sent by mail it is logical to accept by
mail)

26

Postal Rule originated in Adams v. Lindsell 1818

Holwell Securities v. Hughes (1974) CA


Facts

Issues

Held

Reason
s

-Hughes granted Holwell option to purchase land, exercisable by notice in writing within six
months. Holwell posted letter posted prior to the deadline, but it never arrived.
-Defendant refused to sell the land because they did not get the letter.
-D never received letter, but a letter to the defendants lawyers from the plaintiffs lawyers WAS
received, and the defendants lawyer talked to his client.
- Case had an option caluse which is a legal term
- contract stated that the offeror required notice in writing
Does the Postal rule apply here?
ISSUE: did the plaintiff exercise an option to purchase the premises by posting a letter to the
defendant which he never received???
NO! required notice!
Option clause (legal form of art, it is a lawyer type clause, need to be concise) does not fall
under the postal rule.
Acceptance not tendered because the letter was not received in writing.- the act of posting
the letter was not enough-acceptance had to be communicated to offerer
Postal Rule does not apply because both parties did not agree to use post office as
common agent.
Court says 2 ways to examine this issue:
1. Short Rule- notice in writing implied that they needed actual delivery (if it was acceptance
in writing postal rule would have appliedbut NOTICE in writing means offeror must be notified
of the acceptance!)
But Plaintiff argued that the postal rule applied as main argument. But courts says is a little
different because of the notice in writing clause which requires actual delivery.
So, the Plaintiffs then argue that the defendants knew the letter was coming because the
defendants solicitors knew because the Plaintiffs sent two letters one to the solicitors and one
to the defendants stating that they wanted to exercise their option.
The plaintiff did not follow the options requirement for notice (i.e. actual notice).

Ratio

- where you have an option and it says notice in writing, the postal
rule will not apply because the law relating to options is that the
grantee must comply strictly with the conditions stipulated..
acceptance in writing (postal rule) does not equal notice in writing (actual direct
communication)
- notice in writing = acceptance must actually be delivered
- >Notice in writing= only received when acceptance actually reaches offeror.
>Acceptance in writing= received when mailed.
- where you have notice in writing, the postal rule will NOT apply guarantee must comply
strictly with the conditions stipulated

Qualifiers to the Postal Rule in cases decided since then:

1. Postal rule only applies when offerer does not prescribe a particular mode of communication
2. Where application of rule would lead to an absurdity or manifest inconvenience it will not apply
3. Applies only when the letter of acceptance is properly stamped and addressed (i.e. Adams v.
Lindsell - supra)

POSTAL RULE APPLIES ONLY TO ACCEPTANCE

27

Instantaneous Communication (faxes, emails, etc)


-

contract is formed when acceptance is received by offeror (different than


postal rule)

email when it is openit is offerors responsibility to read emails in a responsible manner

Brinkibon Ltd. v. Stahag Stahl Und Stahl Warenhandlesgesellschaft


mbH (1983) English Case (Instantaneous Communication)
Facts

Issues

Holding

Argumen
ts

Reasons

- buyers suing sellersmust show that contract was formed in UK so that UK law applies, not
Austrian.
Parties in London are trying to say the contract came into effect in London the second they sent
the telex to Vienna (as per the Postal Rule).
Brinkibon wanted to buy steal from Stahag
May 3rd counter offer from seller (in Vienna) to buyer (in London) indicating buyer must open a
letter of credit.
May 4th the buyer accepts by opening the letter of credit and sending telex back to seller (is
received same day)
Where did acceptance occur?
When a telex is instantaneous communication where is the contract formed?
ISSUE: Is an acceptance of an offer by telex from London but received in Vienna a contract
made within the jurisdiction within the UK?
for respondent K formed in Austria
- acceptance by instantaneous communication was received in Vienna!
Ratio: A contract is formed when acceptance of an offer is communicated by the offeree to the
offerer when dealing with instantaneous communication
Brinkibon said that contract was formed in London (needed to show that contract was made in
Britain jurisdiction in order to sue)
But rule form Entros says that Contract was formed in Vienna (where acceptance was received)
Brinkibon try to argue that their case does not apply to Entros Rule because they say that when
they opened the letter of credit in Britain they were accepting the contract by conduct the
telex was just confirmation of their already completed acceptance.
But Court says that was not acceptance, the telex was the acceptance. Thus, the contract was
completed where the acceptance was received
The letter of credit was not conveying their acceptance to the seller and the telex was the
actual acceptance.
-

although in cases where two parties agree where contractual obligations occur by
instantaneous communication, general rules should apply, but it isnt a universal rule
(not in all cases) no universal rule can cover all modes of instantaneous
communication, when conflict, look at reference of parties, sound business practice, and
judgements of where the risks should lie.

Concurring judge: Telex should be treated like any other forms of instantaneous
communicationwhen a message has been received on an offerers telex, it is not
unreasonable to treat it as delivered to the principle offeror because it is his
responsibility to arrange for the prompt handling of messages in his office- therefore,
once it is received by offeror, because delivery failures can be seen by person sending so
they can resend.

The telex was instantaneous communication contract formed in Vienna, where it was
received! Offeree is in a better position to know if acceptance has been sent during
instantaneous communication. Party who was accepting knows if the fax did not go through.
Ratio

-In cases of instantaneous communication the contract is completed


when and where (jurisdiction) the acceptance is received by the
offerer, unless the failure of the offeror to receive the communication
28

of acceptance sent by the offeree results from the fault of the offeror
or from a defect in the communication with respect to which the
offeror should be deemed to have assumed the risk
- unless the failure by the offerer to receive the communication of acceptnce results in a fault
from the offerer or from a defect in the communication- offerer should be deemed to have
assume the risk.
- This is not a universal rule, in cases where there was an error

receiving an email, or other situations, look to the intentions of the


parties.

This rule applied for instantaneous communications

Entores v. Miles Far East Corp. (U.K.) 1955 - In instantaneous communications, a


contract is formed at the place where the acceptance is communicated.
(Basically, at the location of the offeror.) Not often that the Place where the offer is
accepted is important.
Postal Rule does not apply to instantaneous communication Instantaneous
communication is different then postal rule. Acceptance is communicated
when it is received.

Rudder v. Microsoft Corp. (1999) Ont SCJ (Instantaneous


communication)
Facts

Issues
Arguments

Holding

customers of MSN had complained that Microsoft breached contract by taking money from credit cards
and not giving the customers a proper accounting.
Class action suit and was looking for 75 million dollars
Now these two guys bringing suit on their own (no more class action)

R clicks I agree in an online agreement with M


- R wanted to bring wanted to bring action in Ontario
- M said you can only sue us in the State of Washington which is the jurisdiction in the
forum selection clause
- R says forum selection clause is fine print
--clause said agreement governed by the laws of state of Washington!
(member agreement required all members electronically executed prior to receiving
services of Microsoft)
was contract formed on I agree icon on computer screen , did acceptance apply to all
terms set out in agreement including those not read by the offeree?
Microsoft wants a stay of proceedings (haulting further legal process in trial) because
within the contract fine print it says that the jurisdiction for cases to be brought are
regarding the contract is Washington, so these two guys cant bring issue up in Ontario
Microsoft says that by accepting the contract parties are accepting the exclusive
jurisdiction of Washington and that the agreement was thus governed by the laws of
Washington.
The two guys argue that the courts ought not to give credence to the forum selection
clause contained within the contract. Contends that the representative plaints read only
portion of the member agreement and thus had no notice of the forum selection clause.
In other words, the way the screen was made up that the clause for jurisdiction was
fine print and that it wasnt significantly brought to their attention therefore they should
not be bound.
This argument is using old contract law about fine print
Yes- Contract, Yes- all terms apply For Microsoft Washington jusrisdiction!, whether or
not the offeree read them all.
Its not fine print, its easy enough to flip through it. Scroll mechanism used to read
through the Member Agreement on computer is equivalent to flipping pages
Plaintiffs bound by the jurisdiction aspect of the contract,

29

The exclusive jurisdiction to Washington Law is permanently stayed.

Old contract rules do not apply to ECommerce


Judge looks at fine print argument (also known as draconian clauses), everything online,
no voice or written correspondence with Microsoft when accepting contract boys say
that after seeing costs they stopped reading like would with any contract full of fine
print BUT contracts online, you need to read the WHOLE contract
Judge says that we cannot apply old contract laws of written documents to E-Commerce,
and that the plantiffs are bound.
The forum selection Claus is dispositive and there is nothing in the factual record which
persuaded me that he should use his discretion as to permit the plaintiffs to avoid the
effect of the contractual provision (granting relieve sought by defendant)
Burden for showing a strong cause to change exclusive jurisdiction rests with plaintiff;
not sufficiently demonstrated in this case.
The court disagreed with arguments by plaintiffs because they had ability to read terms
and hit I agreeall terms were displayed in same format, were not in fine print, despite
the fact they couldnt appear on the screen at the same time (had to scroll down)
In cross-examination, Plaintiff admitted he scanned through the agreement (so didnt
read all of it), also admitted that he saw the screen which indicated that a failure to read
the terms still bound the plaintiff to the terms of the agreement

Reasons

Ratio/ Rule

- Normal rules of contract formation that are applicable to agreements in


writing are also applicable to agreements created in electronic form
- Acceptance may be communicated by simply clicking on the I agree
icon on a computer screen, and such acceptance is held to apply to all
the terms set out in the agreement including those not read by the
offeree
- Courts will support e-commerce where it is reasonable to do so.
And electronic contracts are treated slightly different than standard form
contracts
An online agreement must be afforded the sanctity that is given to any
agreement in writing
A foreign selection clause is valid and binding as long as both parties agree to
it.

4. Termination of Offer
NOTE FOR EXAM:
General rules 5 ways an offer can be terminated:
1. Revocation of an offer at any time before it is accepted MAIN ONE
a. Minor Exception: the revocation must be communicated,
b. unilateral contract there is no revocation where one of the parties has decised to substantially
perform.
2. If you refuse an offer that puts an end to it
3. Lapse of Time: if an offer is stated to be open for a fixed time then it cannot be accepted after that time
4. Non-Occurrence of a Condition suggests there is revocation.
5. Death: the offeree cannot accepte the offer after ther person who makes the offer dies and the offeree knows
about it
o Exception: if the offeror dies and you dont know and you accept than in some cases the
representatives of the dead person may be able to conclude the contract

a) Revocation
Dickinson v. Dodds (1876) CA
Facts

Wed June 10 - Defendants offer to sell to the Plaintiffs (the offer was open until Friday 9am)
June 11 - Plaintiffs accepts the offer
30

Issues
Held
Reaso
ns

Mother in Law forgot to pass on message from the Agent that the land had already been sold.
A third party (real estate agent) told the Plaintiffs Mother in Law that the land had been already
sold
Plaintiff tries to accept on Friday 7am at the train station
But the property was already sold.
Plaintiff brings an action for specific performance
whether or not the defenent was in breach of contract by selling property to third party before
expiration of the offer?
Plaintiff had knowledge of revocation, therefore there was no contract.- mere promise to hold the
offer for a period of time was not binding on the defendant- free to wtihdraw
Mr. Dodds the Defendant had communicated revocation through a third party (real estate agent)
There was no contract. Although offer was open until Friday, that did not bind Dodds.
Offeror may accept the highest bid despite giving the initial offeree a specific deadline.
The deadline was not a promise to the Plaintiff it was a general deadline
Dodds did not have to keep promise of deadline (it was not a binding promise) because he does
not say it was a revocable deadline and it is just an offer not a contract (there is no consideration
for the promise)
BUT If there were no other offers, Dodds would be bound.
Because he hears it was sold to someone else, There was no meeting of the minds.
BUT If Dickinson (plaintiff) had received no information of revocation from the agent he would have
won the case.

Ratio

Notes

-- an offer can be withdrawn before it is accepted even if the offerer states that it will
be open for a period of time- must be communicated or otherwise come to knowledge
of offeree
-- Revocation can be communicated indirectly, through a third party. (If the other party
has knowledge of the revocation, that is sufficient.)
BUT what if the third party was not reliable, this concept is vague in this case and that is a
problem. The case does not clarify who constitutes a third party, what if this third party was a guy
from the bar.
Conforms to the rule that revocation can take place at any time before an offer is accepted. Pp 97

Byrne v. Van Tienhoven (1880) CA


Facts

Issue
s
Reas
ons

Oct. 1sellers (defendants) make an offer to buyers by Mail


Oct. 8defendants mail a revocation of offer
Oct 11Plaintiffs accept the offer telegram and OCT 15 by letter
Oct 20Plaintiffs receive letter of revocation
Plaintiffs say that there was a contract and that this was a breach.
They argue that the contract was concluded on Oct. 11 and that they had no revocation of it, they
say the minute they put their acceptance in the mail they had a binding contract
The defendants argued that they had the freedom to revoke any time prior to acceptance
Argue that the revocation was communicated as soon as they posted it.
Defendants also argued that there was no meeting of the minds because they had already sold
the land to someone else prior to getting acceptance.
If revocation is sent by mail, when does it take effect?
Postal rule does not apply to revocation and only applied to acceptance
Therefore the defendants could not argue that revocation was communicated
The Plaintiffs acceptance was communicated before the revocation because of the double standard
of the postal rule.
Court held that meeting of the minds irrelevant because of the postal rule not applying to
31

revocation.
Ratio

Postal Rule applies to acceptance but does not apply to revocation - must be
communicated. Revocation must be communicated before acceptance.
No communication of revocation is equal to no revocation
-- it is a principle of law that an offer can be withdrawn before it is accepted, and it is
immaterial whether or not the offer is expressed to be open for acceptance for a given
time frame or not
- The only thing that falls under the postal rule is a letter of acceptance

Com
ment
s

(1880) (C.A.), p. 99
an offer can be terminated any time before it is accepted
termination of an offer must be communicated
person who has accepted an offer not knowing it has been revoked has a valid acceptance and can
act on the footing that the offer and acceptance constitute a binding contract

Errington v. Errington and Woods (1952) K.B CA (not Canadian)


Facts

Issue
s
Held

Reas
ons

Plantiff (Fathers Estate) put down payment on house and promised the son that if they paid the
mortgage then he would transfer the title to them once mortgage was done. (1930)
Action for Widow to have the Daughter-in-Law Evicted (an eviction action) in 1945
Couple had been paying the mortgage for over ten years faithfully
Father dies before he transfers the title to the son and daughter in law
So then the title is sent to the Fathers estate and thus the Widow
The Son runs off and leaves the Daughter-in-law in the house
The Widow wants the Daughter out and technically has title of the property
Estate argues there was no contract. Was there proper acceptance to create a contract?
if the daughter in law pays all the building company installments, will the couple have the
property transferred to them as soon as the mortgage is paid off?
Yes.
Fathers contract was unilateral offer to the daughter in law and son, and could not be revoked once
performance began. As long as the daughter in law continued the payments
Denning
To help the daughter out of this dilemma Lord Denning says that there was a unilateral contract that
gives the Daughter-in-law title
Father made the offer, the daughter-in-law paying the mortgage was acceptance, once the
mortgage is paid then the contract is binding
This is a unilateral contract. (Not bilateral? In bilateral, there is an exchange of promises they did
not promise to make the payments.)
As it is unilateral, the promiser cannot revoke the unilateral offer if the other party has started
performing
Payment of the mortgage was performance i.e. acceptance
--unilateral offer could not be revoked once they started making the paymentsif they failed to
make installments, then it would be an unfulfilled condition

Ratio

Note
s

- A unilateral offer can be made to a single person, it doesnt have to be made to the
world at large.
- the promissor cannot revoke a unilateral contract if the offeree has started
performance of the act that is required by the offer
Note: persuasive, not adopted universally in Canada
A Little Background For Understanding this Case:
Contract law is very Eurocentric and reflects the practices of Europe.
In this culture the notion of property ownership was very strong
There is commenced performance, sustained performance, and substantial performance

pp 102
32

b) Lapse
Barrick v. Clark (1951) SCC
Proced
ural
History

Facts

When an offer is lapse in time


Trial Judge found for Barrick
Appeal Judge found for Clark
This court finds for Barrick
Clarke sought decree of specific performance (contract formed between himself and Barrick)
action dismissed at trial, but reversed on appeal (correspondence does not set any date before
which the offer must be accepted and in judges opinion the offer did not elapse by effluxion of
time. Defendant did not withdraw his offer, must in law be considered as making it until the
plaintiff received it. It was then accepted)
Oct 30 Clarke offered to purchase land for $14,500 with possession date anytime between
January 1st and March 1st 1948, asking Barrick to reply be telegram
Nov 15 Barrack did not reply by telegram instead writing by letter to Clarke
Letter said: prepared to sell to you for $15,000 if satisfactory deal can be closed immediately by
preparing an agreement to be given to you on receipt of initial payment of $2,000 with transfer
of clear title on Jan 1st 1948 on receipt of balance of payment ($13,000). Trusting to hear from
you ASAP
Letter delivered on November 20th when Clarke absent and on a hunting trip
Mrs. Clarke received letter and wrote back to Barrick informing him that Mr. Clarke was on
hunting trip and would be back in 10 days, and she would try get hold of him
Mrs. Clark requested that Mr. Barrick hold the deal open until you hear from him
Clarke returned on December 10th (MORE THAN 10 days) and wrote to Barrack: accepting the
$15,000 Barrack requested and enclosed the $2,000 deposit and agreeing to pay the balance
by Jan 1st upon clear property transfer; kindly acknowledge receipt of this letter by return mail
In meantime William Hohmann without knowledge of Clarkes response, on November 28
inquired about Barricks land and on November 30th offered him the land for $15,000 cash
Barrick accepted this offer on December 3rd
The day after writing his letter (on December 10th) Clarke heard a rumour regarding Hohmanns
purchase and wired Barrack: (got home yesterday from hunting and mailed letter including
$2,000, heard rumours that Hohmann bought desired land, presumed you received Mrs.
Clarkes letter and trust rumour is not correct)
Barrick received wire on December 12th and wrote Clarke: received your wire today and Mrs.
Clarkes letter on Nov 20th. Mrs. Clarke requested me to hold deal open until your return from
hunting trip. Held deal open until Dec 6th when I received offer from Hohmann and accepted
having no reply from you. Solicitor is preparing transfer of title to Hohmann once I receive
$15,000. If he fails to do so I shall be at liberty to sell to someone else. Sorry hitch occurred and
will return your $2,000

C offered to purchase land from B


- B made counter immediate closing-offer but C out of town hunting and wife
sends letter saying C will be back in 10 days
- C was gone for longer than 10 days and B sold the land to a third party
C returned and tried to accept

Holding

given that there was no date for acceptance, what is a reasonable time for the offer to stay
open? Did the counter offer lapse in time?
Defence (Berrick) is arguing that too much time had elapsed so the offer had expired so re
made offer to someone else
Plaintiff (Clark) argues that it was not a reasonable time because there was no rush, the
negotiations had history,
Finds for Mr. Barrick, a reasonable time had lapsed and the offer was expired

Reason
s

No time was specified so then you look at the circumstances to figure out what a reasonable
time for Mr. Barrick to have kept the counter offer open he said reply as soon as possible.

Issues
Argume
nts

33

Court says that the letter itself has a sense of urgency since he uses the word immediately
Also because there was a third party interest then perhaps there is urgency to sell
Main reason for saying that this was a reasonable time was because no time to finish the sale
before Jan 1st
Ratio/
Rule

Notes

If offerer doesnt stipulate time for acceptance, it must be within a


reasonable time.
- Reasonable Time (determining whether there is a lapse) depends on;
(1) the nature and the character of the subject matter,
(2) normal business practice,
(3) circumstances of offer, including conduct of parties and the course of the
negotiations.
- What this really means is that there is lots of room to argue your particular case
Case is important for knowing what constitutes a reasonable amount of time
This is a case by case test

Revocation

An offer can be revoked any time before acceptance


If a revocation is sent by mail, it only becomes effective when it has been received opposite of the
postal rule (Byrne)
Revocation can be communicated through a reliable third party (Dickinson)
If there is a partial or substantial completion of the act in a unilateral contract, then the offer cannot be
revoked (HOWEVER, offeree is not obliged to complete the performance of the act and once they stop the
offer can be revoked (Errignton)
Nudum Pactum a bare statement e.g. we will keep the offer open to you for a day longer (no
consideration) (Dickinson)

Lapse

Did a reasonable amount of time pass if it has then it can be inferred that the offeree has rejected the
offer (look at the commodity itself, circumstances, nature, character and usual course of business in
industry)
In the offer, is there an implied term that it will expire after a reasonable time
Did the offeree state that they were still thinking about the offer, if they did then the offer remains open
but the offeror still has the opportunity to revoke it

Chapter 3: FORMATION OF THE AGREEMENT:


CERTAINTY OF TERMS
Certainty of Terms: If terms are too vague or incomplete, it may be that you dont have a contract at all.
Uncertainty is always present though the notion of uncertainty has to be that it is so uncertain that the
contract itself is uncertain.
- both parties must share an unambiguous understanding of their rights and obligations consensus ad
idem
- language used is often whether a contract is void however, the real issue is whether a contract has
come into existence at all. Contract formation, not enforcement- difference between enforceable and
failing to enter at all: when fail to enter into agreement at ALL- the damages are non existent- no contract
at all- in unenforceable agreement there are damages.
- important factor is the intention of the parties =
(1) If there is a degree of uncertainty in the terms of an agreement, the court will not attempt to resolve
that uncertainty in aid of the conclusion that the agreement is a contract unless it is clear that the parties
intended to contract;
(2) Determining whether the terms of an agreement define the parties obligations with sufficient
certainty depends upon ascertaining the parties intention as to the meaning of the language used.

34

1. Vagueness
R. v. Cae industries ltd. (1986) CA
Vagueness (court of appeal) leave to appeal to S.C. refused
Facts

Government sought out the respondents to take over the running of their Air Canada
maintenance base
Government wanted to maintain a valuable air space and keep employment opportunities in
the area for the city of Winnipeg
Negotiations took place b/t the government of Canada (appellant) and CAE Industries (the
respondent) about the possibility of the respondent taking over and running an aircraft
maintenance base.
March 26 1969- letter was written to respondent giving certain assurances in connection
with the proposed purchase of the maintenance base (letter written in response to request for
assurances from the respondent)
Clause B of the article was a guarantee of a certain level of workload government will
employ its best efforts to secure the employment hours
After receiving letter the respondent then arranged for the purchase of the base by a
subsidiary company
1971 the workload at the maintenance base diminished and the respondent sued for breach of
contract

Issues

Argume
nts

Holding

Air Canada wants to close down base


Govt cant maintain man hours
R assured C that if he bought the base R would guarantee minimum man
hours and its best efforts to get more
D bought base on these promises but workload decreased so he sued

Did the government intend to create this contracts


Was a binding contract intended
Is the contract vague and uncertain or incomplete
Appellant contends that in the circumstances the letter of March 26 was never intended to
become a binding legal contract
Appellant contends that the language used by the parties is so vague and uncertain as to
render the contract unenforceable
Appellant says they need to protect the public interest saying that if they committed
themselves to 40 000 man hours without taking into consideration the rest of the country and
their needs
Respondents argue that there was a breech of contract, and that the government intended it
to be a contract because they have been fulfilling it on an ongoing basis
Yes a binding contract intended
No contract not vague, uncertain, or incomplete
Contract enforced

Reason
s

Was there an intended binding contract?


Court dismissed appellants argument and held that the circumstances in which the letter was
written did not disclose an intention to enter into a purely political arrangement rather than a
contract
The evidence shows that the government wanted to sell the Air Canada base, they are the
ones that contacted the respondent thus it is hard to conclude that there was no intention to
contract
35

Court held that it was clear from the evidence that the parties treated the document as a
binding contract to the extent that is was partly performed
Is the contract vague and uncertain or incomplete
Court held that the conclusion that the parties intended to enter into a binding contract does
not mean that they succeeding in doing so
Court held that the document was not too vague to render it unenforceable

Ratio

Looked to Marquest Industries Ltd v. Willow Poultry where it was states: difficulties in
interpretation do not make a clause bad as not being capable of interpretation, so long as a
definite meaning can properly be extracted
Court held that the agreement does not leave anything unsettled that was necessary to be
settled between the parties. It in itself an entire contract capable of standing on its own feet
Contract can still be enforced despite a certain looseness of language used throughout the
agreement (i.e. assurances can guarantee set-aside and best efforts)
Court said that as they understand the words the government has a binding contract to
provide 40 thousand hours of man hours
Court says language not so uncertain that a contract cannot be formed and enforced and
nothing against the public interest
- **- Government pronouncements may be legally binding rather than mere
statement of intention ******
- To determine the intention of the parties, apply an objective test/reasonable
person test- would reasonable person think it was intended to make contract
- contract will not be held void for uncertainty if terms can be meaningfully
construed.
-Public interest must be kept in mind in determining if a contract exists
from class:

-government pronouncement may be binding rather than a mere statement


of intention

- the court will use an objective test, the reasonable person test, to
determine gov intention

- contracts will not be held void for uncertainty if terms can be


meaningfully construed

- public interest must be kept in mind in determining if a government


contract exists
Notes

This case created a problem, because now ministers have to check their letters to ensure there
is no contract put in place, however this case was still decided correctly
THIS CASE HAS BEEN RELIED ON BY OTHER CASES
Intention to contract is t be gathered from the surrounding circumstances, the government took
the initiative to find the purchaser and therefore this is not merely a political initive but an
intention to contract.
- most cases in which a person sues the government for non-compliance of a
proclamation or statement, usually do not succeed.

Note
1. The symbiotic relationship between the question of intention to contract and that of certainty of terms is illustrated by
the decision in V.K. Mason Construction ltd v. Bank of Nova Scotia. In this case a construction company contracted with a
property developer to build a retail complex on the strength of what is often called a comfort letter provided by the
property developers bank. The letter advised that we have accorded the developer interim financing sufficient to cover the
cost of the complies? The construction company competed the project but the bank did not advance sufficient funds to over
the developers costs with the result that the construction company was not fully paid. The court acknowledge that in
principle a unilateral contract could have arisen if the letter amounted to a promise on the part of the bank that if the
construction company agreed to build the compel the bank would ensure threat sufficient funding was available to cover
the developers costs. However, it concluded that based on the imprecise wording of the letter, no contract arose. The
courts decision on the point was fulfilled rather because the language used did not define the nature of and extent of the
banks obligations with sufficient precision, it could not support the conclusion hat the bank intended the letter to have
contractual effect. Though the bank was therefore not liable for breach of contract.

36

2. Incomplete terms
- whether an agreement that incomplete on its face, may constitute a contract.
- Parties leave an aspect of their agreement unspecified in these cases, is it possible to
determine the parties
obligations? Remember, a contract to contract is not enforceable.

May & Butcher Ltd. v. R. (1934) UK last years mid term


Facts

incomplete terms/ vagueness


The suppliants entered into an arrangement with the disposals board for the purchase of
surplus tentage which remained in government hands following the end of the first world
war
June 1921 correspondence from the controller of the disposals board outlined the terms of the
arrangements (set out agree to sell, agree to price, agree to delivery, agree to dispute
resolution)
7 Jan 1922- second letter Disposals Controller referring to verbal negotiation and in letter
confirmed the sale of the tentage available for disposal up to March 31 1923
They sold some of the tentage but then there was a switch over of administration and
eventually the government stops selling/ providing the tentage

- M agreed to buy surplus tentage, price could not be agreed upon


- New management wanted to change inspection process and price but M
disagreed M wants to go to arbitration
- R renounced K
Issues

Argume
nts

Holding

Whether or not the terms of the contract were sufficiently defined to constitute a legal binding
contract
Was the agreement to agree on a price a contract
Crown argues that the price was never agreed
Suppliants say even if the price was not agreed, the arbitration clause in the contract was
intended to cover this very question of price and that consequently the reasonableness of
piece was referred to arbitration under the contract
Contract not binding. No K, merely an agreement to agree. failed to agree on essential factor
of price
it is a recognized principle of law that two parties enter into an agreement where essential
terms of that contract are undefined then they entered into no agreement at all .

Reason
s

Court held that the first arrangement made between the Disposals Board and the appellants
was in April 1920
Transactions between the parties appear to have taken a similar form, there was an
agreement for the sale of the goods; there was an agreement that the price for the goods
should be subsequently fixed between the parties; and there were provisions with regard to
arbitration in the event of dispute
Court held that the whole matter depended upon the construction of the actual words of the
bargain itself
What resulted was this: it was impossible to agree to the prices and unless the appellants are
in a position to establish either that this failure to agree resulted out of a definite agreement to
buy at a reasonable price or that the price had become subject to arbitration it is plain on the
first tow points that this appeal must fail
Under Sale of Goods Act, section 8. says if parties cannot agree on a price it will be a
reasonable price, but the court holds that this only applies if the contract is silent about how to
get a price, but here it was not silent, the contract had a mechanism of how to come to an
agreement to come to a price, but it failed.
Court says there is no way at getting at a price with this contract
37

Court held there never was a concluded contract between the parties because there was no
certainty of terms
Looking at the arbitration clause which refers disputes with reference to or arising out of this
agreement to arbitration but until the price has been fixed, the agreement is not there
The arbitration clause relates to the settlement of whatever ma happen when the agreement
has been competed and the parties are regularly bound (no contract, thus arbitration clause of
no effect)
Ratio

- an agreement btwn parties to agree in future on essential terms like


price is no contract at all.
- an agreement to agree is not enforceable
- They negotiated price, but could not agree, therefore Sale of Good
Act (if no price, go to arbitration) is not relevant only relevant when
a K has been formed.
Where a contract does not have a price fixed or a means to obtain a price the contract is not
binding it is void for vagueness, price is an essential term of a contract

From class:
--a reference to a future agreement may not be necessarily construed as an intention
not to be bound
--an agreement between parties to agree in the future on a essential term such as
price is no contract at all

Incomplete section because no price


Vagueness section because no price makes it vague
Notes

A lot of people think this case was wrongfully decided


The courts neglect the fact that the parties had started performance on this agreement
Also the Disposal Board put out the terms so if the Government wanted to back out they could
have.

Hillas & Co v. Arcos Ltd (1932) UK HL


Facts

1930 there were a consortium of companies who bought lumber from the Russian distributorArcos
1930 all these companies backed out on Arcos except for Hillas and made contract to buy the
lumber
In this contract they got a special dealoption for contract for following year in 1931 for
discounted price of 5% of the list price (clause 8)
Gave them the option to enter into a contract for purchase of 100,000 standards for delivery
during 1931 at the discounted price (clause 9)
End of 1930 the consortium came back and offered to buy all the timber and before Jan 1 1931
they sold all of their timber
Thus there was no timber left for Hillas to buy the timber at their 5% discount
Hillas sues for a breach of contract
Trial Judge said that Clause 9 was a binding contract
Court of Appeal reversed the decision based on May and Butcher case saying that it was not a
contract it was an agreement to agree because no price fixed

- Plaintiff has option clause to buy timber from Arcos (Russ. Govt)
- Prior to exercise of option, Arcos sells all timber to another bidder.
- Arcos argues option clause was merely agreement to agree no terms (price,
quantity etc) were agreed on.
- Hillas argues these terms are never in place, are to be negotiated.
Issues

Was option clause a binding contract?


38

Argume
nts

Holding
Reason
s

Ratio

Notes
Court
1.
2.
3.
4.

Arcos argues that the option provision was not binding because it was not a contract, it was an
agreement to agree
Argued that the terms were to vague for the contract of 5% discount and 100,000 standards
Hillas argues that everything in this business is negotiated and that there was never a fixed
price or a fixed anything in this business
YES clause part of original K, an installment K.
- opposite to May case
Court says that when you read the contract as a whole and the context of the business
Contract had already begun to be performed because the buying of the lumber when the
consortium left was beginning performance
Both parties actions appear that they intended to contract
With respect to the price, the court held that the price was certain because it was less than 5%
the going rate, so even if you have not put out a price list, a price list will come.
With respect to the time, dates, ect are to vague the court held that the standard business
practices and in the industry are this way (negotiating every single thing) then it is not too
vague
Courts will strain to find a contract where they can if the parties look like they intended to
contract
This court looked back at the trial judge, because the trial judge hears things first hand, and
because the trial judge was an expert in this area (international business transactions)

- A court will strain to find a contract where common sense tells you that there is one

- In contracts for future performance over a period the parties may not be able to nor
may they desire many matters of details (such as delivery) but leave them to be adjusted in the
working out of the contract and this still may constitute a valid contract
(above is from class)
Courts will look to any prior part performance between the two parties to help determine
contract.
- If any missing terms, courts may infer them from any prior part performance between
parties, and customs of the trade.
- It is the duty of the courts to strain in favour of enforceability, without demanding perfection
from the terms of the contract. Dont be too clever in finding defects.
- An agreement to agree may in some cases, be an enforceable contract
Different from May and Foley above, but court of appeal in procedural history uses May
Makes up Test
Was there an intention to contract by parties?
Look at the language and context
Look at the practices of the trade
Was this a contract to contract?

Foley v. Classique Coaches ltd. (1934) KB CA


Facts

Defendants (classique coaches), the operators of a fleet of motor coaches agreed to purchase
a piece of land from the plaintiffs who operated a service station on adjacent premises
Sale was made subject to the defendants entering into a supplemental agreement to purchase
all the petrol required for their business from the plaintiffs a a price to be agreed by the
parties in writing and from time to time
Supplement agreement had an arbitration clause, clause 8: if any dispute of difference shall
arise on the subject matter or construction of this agreement the same shall be submitted to
arbitration in the usual way in accordance with the provisions of the Arbitration Act, 1889
Following the execution of the supplemental agreement on the same day as the agreement for
the sale of the land, the land was conveyed to the defendants
For 3 years the defendants obtained all their petrol from the plaintiffs until they thought they
could purchase their supplies on the better terms elsewhere.
Defendants then attempted through their solicitor to repudiate the supplemental agreement
39

and the plaintiffs sought a declaration that the agreement was binding and an injunction to
present the defendants from purchasing their petrol elsewhere
The plaintiffs and defendants had what was called a solus agreement which meant that as
part of their agreement for the land they Classique Coaches had to buy all their oil and gas
from this single supplier
PROCEEDRUAL HISTORY: at trial decided in favour of the plaintiffs, defendants appealed

- F sold land to C on condition that C buy all petrol from F at price to be


agreed upon in writing from time to time
- K has been going on for 3 years
Issues
Holding
Reason
s

Ratio

Notes

Was the supplemental agreement binding?


Was the agreement void because it was too uncertain i.e. time to time price clause?
YES 3 years performance equals consideration. Arbitration not too vague.
This case turns upon the effect of two decisions of the House of Lords, which do not easily fit
in with each other
In Hillas the House of Lords said that they had not laid down universal principles of
construction in May and Butcher and that each case must be decided on the construction of
the particular document while in Hillas they found that they parties believed they had a
contract.
In the present case the parties obviously believed they had a contract and they acted for three
years as if they had
Compnay wanted to cherry pick their agreements i.e void the solace agreement but
maintain the land agreement
Where you have clause that says price to be fixed from time to time, it may be
enforceable where parties have engaged in course of dealings in the past
- missing terms such as price may be implied and a reasonable price assumed
-Court imports objective standard where there isnt a price, you imply a reasonable price
From class:
where there has been a course of dealings in the past, missing terms like price may
be implied and a reasonable price assumed.
Similar to May above because course of dealings (but different outcomes because Mays course
of dealing not so serious)
Different from Hillas because there had been a course of dealings in this case (contract had
been underfoot for three years) therefore had to go to arbitration
- Court applies Hillas v. Arcos , not May v. Butcher

4. Agreement to Negotiate

Agreements to agree (good faith negotiations) where parties to an agreement have done no more
than agree to agree in the future, and thus have not accepted any obligations no contract. As
opposed to an agreement to contract on unspecified terms, or on terms to be agreed in the future.
Agreements to negotiate have found few adherents among the judiciary
1. It is impossible to determine the content of a duty to negotiate; the notion is too
uncertanint to support a contract
a. What is it that a person subject to such a duty is obliged to do or not do?
2. There is no basis upon which to determine damages for breach of such a duty
a. It cannot be presumed that the negotiations would have succeeded if undertaken.
b. No way of determining what the terms of the agreement would have been
c. No basis to assess monetary value

Empress Towers Ltd. v. Bank of Nova Scotia 1991 NSCA


Facts

GOOD FAITH NOTION


Agreement to Negotiate
Bank of Nova Scotia renting from Empress Towers

40

The landlord Empress Towers Ltd. brought a petition under s. 18 of the Commercial Tenancy Act
R.S.B.C. 1979 c. 54 against the tenant, the Bank of Nova Scotia, seeking to obtain a writ of
possession under s. 21 of the Act.
1972 first lease between the parties was made
1984 first lease expired; new lease was made with clause for renewal landlord grants to the
tenant two renewal rights of 5 years eachaccepting the rental rate at market rate for any
renewal price mutually agreed between landlord and tenant.
May 25 1989 bank exercised its option to renew the lease for further term of five years from
Sept 1 1989
June 23 1989 bank proposed a rental rate of $5,400 (market value) a month up to $3097.92
under the lease that was about to expire
No written reply was received from Empress Towers
July 26 1989 the bank wrote again to Empress Towers
July 26 1989 solicitors for Empress Towers wrote to the bank saying that his client was still
reviewing the offer
August 23 1989 Nova Scotia wrote to Empress to see if any progress had been made
August 31st on the day when the first five-year term was due to expire, Empress Towers made
its response, allow the bank to remain on a month-to-month basis if $15000 was paid before
September 15 1989 and a rent of $5,400 a month was paid thereafter
The $15000 was motivated by the fact that Empress Towers had been robbed and lost $15000
from the insurance compensation (very strange factor)

FACTS:
Renewal clause which said that the rent was to be the market price as
mutually agreed
Bank exercised option in accordance with terms
E waited until day before lease expired and then demanded much higher
amount
Issues
Argume
nts

Holding
Reason
s

whether the renewal clause was void either for uncertainty or was fundamentally the same as
an agreement to agree?
Empress Towers argue that the renewal clause was void for uncertainty because they put in a
clause saying they needed mutual agreement an agreement to agree and there was no
mutual agreement here.
Nova Scotia argues (essence of this case) that the Landlord failed to negotiate even though
there was a clause for an agreement to agree.
For Bankclause is binding contract. Landlord did not negotiate in good faith
It is well established that if all that the parties say is that they will enter into a lease at a rental
to be agreed, no enforceable lease obligation is create there may, however, be an obligation
to negotiate
In Brown v. Gould thee categories of options analysed:
o Rent is simply to be agreed
o Rent is to be established by a stated formula but no machinery is provided for
applying the formula to produce the rental rate
o The formula is set out but is defective and the machinery is provided for applying the
formula to produce the renal rate
Courts will try, wherever possible, to give the proper legal effect to any clause that the parties
understood and intended was to have legal effect
Requirements are to negotiate in good faith and not to withhold agreement unreasonable
Court said that the Landlord failed to negotiate at all.
Courts implied that there was an obligation on the Landlord to negotiate in good faith
Landlord did not negotiate in good faith.
41


renewal clause came with it an implied term that the landlord was to negotiate in
good faith with the objective of reaching an agreement of what the market rental rate would be

- also required not to unreasonably withhold a renewal agreement at the market rate

- parties had said they were willing to enter a rental rate to be agreed, then it was
likely an unforceable agreement for lack of certaintybut because they included the
requirement of mutual agreement, it implied to negotiate in good faith and not to unreasonably
withhold its consent to a renewal agreement

- market rate was something that could be ascertaineddid not fail for lack of
certainty
Ratio/
Rule

If parties state they must agree on a term (i.e. will go with market rate as agreed between us),
there is an obligation to proceed in good faith (prevents stalling on behalf of some parties).
An agreement to negotiate in the absence of some objective measures is void in law and
unworkable in practice.
In an agreement where terms are to be negotiated a court MAY imply a term that parties will
negotiate in good faith.
(1) Where rent is to be agreed cannot be enforced.
(2) Rent formula present, no machinery courts will supply machinery.
(3) Rent formula is defective, machinery present courts use machinery to cure formula
In an agreement where the terms are to be negotiated the courts may imply terms that the
parties will negotiate in good faith and that agreement may not be unreasonably withheld

-Courts will try wherever possible to give the proper legal effect to any clause that the parties
understood and intended to have legal effect

- In cases where there is an objective standard to measure a duty for


example market value, the courts may imply terms of (1) negotiating in good
faith and (2) not withholding agreements unreasonably to promote the business
efficacy test and the officious bystander principles.
- in certain cases there will be an obligation on the contractual parties to negotiate in good
faith, where a negotiation clause exists in an existing contract.
Comme
To Bargain in Good Faith does not mean the contract cannot be set aside, just means that
nts
parties must attempt to negotiate.
First time a court has found in a contract case that there is an implied term, stating that when there are is
a clause which says the parties will negotiate a term, they must negotiate in good faith.

- Key to decision- if they failed to agree than there is a right of termination- would have been
an entirely different decision if no clause there at all.
In my opinion the effect of the requirement for mutual agreement must bte that the landlord cannot be
compelled to enter into a renewal tenancy at a rent which it has not accepted as market rental - also
carries with it Applied term- that the landlord will negotiate in good faith with the tenant with the
objective of reaching an agreement on the market rental rate
- mentioned business efficiency test and officious bystander testTechnique for
determining if an unexpressed condition was implied at the time a contract was drawn. In this test,
an arbitrator or investigator tries to ascertain what would have been the reply of the contracting
parties if a nosy-bystander had then asked them, "Do you intend to include the term 'x' in the
contract?" If the parties, under the circumstances prevailing at that time, would have answered
"Yes, definitely!" then the term 'x' is assumed to be an implied term. However, a term is not
deemed to be implied simply because it seems logical or reasonable under the current
circumstances.
That agreement on a market rental will not be unreasonably withheld

Mannpar Enterprises Ltd v. Canada 1999 BCCA


Facts

Mannpar held a permit under contract with the crown, acting through the department of
Indian and northern affairs, to remove and sell sand and gravel located on an Indian reserve
The permit had an initial term of five years and included clause 7: the permittee shall have
42

the right to renew this permit for further five year period subject to satisfactory performance
and reneogotiation of the royalty create and annual surface rental under no circumstances
shall the royalty rate or surface rental be less than the rates received in the preceding term.
In early 1933, Mannpar gave written notice of its intention to renew the permit for an
additional 5 years commencing September 1933
Skyway nor the Department Band were willing to renegotiate prior to the expiration of the
original permit on August 31 1993
Ultimately Mannpar took the position that the Department was repudiating its obligations to
renew in good faith (cited Empress Towers case) and elected to accept the repudiation and
sue for damages
Mannpar argues that this contract was intended to last until the job was done, the job is
clearly not done, why should the government be able to pull out now, should be required to
renogtiate the contract.
Mannpar submitted that there was duty cast upon the Crown in this case to exercise good
faith by negotiating to see if an agreement could be reached between them.
Mannpar asserts that the Crown failed to do this and that the failure should sound in damages
with the damages to be asses by the trial court
Crown argues that they have a fiduciary duty to the skyware band which effects their decision
to renew the contract
Crown argued that there was a clause that said the work would be completed within 5 years
(a limited time)

FACTS
Renewal clause for permit subject to successful negotiation (remove gravel
from Indian Reserve)
Govt refused to negotiate b/c they had fiduciary duty to Band
M claimed repudiation of obligation
Issues

1. Having regard for the language used in the permit agreement, was the renewal clause
uncertain? (in trial, yes) yes, uncertian
2. ought there to have been an implied term requiring the defendant to negotiate for the
renewal or negotiate in good faith for the renewal? No, there was no contractit was
unforceable

Procedur
al
History

Trial judge held that there was no obligation to negotiate in good faith
On appeal two substantial issues were raised:
1. Having regard to the language used in the permit agreement, was clause 7 uncertain as
found by the learned trial judge
2. Ought there to have been implied a term requiring the defendant respondent, Her Majesty
the Queen in right of Canada, to negotiate for a renewal or negotiate in good faith for a renewal

Holding

Renewal clause uncertain


No enforceable agreement arose out of the language of the renewal clause
renewal clause was merely agreement to agree. no duty to negotiate in good faith as allegedrulled against P.
there is no contractual duty to negotiate in good faith in the absence of a
benchmark against which to measure the duty. Distinguishes this case from previous one
- In Empress Towers v. Bank of Nova Scotia the benchmark was the market rate. Also, there
the word renegotiate is different from the word negotiate. renegotiate implies starting all
over again

Reasons

Court found that the language used by the Crown was deliberately loose and broad and there
was no arbitration clause
It was clearly a contract intended to end after 5 years if it needs to
It appears that the court went out of its way to hold for the Crown
43

Court finds it too uncertain;


There was no renewal formula; no agreement to negotiate within the contract (as in Empress
Towers)
An objective measure is required or benchmark therefore the court held the facts of this case
were different from Empress Towers (but an argument could be made that the benchmark
could be the old contract)
Nothing to tell parties how to negotiate the royalty.
No duty to negotiate in good faith
Ratio/
Rule

Court will not imply essential term.


When you have a clause (i.e. option to renew), ask why option clause is valid or invalid due to
uncertainty.
To negotiate in good faith need a certain agreement to begin with.
In the absence of a certain benchmark there is no obligation to negotiate in good faith\
No contractual duty to negotiate in good faith in the absence of a benchmark against which to
measure a duty

1. each contract must be looked at in the context of its own facts and the
language used by the parties
2. the implication of a term can only be made if it is the case that both parties
would be likely to agree that such a term should be applied in order to satisfy
the officious bystander test
3. a term can be implied in a contract if it is found to be necessary by a court in
order to give business efficacy to the contract
a court will not however imply a term into a contract merely because the
court may think that such term would be reasonable or would likely be more
satisfactory
4. a duty to negotiate in good faith will not be implied into an agreement if
there is no language that could provide an objective benchmark to measure
such a duty such as fair value or market value
Commen
ts

Following Empress Towers, courts seem to be reluctant to impose an obligation to negotiate in


good faith
NOTE DIFFERENCE
Empress had a bench mark so that is why there is a duty negotiate
Mannpar has no bench mark so that is why there is no duty to negotiate
Court cannot infer what good faith would be

fiduciary obligation exists in relationships where one party has special rights
(Crown and native peoplescrown has fiduciary duty to them, same with doctor and patient)

In Lease Renewal Cases (where there is an existing contract) You must look to see if there is a
benchmark. If there is no benchmark to measure against, no duty to negotiate in good faith. On an exam,
mention both cases on one hand, Empress, on the other, Mannpar
Manpar Enterprises has been cited as authority for the general principle that there is no contract duty to
negotiate in good faith. Manpar established that concept of a duty to negotiate is unworkable in the
absence of an objective benchmark or standard against which to measure the duty.

Wellington City Council v. Body Corporate 2002 Newzealand CA


Facts

The Wellington city council appeals from a judgment of Wild J. holding it liable to pay to the
second respondent, Alirae Enterprises Ltd damages of $580,209 for breach of contract
The contract which the trial judge held to have been breached was what he called a process
contract (not in Canada, in Canada might be understood as an agreement to agree)
This process contract obliged the Councils officers to negotiate in good faith with Alirae the
sale of the Councils interest (sell the title) of the premises at 20 Brandon St. Wellington
44

March 2 1999 Council wrote to Alirae council officers will negotiate in good faith sales of
councils leasehold interest to existing lessees at not less than the current market value of
those interests

Issues

Contract A - Council sends letter to defendant saying we will negotiate sale in


good faith of the property of the land, at no less than market value (a
benchmark?)
Contract B the sale of the property itself
legally enforceable K in existence? i.e. was there a duty to negotiate in good faith?

Argum
ents

Councils first challenge to Wild Js judgment rests on the proposition that the process contract
was not a contract enforceable at law
Council argued that the process contract amounted to no more than an agreement to try to
agree which the law does not recognize as an enforceable contract
Council further argued that an agreement of this kind did not present sufficient certainty to be
enforceable by law
Alirae argues that the council offered to negotiate in good faith, which offer Alirae accepted by
its conduct in entering into negotiations on that basis. Each party provided consideration to the
other by their mutual exchange of promises

Holdin
g

not enforceable K, no duty to negotiate. good faith only an agreement to agree,


thus no K.

Reaso
ns

If the substantive agreement is not sufficiently certain to be enforceable, the negotiations will
have failed to bring about an enforceable contract
Handley JA in Coal Cliff held that a promise to negotiate in good faith is illusory and therefore
cannot be binding. His primary reasons were that parties negotiating for a contract are free to
pursue their own interests
An obligation to negotiate in good faith is not the same as an obligation to negotiate
reasonably an obligation to negotiate in good faith essentially means that the parties must
honestly try to reach an agreement
Looked at Walford v. Miles, Courtney and Fairbairn Ltd v. Tolaini Brothers (Hotels) Ltd, and
Channel Home Centers, Decision of Grace Retain Corporation v. Grossman-- position summed
up as: for there to be an enforceable contract, the parties must have reached consensus on all
essential terms; or at least upon objective means of sufficient certainty by which those terms
may be determined. Those objective means may be expressly agreed or they may be implicit in
what have been expressly agreed
Good faith is essentially a subjective concept, there is thus no sufficiently certain objective
criterion by means of which the court can decide whether wither party is in breach of good faith
obligation
It is implicit in what we have just said that there will be come circumstances in which a process
contract is enforceable i.e. tender cases
Found that there is no such thing as a negotiation in good faith, because subjectively we can
never tell if someone is negotiated in good faith, because in contract law we always have our
own interests in mind
An agreement to agree is not enforceable
This case shows that the notion of negotiating in good faith is alien to contract law
Negotiate in good faith WAS NOT in the actual contract and that is why it is not enforceable
Reasoning/Ratio: in New Zealand there is no duty to negotiate in good faith in contract law
- would not be very persuasive in Canada
- An obligation to negotiate in good faith essentially means that the parties must honestly try to
reach agreement.

Ratio/
Rule

45

The enforceability of contracts to negotiate will depend on their terms


and particularly on the specificity of those terms
- for there to be an enforceable contract, the parties must have
reached consensus on all essential terms for at least upon objective
means of sufficient certainty by which those terms may be determined
Comm
ents

Process contract between the council and Alirae was a contract to negotiate in good faith with
no more definition regarding obligations and thus unenforceable.

Notes
4. One of the objections raised by Lord Denning to the enforcement of an agreement to negotiate
is the difficulty of assessing damages for breach of that duty. Damages cannot be based on loss of
the benefits associated with performance of the contract to be negotiated, since it cannot be
presumed that, even if properly pursued, negotiations would have resulted in agreement. Further,
there is no way of knowing what the terms of the agreement would have been, had agreement
been reached.
5. Agreement to negotiate- case has to do with corporate divorce. Initial agreement that Brasscan
would sell off its interests in labatts to the public and in return Labbattte would sell its interests in
Brasscan. These companies made a letter contract Labbatts refused to take some shares offered
by Brasscan. A price mutually agreed is uncertain. (SEE NOTE p 143)
Court of appeal in Ontario said this negotiating in good faith not really part of canadian contract
law e.g Edward brasshand or note 7 (p.160) Denning- we cant do this because we wont be able to
assess damages or note 8 says is possible to assess damages. Thus this is a LIVE issue in
Canadian contract law should we put this notion of forcing parties to negotiate in good faith as
part of formal contract law. DOUBLE CHECK THIS WITH PROFESSOR! Cases say there is a
duty but this says no duty very confused

5. Anticipation of Formalization

The issue of uncertainty as an impediment to contract formation is often presented in


circumstance in which one party seeks to enforce an agreement, either oral or in writing,
that is allegedly binding notwithstanding that the parties contemplated that full details of
their agreement would be embodied in a further written document
When in written form, an agreement of either kind may be referred to variously as a letter
of intent, memorandum of agreement, memorandum of understanding or heads of
agreement.
From the perspective of contract formation, the issues typically arising from such an
agreement are whether the parties intended to be bound by its terms and, if they did,
whether those terms are sufficiently certain to give rise to a contract
The party challenging the enforceability of an agreement generally does so on the ground
that it is simply an agreement to agree of an unenforceable agreement to negotiate, even
if, in the latter case, the agreement expressly contemplates negotiation in good faith
In practical terms, preliminary agreements may play an important role in the negotiation
and conclusion of a transaction. They serve the purpose of motivating the parties to invest
the time and resources required to finalize the details of what may be a complex
agreement by providing some assurance that those investments are likely to yield results.
In addition, they allow the parties to move forward towards the conclusion of their
agreement by stages, addressing a limited number of issues at a time.

Bawitko Investments Ltd. v. Kernels Popcorn Ltd 1991 ONCA


Facts

Anticipation of Formalization
Appellant is Ontario Kernels Popcorn Company
Respondent is Bawitko Investments Limited
March 23 1984 Respondents real estate broker Anthony Passander approached
appellant with a view to acquiring franchise rights for the Kernels store in Jackson
Square
April 3 1984 the appellant provided Passander with a Kernels information package
46

and an application of franchise


April 18 1984 Passander met with Sadowski (represents appellant) the meeting
ended with Sadowski shaking hands with Passander and saying youve got a deal
The trial judge held that at that point there was a contractthe defendant granted
the franchise

FACTS
B wanted to buy popcorn franchise from K
B took steps to secure loan and a deposit was put down in accordance
with oral agreement but no formal agreement had been signed
Some essential terms were missing
Arguments

Appellant argues that even if there was an oral contract it was unenforceable by s. 4 of
the statute of frauds RSO 1990.
Appellant said they needed formal written document and that the essential parts of the
contract were not agreed upon

Issue

Was the oral contract found by the trial judge, a complete and binding
contract or was its enforceability subject to the parties subsequent
agreement on all of the terms and conditions to be contained in the
contemplated written franchise agreement?

Can the oral contract in itself constitute, as trial judge held, a complete and
legally enforceable contract, or is the oral contract not in itself a complete and
legally enforceable contract but was subject to and dependent upon a formal
written franchise document being settled, approved and executed by the
parties.
Decision

No K b/c essential terms were missing/still open for negotiation! Appeal


allowed.

Reasoning

Examine the conduct of the parties after April 18th


Saw that Passander made payments by way of deposit towards the initial franchise fee
and construction costs pursuant to an extended payment schedule agreed to after
April 18th
Court held that there had been sufficient part performance to avoid the application of
s. 4of the statue of Frauds
Oral contract was incomplete because essential provision intended to govern the
contractual relationship had not been settled or agreed upon
The contract was too general or uncertain to be valid in itself and was dependent on
the making of a formal written contract
The very nature of the Franchisor-Franchisee relationship mandates that there be
express agreement on the detailed provision set up to regulate the business
relationship of the parties.
There was no meeting of the minds necessary for a completed contract
The parties conduct after the 18th supports the conclusion that no final agreement had
been reached, there was still aspects to be negotiated
The oral contract did not satisfy the standards of certainty which the law requires as
prerequisite to incurring binding and enforceable contractual relations
A preliminary contract to be enforceable must encompass all the essential terms and
conditions of the final contract

Ratio

To be enforceable, a preliminary agreement must contain all essential terms and


conditions of the formal agreement.
-when the original contract is INCOMPLETE because essential provisions intended to govern the
contractual relationship have not been settled or agreed upon (original incomplete) OR
-the contract is too general or uncertain to be valid in and of itself and is clearly reliant on formal
agreement OR
-the understanding or intention of the parties even if there was no missing terms or no uncertainty, is that
there legal obligations are to be deferred until formal contract is to be created, the original agreement
cannot be an enforceable contract
47

1. when all of the essential provisions to be incorporated in a formal


document are decided upon and there is an intention that the
agreement shall there upon become binding, the parties will have
fulfilled all of the requisites for the formation of a contract. The fact that
a formal written document to the same effect is to be subsequently
prepared and signed does not alter the binding validity of the original
contract
2. when the original contract is incomplete because essential provisions
intended to govern the contractual relationship have not been settled or
agreed upon, or the contract is too uncertain or general to be valid in
itself and is dependant on the making of a formal contract for the
understanding or intention of the parties even if there is no uncertainty
as to the terms of their agreement but their legal obligations are to be
deferred until a formal contract has been approved and executed, the
original or preliminary agreement cannot constitute a formal contract
Procedural
History
Comments

This is an appeal from a judgment declaring the appellant in breach of an oral contract
To be enforceable a preliminary contract must contain all essential terms and
conditions
IMPORTANT recognize these words in preliminary contracts/ agreements
subject to contract means there is no contract until the formal document
a letter of intent there may be a contract fairness will make the courts say it is
enforceable.

NOTES:
3. In this case, court accepted that parties had in fact agreed orally on the terms alleged by the
respondent, but concluded that those terms did not give rise to a contract. The Statute of Frauds
ordinarily requires that to be enforceable, a contract for the sale of an interest in land (including a
leasehold interest) must be evidenced by a written memorandum.
4. Parties who anticipate the completion of a further written document embodying their contract
may stipulate that the agreement outlined in an informal document is subject to contract or
wording to similar effect. Language of that kind is often viewed as an indication that the parties
did not intend to be bound until a formal document is executed.

Chapter 4: THE ENFORCEMENT OF PROMISES


Three ways in which a promise can become enforceable.
a. As a contract: promise is not a binding contract unless it forms part of an
agreement in accordance with the doctrines of offer and acceptance and the
principles of certainty and completeness. The fact that a promise is part of an
agreement is not sufficient to make it binding as a contract. Criteria: 1) the promise
must be supported by consideration, and 2) the promisor must intend to create legal
relations. A third 3) additional criterion: requirement that the agreement be
evidenced in writing applies in certain cases.
the effect of the consideration requirement is that contracts are bargains:
exchanges in which an act of promise is given by the promisee in return for in
consideration of the promise.
DEFINITION: Consideration: a valuable consideration, in the sense of the
law, may consist either in some right, interest, profit, or benefit arguing to
one party, or some forbearance, document, loss, or responsibility, given,
suffered or undertaken by the other.
The concept of consideration developed to stress a promise, rather than an act,
requested and given in exchange for a promise. The consideration given by each
party does not have to be commercially adequate it is sufficient that each party
provides something of value, no matter how small. This of course gives rise to the
48

policy question of whether the courts should monitor the fairness of contracts and if
so, by what means.
b. As a deed: common law has for centuries afforded people the power to render their
promises legally binding by making them in a particularly formal manner. A promise
that is signed, sealed and delivered is binding as a deed, which is a legal
instrument distinct from a contract but with virtually the same legal effect.
c. By way of estoppel: when the law enforces a promise as a contract or as a deed it
generally does so as a means of giving effect to the exercise by the promisor of a
power to create a voluntary legal obligation. A promise is a potentially powerful
social mover: promises often inspire their recipients to act, sometimes to their
considerable prejudice, in reliance on the expectation that the promise will be
performed. Viewed from this perspective, promises are not binding undertakings but
potentially harmful social acts. The law is alive to this perspective and will in a
limited range of cases enforce promises that are neither supported by consideration
nor given under seal, as a means of preventing harm from befalling a reliant
promise. The doctrine that is concerned with remedying this kind of promissory
mischief is equitable in origin and is known as promissory estoppel.

1. Exchange and Bargains


The Governors of Dalhousie College at Halifax v. The Estate of Arthur Boutilier,
Deceased (1934) SCC
FACTS
Appellant: college (D)
against Respondent:
Estate (B)
B pledged $ to
school (not
accompanied by
letter from B as to
terms of payment)
B died without
paying
D claims pledge was
binding K
Issue: was there good
and sufficient
consideration to make
a binding contract

Reasons: Insufficient
consideration: (1) Just because D
started spending money they had
not yet received is not enough for
consideration; (2) This was a
pledge drive, he pledged, he did
not expect anything in return, (3)
Just because other subscribers
pledged does not mean there was
consideration. [If B said he would
donate in subscription of others it
would be different.]
--A bare promise by way of a
pledge or gift cannot be converted
into a binding legal obligation
because there is no consideration

RATIO a bare promise by way of a pledge


or donation cannot be converted into a
binding legal obligation because there was
no consideration

- A charitable pledge even


though formally made and
seriously intended is considered
a mere gift and even the
detrimental reliance of the party
who is receiving the gift is not
sufficient consideration

EXCEPTION: But if the university had say


given him a pin, or put a plaque with his
name on it, etc. that would have been
consideration and he may have been
bound.

Holding: No binding K
for lack of
consideration. Appeal
dismissed.
note: if you put the donators name on a building, this may constitute consideration

Brantford General Hospital Foundation v. Marquis Estate [2003] ON SCJ


FACTS
Marquis are prominent
donors to hospital
After Mr.Ms death, Mrs.M
agrees to a $1m pledge
without any consideration
Mrs. M then passes away

Issue: does the pledge form


constitute a legal and binding
contract enforceable in law or
does it fail for lack of
consideration? (naked
promise)?
Held: action dismissed---NO!

(1) for a pledge to be considered


an enforceable contract,
consideration must be shown;
(2) name recognition for the
donor is not sufficient
consideration unless it can be
proven that the donor required
49

after only making one


payment of $200,000

such name recognition as a


condition to making the pledge.

Reasons: the idea that the


unit be named after the Ms
came from hospital, not Mrs.
M. there was no mention
whatsoever of the naming
promise in the pledge
document itself. Mrs. M never
sought naming of unit as a
condition for making the
pledge.

Wood v. Lucy, Lady Duff-Gordon 1917, U.S


FACTS
W got exclusive right to sell Ls
design and what she approves to
be hot
L then sells her own name and
designs and breaks the exclusive
contract
L argues no K b/c W has no duty
to do anything, therefore no
consideration.
Defendant is a fashion designer
Employs the Plaintiff to market her designs
Plaintiff was to have exclusive rights to her
designs for one year from April 1 1915 unless
terminated with 90 days notice
They would then split profit of sales by 50%
Agreement of employment was signed by
both parties
Defendant placed her designs on fabrics,
dresses and millinery without Plaintiffs
knowledge and withheld the profits
He Sues for damages and breach of contract

Issue: Was
there consideration and if
so can the contract be
enforced when there is no
explicit promise made by
the plaintiff?
Holding: There was
implied consideration b/c
there was an exchange of
rights and money.
Reasoning: A promise
may be lacking and yet the
whole writing may be
instinct with an obligation
imperfectly expressed if
that is so there is a
contract. Defendant L gave
plaintiff W an exclusive
privilege (shows
consideration). Plaintiff had
duties.

- Implied promise of
one party can be sufficient to
constitute consideration for a
contract and to support a
cause of action against the
other party for breach of the
contract

-Courts will reach to


find something reasonable for
the purposes of salvaging an
enforceable contract

- Mutuality (of promises)


must be constructed where it is nonexistent in the contract
A contract that does not explicitly state terms
there can be an Instinct with an obligation
imperfectly expressed. (consideration can be
implied)

2. Past Consideration
-

Past consideration is no consideration at all


a promise given in recognition of or in return of for benefits received by the promisor in the
past is unenforceable
Example; I voluntarily pay for your education no agreement of pay back, ten years passes you
say I will pay you back, that is a promise made for past consideration- key is that I voluntary
paid for that, no valid agreement when he says I will pay you back, I cant enforce that because
I paid voluntarily.
Past consideration does not imply to existing promise- but you can make it enforceable if you
attach consideration to this promise
Example- say I pay for education, and he says I will pay you back, and I give him consideration
in return, i.e. I will work on your farm for the money.= enforceable- need to give a promise in
return
Promisory Estoppel enforcing a promise where there hasnt been consideration

Eastwood v. Kenyon (1840)


FACTS

ISSUE: is there a

RATIO:
50

Young woman Sarah whose


guardian E borrows money to
pay for her education
She pays back interest for one
year and she marries K and she
no longer has power to contract
b/c woman
K tells E will pay back $ but he
never does
Eastwood was guardian to Sarah;
Eastwood borrowed money and paid for
Sarahs education.
Sarah promised to pay back when old
enough.
Sarah pays interest for one year
Kenyon marries sarah (so sarah no longer a
person able to contract) and also promises
to pay back the money.
Eastwood (plantiff) sues Kenyon
(defendant) for failure to keep promise to
make payments

binding contract to
pay the amount of the
loan or does it fail for
lack of consideration?
HELD: NO!- Held for
defendant, E not
entitled to money. No
consideration
REASONS: Taking the
promise of the
defendant to have
been an express
promise, the
consideration for it
was past and executed
long before and was
not laid to have been
at the request of the
defendant. All there is
here is a benefit
voluntarily conferred
by plaintiff and
received by defendant
with an express
promise by defendant
to pay $
- what purchased
the promise of the
husband?
- The husband
hasnt done
anything new!

-Past Consideration is no
consideration at all: a
preexisting moral duty does not
confer consideration
- Theres no consideration for
voluntary gift not requested by
the person receiving it

Reasons behind this conceptcourts dont want people engaging


in voluntary actions and then
expecting something in return.

-No consideration for a voluntary


gift not requested from the person receiving
it
- benefit received in the past, which had
not been requested, is not consideration
for a future promise
- TO DISTINGUISH, LOOK AT
LAMPLEIGH WHERE REQUEST IS
MADE BY PARTY!
- Note: Lord Mansfield is [alleged to
have] considered the rule of nudum
pactum as too narrow, and to have
maintained that all promises
deliverately made ought to be held
binding. Such a doctrine would
annihilate the necessity for any
consideration at all, inasmuch as
the mere fact of giving a promise
creates a moral obligation to
perform it.
- The enforcement of such promises
by law, however, plausibly
reconciled by the desire to effect
all conscientious engagements,
might be attended with
mischievous consequences to
society; one of which would be the
frequent preference of voluntary
undertakings to claims for just
debts.
- Suits would thereby be multiplied,
and voluntary undertakings would
also be multiplied, to the prejudice
of the real creditors
- Must find some way to favour the
creditor!
only time someone can be sued for breach of a contract formed when the defendant
was a minor, is when the services provided were the necessities of life. Would
have to pay when she became of age. Policy reason for this is that the courts dont
want people denying children the necessities of life
Policy argument: shouldnt encourage guardians to go into debt on expenses that
the child may not have spent money on themselves. Burdens the child.
Plaintiff couldnt sue Sarah, because she was married and lost her status to
contract.

51

Lampleigh v. Brathwait (1615)


REQUEST ON PART OF PARTY IS THE DISTINGUISHING FACTOR---UNLIKE SARAH
CASE!
FACTS
ISSUE: is there a binding
Ratio: a request, coupled with a
D kills someone. Asks P to try
contract or does the
promise to pay, is good
to obtain pardon from King. P
promise fail for lack of
consideration = amounts to a
travels to see King at own
consideration?
binding contract, even if request
expense.
and promise occur at different
- D promises to pay money.
HELD: for P yes binding
- D now doesnt want to pay,
contract
times (subsequent 100 pounds)
argues P did nothing except
ride horse back and forth.

Request of
defendant!!
Distinguish based
on this fact

look at the contract as a whole


Request was made with an expectation
to pay
Key is the Request by the
Defendant

3. Consideration Must be of Value in the Eyes of the Law


-

Application of doctrine of consideration does not require that consideration be adequate


Also wont question whether its equivalent in value
But consideration must be sufficient- thats what it means by it must be value in the eyes of
the law
For the purpose of contract wont look at adequacy or value but if its significant
Concept of pepper corn theory- you can give a peppercorn (no value) as consideration and the
courts can hold that promise to be enforceable
A dollar can be transferred for the transaction to occur and for the promise to be enforceable
- PEPPERCORN THEORY IS THAT IF SOMETHING OF VALUE IS EXCHANGED IN
RETUNR FOR OSMETHING OF LESSER VALUE, THE CONTRACT IS STILL
ENFORCEABLE!---BUT COURTS WILL LOOK TO NOMINAL CONSIDERATION purpose is
to pick up on cases of fraud (if picking up a house for 10000 that is valued at 50000,
they are going to pierce that to see if any fraud,,,they might not consider that
sufficient consideration

Thomas v. Thomas 1842, Q.B


FACTS
In evening before his death,
he expressed orally a wish to
make some further provision
for his wife (promising his
wife a house), and on the
following morning he
declared orally, in presence
of 2 witnesses, that it was
his wish that his wife should
have either the house in
which he lived and all that it
contained, or an additional
sum of 100 pounds instead
thereof.
After his death this promise
was executed in will
Will said as long as she paid
a pound a year in rent house
was hers
After death one of the

Issue: was there


consideration for the
promise? Was it sufficient
value of the court (1
pound per year)? was
there an obligation to
allow the wife to retain
possession of the house
under the contract?
Holding: Consideration
was found in the yearly
rent. Widow allowed to
live in house for life.
Reasoning: Do have to
have consideration to have
a contract.
The presence is important,
and the one pound is
sufficient

Consideration may be
something small as long as it
is of some value in the eyes
of the court; Courts do not
inquire into the adequacy of
the consideration

Consideration must generally come


from the promisee (person benefiting
from agreement)

looked at the contract that was entered into


between the executors and the wifesaid
this was not just a mere gift, it was a n
expressed agreement to pay what seems to
be a portionment of ground rent----this is
sufficient consideration (even though only a
pound)

52

executors (brother of
husband) refused to honour
the promise
She must demonstrate that
there is contractual
obligation
Peppercorn Theory consideration must be of value in the eyes of law however it can be as
meager as a dollar, or a peppercorn

5. Bona Fide Compromises of Disputed Claims


Callisher v Bischoffsheim (1870) QB (Forbearance)
FACTS
Plaintiff had alleged that certain
moneys were due and owing to him.,
to wit, from Government of
Honduras, and from Don Carlos
Gattierez and others, and had
threatened and was about to take
legal proceedings against the said
government and persons to enforce
payment of same.
In consideration that P would forbear
from taking such proceedings for an
agreed time, D promised to deliver
to P certain securities, to wit, bonds
or debentures, called Honduras
Railway Loan Bonds, for sums to the
amount of 600 euros.
Argues defendant breached because
he had not delivered to the plaintiff
the bonds, or any of them.
Arguments: D says P did not take
any proceedings during the agreed
period, or at all, and that all
conditions had been fulfilled
necessary to entitle him to sue in
respect of the matters before stated.

Issue: was there


consideration for the
promise?
Holding: judgment for the
plaintiff.
At the time of making alleged
agreement, no moneys were
due and owing to the plaintiff
from the government and
other persons.

B. (D.C) v. Arkin 1996 Man CA


FACTS: Plaintiff seeks to
Issue: Can the plaintiff
recover the funds that she paid recover money paid over on
to Zellers in damages to
the ground that Zellers never
resolve a civil suit brought on
had a valid claim against her
against her son for shop lifting
personally?
Arguments: Z submits that,
the plaintiff voluntarily paid the
compensation sought and, in
effect, entered into a valid and
enforceable contract with Z:
there was consideration
moving both ways; in
exchange for sellers

Reasoning: The plaintiff


honestly believed that the
claim was a serious one and
that if she did not pay it the
defendant would sue her.
The court could not believe
that Z seriously thought that

Just because
something is taken to
be true that destroys
an argument, it does
not vitiate the contract
and destroy the
validity of what is
alleged of the
consideration.
Ratio:

Ratio: Forbearance to sue is


good consideration and that
monies paid in exchange for
a promise not to sue is a
valid and enforceable legal
contract.
Forebearance giving up a
right for payment of a
promise. (settlement
agreements)
Qualifications to this!
53

forbearance to bring suit


against her, the plaintiff
voluntarily paid to them the
sum of $225.

this claim could succeed or


that they seriously intended to
pursue it to court if it was not
paid.

Plaintiff submits that, in the


circumstances, the law will not
countenance such a contract;
Z were never entitled to claim
or get ay money from the
plaintiff; and that the sum in
question should be returned to
her on equitable principles.

The parent is not responsible


for the actions of the child
unless they are negligent in
the case as well.
There state of knowledge is
important

1. Claims known to be
invalid. A promise is
not binding if the
sole consideration for
it is a forbearance to
enforce a claim which
is invalid and which
is either known by
the party forbearing
to be invalid or not
believed by him to be
valid.

Holding:
Zellers isnt giving up
anything if there claim is
invalid in the first place.
Appeal is allowed and the
plaintiffs claim is allowed with
interest and costs.

Forbearance:
- A forbearance of a disputed claim can be valid consideration if:
o The claim is reasonable in itself, and not vexatious or frivolous
o That the claimant has an honest belief in the chance of success and,
o That the claimant has not concealed from the other party any fact,
which to the claimants knowledge, might affect the validity of the
claim
o [claimant has a serious intent to pursue the claim]
6. Pre-Existing Legal Duty
Suppose that B makes a promise to A in return for As promise to perform some act X that A is
already legally bound to perform. Is the promise by A to do X good consideration for Bs promise?
One could argue that the answer ought to be o. from the laws point of view, a promise to do what
one is already bound to do should be indistinguishable from a promise to do what one has already
done. Having judged that A must do X, the law regards that act as non-optional: as far as the law
is concerned X is necessary. From this perspective then, A gives up precisely nothing in exchange
for Bs promise. He simply does or promises to do what from the laws exclusively prescriptive
point of view there was no possibility that he would not do.
Example (mentioned in class) (p. 179)
Ward v. Byham
- Where the father pays up to 1 pound a week allowance for the mother to make sure
that the illegitimate child will be happy and also that she is allowed to decide for
herself whether she wishes to live with the father or mother
- Father stops paying because he said she has a public duty to do that
- However, there is consideration
o That she remains happy and has a choice
o Also gains the right to enforce these obligations, whereas typically only
the crown would have that right to enforcement.
-

Statutory duty: basic rule is that this is not consideration.


Duty owed to 3rd party: there is sometimes consideration (this is exception to basic
rule).
Duty owed to promisee: basic rule is that this is not consideration, but there are many exceptions.
54

Duty to pay a debt: general rule is that payment of a lesser sum cannot be satisfaction for whole several exceptions.
PUBLIC DUTY
- the principles that stem from this are pretty self-evident, make common sense
- idea that promises given by public officials to carry out their public responsibilities are
unenforceable when talking about contracts
- general rule is that if you are agreeing what you have a public responsibility to do then
there is no contract, but if it goes above and beyond then it may constitute a contract
- example cant agree to be a witness to a crime in a contract, already obliged by public
duty
- traditional view: if, in exchange for a promise, the promise agrees to perform, or actually
performs a public duty, there is NO CONSIDERATION. However, courts were able to find
consideration if it could be shown that the promise provided something extra, beyond the
requirements of the public duty.
DUTY OWED TO A THIRD PARTY
The performance of a duty owed to a third party has traditionally been viewed as good
consideration, particularly in the family context.
Promise to a third party is good consideration

Pao On v. Lau Yiu Long 1980 PC duty owed to a third party


Refer to handout on blackboard
FACTS
- there were two defendants
because the first agreement
was with the legal entity of
the corp but the supplement
agreement was with the
representative of the
corporation. i.e. a
shareholder. Therefore,
there was a 3rd party pre
existing legal duty.
- Two contracts
- The sale and the protection
of the sale
- Plaintiff realizes that the
subsequent agreement
doesnt allow them to cash
in on a rise in stock prices
- Defendant claims that they
had to agree to the revision
because they were under
economic duress.
- Defendant says that when
they made the second
agreement, the plaintiff was
already obligated to delay
the shares (past
consideration). They havent
done anything more.

ISSUE: Is there
good consideration, a duty to the
third party?

was there consideration


given the context of the
second agreement or was
it a promise to preformed a
preexisting duty in want of
consideration?
HELD: Yes BC agreement
made with company and
then with share holder.
Note: if the promise was
made to the same person
twice then it might have
not been good
consideration- key is that
there is essentially two
contracts A to B and A
There was three
arguments for
consideration!!

RATIO:

A promise to perform or the


performance of a pre-existing contractual
obligation to a third party can be valid
consideration

An act done before the giving of a


promise to make a payment or to confer some
other benefit can sometimes be consideration
for the promise BUT Requirements:
1. the earlier act must have been done
at the request of the PROMISOR and
2. there must have been some
understanding at the time of the act
that it was to be remunerated either
by a payment or the conferment of
some other type of benefit.
3. The promise itself must be
enforceable, had it been made at the
time of the act

Ratio: sets out a test as to when


such preexisting agreement will be
held (Scarman: past consideration is
normally no consideration, UNLESS):
1. an act done before the giving of a
promise to make a payment or give
a benefit can sometimes be
consideration for the promise
a) the act/promise must
have been done at the
promissors request
(defendant in this case)
b) parties must have
understood that the
act/promise was to be
remunerated either by the
payment or conferment of
some other benefit
c) payment or the
giving of a benefit must
have legally been
55

enforceable had it been


promised in advance

HOWEVER:

DUTY OWED TO THE PROMISOR


Promises to Pay or Provide More
- Promisor may agree to provide an additional consideration under agreement in return for the
promisee undertaking merely to perform preexisting contractual duties
- Unlike cases involving pre-existing legal duties to third parties it is well established(subject to
exception) that the promise to perform a pre-existing contractual duty is NOT good
consideration .
Four party (as opposed to third party)
Can the fulfillment of an obligation between two parties form the consideration between a
subsequent agreement between the same two parties.

Stilk v. Merrick (1809) Eng


FACTS
- mutiny on ship,
captain promises
crew money if
they get ship
home. They get
home, he says he
wont pay them
because there is
no consideration
for the promise.
- Captain
maintained that it
would be contrary
to public policy to
allow

ISSUE: Whether or not there


was consideration for the promise to pay
higher wages?

HELD: No consideration for


the promise to pay higher wages
No obligation to pay crew additional
money
REASONS
The desertion of a part of the crew is to
be considered an emergency of the
voyage as much as their death, and
those who remain are bound by the
terms of their original contract to exert
themselves to the utmost to bring the
ship in safety to her destined port.
Dont want to give the seamen the
exploitative power (policy argument)

RATIO:

The performance of a preexisting contractual duty which is


already owed to the promisor is no
consideration for another contract

Note: FOUNDATIONAL
CASE!!!! REMEMBER THIS FOR
EXAM!!! For pre-existing legal duty!

- Crew hasnt provided any new


consideration.

56

Gilbert Steel Ltd. v. University Const. Ltd. 1976, C.A. Ont


FACTSU contracts G to deliver
steel for 3 different
projects at agreed upon
price
- Manufacturers raise
price of steel
- G sends U new contract,
with price increase and
new clauses it was
never signed.
- Steel delivered, U only
pays old price.
- G alleges U orally
agreed to new price
(increase)
-Def astutely continued
to pay the invoices at the
old rate
-Plaintiff wants the
difference for the new
rate.

ISSUE:
was the agreement to pay the
higher prices legally binding upon
the defendant, or did the agreement
fail for lack of consideration?
HELD: there was no consideration
for the oral contract.
ARGUMENTS:
Defendant (U): past consideration is
no consideration and plaintiff was
already obliged before the alleged
oral agreement was entered into to
deliver the steel at original prices.
Plaintiff (G): the promise of a good
price on the second building was the
consideration the defendant
received for agreeing to pay the
increased price on the first. (the
court said that this is too vague
P claimed that the parties agreed to
give up the established agreement.
Court said that this would be valid if
the evidence supported it.
Third argument: the plaintiff said by
giving them 60 days to pay. And by
extending the period to pay, they
have given more credit to pay the
higher price.
Fourth Argument (not on
consideration): that the D is
estopped from denying that he
would pay the higher price. (the
court said the estoppel should be
used as a defensive argument, not
as a sword)

-Promise to perform a
preexisting contractual duty already
owed to the promisor does not
constitute consideration without
something more i..e cant amend an
existing contract without something
more than a promise from one party to
do what they were already obligated to
do in the context of the original
agreement

- Where parties have


agreed to a mutual recession of a
prior agreement and the
substitution of a new agreement
(accord and satisfaction) the new
agreement is binding even though it
may place increased burdens on
ONE party alone!

DULUX FRENCH FRIES LTD. v. McCARDLE (1976, PEI CA) opposite case to Gilbert Steel preexisting legal duty may be abandoned where the parties rescind and create a new contract, where
price went up over the term of the contract.

Williams v. Roffey Bros & Nicholls (Contractors) Ltd. 1990, C.A. England
FACTS
-Plaintiff hired as
subcontractor by
Defendant to work on
apartments
- D (Contractors)
worried project would
not be completed on
time, and they would
be liable (financial
penalty)
- D offers P (Bros) more
money to finish
apartments on time,
per apartment finished.

varying contracts in the middle of construction.


ISSUE: Was this a pre-existing legal duty RATIO: in some cases of preexisting duty,

owned by the sub-contractor therefore voiding the


additional promise of 10,300 pounds voiding the for lack
of consideration?

Did P breach contract by stopping


its work?
----was there consideration for the
defendants promise to pay an additional
10300, for completion of the flats
HELD: Was clear that the original contract
was for too low of an amount!
there was good consideration: mutual
benefit - no late penalty, dont have to find
different sub-contractor (practical benefit).

YES valid consideration, obligation

courts will strive to find consideration (if


economic duress) or where this reflects a
commercial reality and where both parties are
in equal positions.
- a promise to perform an existing
obligation can amount to good
consideration provided there are practical
benefits to the promise.

1. the court will strain


to find consideration where it is
just under the circumstances and
where it reflects commercial
reality

2. where party
undertakes to make an additional
57

- Key that the plaintiff


changed the way that
the work was to be
completed. By finishing
one flat before moving
on to the next.
- D makes one
payment, refuses to
make others so P stops
working.

on the part of the general contractor to pay


the additional contractor even though the
sub-contractor was already under a legal
obligation to perform what it was in the
original agreement!

payment on an already existing


contract, because by doing so it
will gain an advantage arising out
of the continuing relationship,
the new bargain will not fail for
want of consideration

Note: had the plaintiff distracted

3. it is no longer
the promise to pay more as a result of
necessary to look for an
economic duress, the promise would have
exchange of promises or
been unenforceable
detriments on the part of the
Modern approach; different from Gilbert promisee to enforce a variation of
Steel.
a contract so long as the
promissor obtains some benefit
or advantage

Mutual benefit- i.e. two


business partners- no economic
duress
This case is opposite to Gilbert Steel - judges find rigid approach to consideration (as applied in
Stilk v. Myrick) is neither necessary nor desireable
The content of the consideration doesnt need to be much. If we can find practical benefit.

Greater Fredericton Airport Authority Inc. v. Nav Canada [2008] NBCA


FACTS
Dispute over who should pay for new
equipment required as Nav Canada should
contractually supply equipment but it
requires a revamp of an old runway, not part
of the agreement. Airport authority pays
under protest in order to get it done, Nav
Canada acquired and installed necessary
equipment, but Airport Authority refused to
make the promised payment.
Arbitration: ASF agreement did not entitle
Nav Canada to claim reimbursement for the
DME. BUT correspondence between parties
gave rise to separate and binding contract
entitling Nav Canada to reimbursement.
Airport Authority appealed to Court of
Queens Bench, which overturned arbitrators
ruling. Nav Canada appealed to NB Court of
Appeal which held that court below erred.

Issue: was the promise made by the airport authority to pay


for the DME supported by consideration? And was it legally
binding?
Held: NO. Appeal dismissed.
Found economic duress
Ratio: a post contractual modification unsupported

by consideration may be enforceable so long as it


is established that the variation was not procured
under duress.
Does not follow Stilk v Merrick
3 Reasons why you dont need new consideration:
1. Stilk v Merrick does not apply: overinclusive: captures
renegotiations induced by coercion; underinclusive:
excludes voluntary agreements that do not offend
tenets of economic duress doctrine. Commercial
efficacy: necessary at times to adjust parties
respective contractual obligations and the law
must then protect their legitimate expectations
that the modifications or variations will be
adhered to and regarded as enforceable
2. Promissory estoppel can be used as a sword
3. Doctrine of finding new consideration has outlived its
time because with Stilk v Merrick there was no doctrine
of economic duress

58

Note ^: this case has not been followed like Gilbert Steel.

Can abandon the doctrine of consideration between contracting


parties if there is not the presence of economic duress

ACCORD AND SATISFACTION:


Sometimes, a promise to extinguish an obligation in exchange for partial performance of the same
will be enforced on the theory that the parties intended to rescind the original contract and to
substitute a new one.---traditionally this argument is available only where the obligations of both
parties are at least partially preformed (executory), as the consideration is the mutual release of
the old obligations.
If one party has fully performed (executed) the agreement, and accord and satisfaction is
normally required at common law to release the other party wholly or partially from the
obligations.

Accord and satisfaction a purchase of a release of an obligation whether arising


from a contract or tort by means of any valuable consideration not being the actual performance
itself
Accord agreement by which the obligation is discharged. The satisfaction is the
consideration which makes the agreement operative. agreement to accept some
other promised performance other than the one originally contracted for after the promisor
has fully formed his or her contracted performance.
Satisfaction the execution of the promised performance, which must be good [valid]
consideration
Pinnels rule Payment of a lesser sum on the day in satisfaction of the greater, cannot be
any satisfaction for the whole (Foakes v. Beer)
o Unless couples with a gift
o Unless accepted through a negotiable instrument (cheques)
o Unless prior to the due date
o Unless at a different location
The Mercantile Law Amendment Act has effectively abrogated the rule in Pinnel and
overturns Foakes v. Beer
o If the agreement to accept a lesser sum has not been fully executed and the debtor
has defaulted on the new agreement, then the creditor may revert back to the
original debt obligation and sue for the full amount. Where however, the debtor is
still performing the contract to accept a lesser amount, the creditor may not revert
to the original debt, but must let the debtor continue performing the new agreement
Debtor/creditor:
Payment of lesser sum on due dateno consideration for paying full
Pinnels ruletaking lesser sum on due date no considerationtherefore can ask for full
payment later on
Sometimes people do take less because it is practical.
Even in 1640 when Pinnels rule was decidedgeneral rule that payment of less sum on the
daycannot be satisfaction for whole
Pinnel modifies that rule a bityou cannot pay less for sum that is greatbut if you do
something else (give a horse) for your debtthat will be satisfaction of the wholeor if you
pay earlyor you pay at a different date (those situations may be good enough to say that
you will be fully paid)

59

7. PROMISES TO ACCEPT LESS


A) Accord and Satisfaction : a promise by a debtor to pay a creditor less than the
amount owed in exchange for a full and final release of an obligation is not a valid
obligation on part of the creditor due to lack of consideration i.e. not enforceable contract
Must be a true accord. Example of when someone sends a check, and says if you
cash it, then that is the acceptance of the accord! They can cash the check,
doesnt mean there is an accord!

Foakes v. Beer 1884, H.L

ISSUE: whether or not to treat a


respective agreement for satisfaction of
a debt for serious of payments on
account as binding on law provided that
these payments are regularly made?
WAS AGREEMENT BINDING IN LAW?
HELD: NO. F has to pay interest b/c
didnt pay whole debt by the day due
(even though there was the over time
agreement)
No. Accord and satisfaction needs
consideration! Its not an exception!
There was no consideration b/c B
was not getting anything out of it
b/c she was already owed the
money.
The lesser amount does not
constitute valid consideration for
the full amount!
Interest was implied term
There was no consideration, nor was
there accord and full satisfaction by
the partial payment of the debt and the
agreement was not under seal
- F already owed B money, therefore no
consideration.
See exceptions to Pinnels Rule above

F owed $ to
B and made
payment
plan
B agreed
not to sue
(forbear) if
F adhered
to payment
plan
F paid back
whole sum
Plan did not
mention
interest
B sued for
interest
Ms. Beer
say she is
entitled to
interest for
the five
years.

RATIO followed rule in Pennels case = you


cannot pay less than the whole sum you have
promised to pay
1. payment of a lesser sum on the due date
is not sufficient consideration
2. the doctrine of accord and satisfaction
does not create an exception to the normal
requirement of consideration
BUT an acquittance under seal, in full
satisfaction of the whole would be valid and
binding if there is a seal on the K (new
agreement) then no consideration is
required

Re SelectMove Ltd. 1995, C.A. England


FACTS
Crown brings winding up petition with
compulsory liquidation. The Company S is
delinquent in paying its income tax and
national insurance deductions to the Crown.
Company offers to pay arrears at 1,000$ a
month. Crown rep says hell have to get
approval and will respond if this is
unacceptable. Later, crown sues for arrears
but company made periodic payments.
- Crown argued any such agreement fell
within the principle of Foakes v. Beer
- Plaintiffs looking for a Williams v. Roffey type
decision

Income Tax collection by the Selectmove in national


insurance contributions on behalf of the Government
Selectmove had overdue taxes, so tax collector came
and gave proposed deal (pay in installments and 1000
arrears a month)
Tax collector said he would get back to Selectmove if

ISSUE: Whether
the agreement of
July 15, 1991 (if
it was accepted)
was supported
by
consideration?
HELD: NO. Court
found that there
was no K - Crown
did not accept
and even if there
was an
agreement the K
was
unenforceable
for lack of
consideration.
Appeal

- relies on Foakes, not Williams


(crown relies on williams)
A practical benefit must exist
for consideration to take place
when reducing a payment
Practical argument cannot be
used in cases of old debt.
RATIO:
A lesser sum on the due
date is not good
consideration for the whole
In order for the Crown to be
bound by the newer
agreement, there must be
new consideration and there
was none!
60

repayment schedule was unacceptable to the


government.
No further communication, never got back to
Selectmove
Selectmove made payments as agreed
Government then eventually demanded the payment
for all arrears in full.

dismissed.

Arguments: Government said they do not need to follow deal because they never accepted it and because

there was no consideration (Pinnells rule: there will be no consideration for part payment in replacement of the
whole debt) therefore contract terminated
Selectmove argued that the government stood to derive practical benefits b/c would get more money from
following the agreement (real consideration)

- Court unwilling to overrule Pinnels Rule says look to Parliament for that reform.

Foot v. Rawlings 1963, S.C.C


Narrowly distinguishes foaks v beer
FACTS
D owed P large sum of
money based on
promissory notes
Parties agreed on
payment plan D to
submit post-dated
cheques. In exchange,
P lowered interest rates
and dropped the
payments from $400 to
$300 monthly.
In return R would
forbear from suing on
notes
One cheque misdated
by 2 days and R sued
for full amount

ISSUE: is there

consideration for promise to


pay lesser amount? Was
there consideration for Ps
promise to forebear from
acting on the promissory

note?

If you pay a debt by submitting a


negotiable instrument (e.g. cheque)
then it is good consideration and
satisfaction for paying less on the
due date
Pinnel does NOT apply because the
lesser amount was satisfaction of the
greater amount where the satisfaction
HELD: appeal allowed
was paid through a negotiable
- judge ruled that a
instrument
misdated cheque was not
Post-dated cheques have value and
sufficient to terminate the
are good consideration
agreement.- YES- there is a
RATIO:
binding contract
-A negotiable instrument can be
P agreed to forebear while D substituted for cash and this is an
sent cheques (negotiable exception to the rule in the Foakes v.
instruments). Both are
Beer decision (exception to pinnel rule)
consideration.
- An agreement for good consideration
Creditors accepted a
suspending a right of action so long as
different form of
the debtor continues to perform the
payment which
obligations which he has undertaken
constitutes valid
thereunder is binding.
consideration.
- exception to Rawlings S.C. trying
--There was consideration because
to subtly get around Pinnels Rule
by paying in post dated cheque
sibree
v tripp cited for the proposition that
(instead of cash) so paid in a
the acceptance of a negotiable instrument
different way (like giving the horse
in pinnels) so it was good
was in law satisfaction of a debt for a
consideration.
greater amount
- If the debtor pays the lesser amount
before the amount is due, that can account
for consideration.

Exception to Foakes v. Beer exception to PINNELL RULE


This case shows us how Foakes and Beer does not take into account modern business sense

If the case had been the exact same but paid in cash then R would have won.
WE have the legislative change under Section 16 Mercantile Law Amendment Act RSO 1990.

When you bring the accord, it has to be agreed upon by the creditor.
Need an expressed acceptance!
Need evidence of the accord!
Also, look below

b) Statute ^
61

Judicature Act: RSA 2000 c J-2, s13(1)


(1) Part of performance of an obligation either before or after a breach thereof shall
be held to extinguish the obligation
a. When expressly accepted by a creditor in satisfaction or
b. When rendered pursuant to an agreement for that purpose though without
any new consideration.
Unconscionability
6(2) Notwithstanding subsection (1), an obligation is not extinguished by part
performance where a court of competent jurisdiction finds that it is unconscionable to
so allow.
Right of Revocation
6(4) A creditor may revoke an agreement under clause 6(1)(b) where:
a. the debtor has not commenced performance of the agreement
b. the debtor has commenced performance of the agreement but fails to continue
performance on a date or within a time provided for in the agreement, and it would
be unreasonable in the circumstances for the creditor to give the debtor more time
to remedy the default.

PROMISSORY ESTOPPEL AND WAIVER


PROMISSORY ESTOPPEL: is an exception to the principle that before a promise can be considered
binding it must be supported by consideration
the principle that a promise made without consideration may nonetheless be enforced to prevent injustice
if the promisor should of reasonably expected the promisee to rely on the promise and if the promisee did
actually rely on the promise to his or her detriment.
Rule: a promise is intended to be binding, intended to be acted on, is binding so far as its terms properly
apply and person is estopped from reneging on their promise- there is 6 criteria

6 Basic Criteria that must be present in order to have promissory estoppel:

1. There must have been an existing legal relationship between the parties at the time the statement on
which the estoppel is founded was made. (need this for PE- if waived then there is no legal relationship i.e.
no PE)
2. Promise: there must be a clear promise of representation made by the party against whom the estoppel is
raised, establishing an intention to be bound.
3. Reliance: There must have been reliance, by the party raising the estoppel, upon the statement or conduct
of the party against whom the estoppel is raised.
4. Notice: The party to whom the representation was made must of acted upon it to his/her detriment
5. Equity: The promisee must of acted equitably
6. Sword or Shield? : Promissory estoppel can only be argued as a shield, not a sword.

Hughes v. Metropolitan Railway Company AC HL 1877


Respondent Railway leased from
appellant landlord
Oct 22: letter form landlord asking
for the property to be repaired
by tenant within 6 months
Tenants write back proposing to
defer commencing repairs until
we hear from you on
probability that they want to sell
their lease back to landlord
Negotiations start in Nov. goes
back and forth but break down
in Dec.
Landlord never responded to

ISSUE:
Whether or not the
landlord waived its right to
insist on strict
performance of agreement
such that it could not now
insist on a strict
enforcement.

HELD: Was a
waiver on part of landlordie in support of tenant!
Was an instance of
equitable Estopple.

RATIO:
If parties who have entered into
definite and distinct terms involving
certain legal results afterwards by
there own act or with their own
consent enter a course of negotiation
which has the effect of leaving one of
the parties to suppose that the strict
rights arising under the contract will
not be enforced or will be suspended
the person who otherwise might have
enforced those rights will not be
allowed to enforce them where it
would be inequitable having regards
62

proposal to delay repairs


Part of the lease is to maintain the
property
3 days before notice to repair was
to expire: landlord served a writ
of ejectment on the tenant
Have negotiations, but dont get to
an agreement.
tenant made repairs in June
landlord brings suit to enforce the
writ
Railway company says that the
deadline should be suspended
during negotiations.

to the dealing which have thus taken


place between the parties

Estopple
denied from pursuing
your legal rights

Equity against
the forfeiture of their
leasehold right.

Central London Property Trust Ltd. v. High Trees House Ltd. K.B. 1947
Most important case in promissory Estopple
FACTS
Plaintiff C promised defendant
H lower rent during war time,
during time when flats were
only partially let.
H keeps paying lower rent after
war time
Now C now wants to raise rent
and get full payment of
arrears
H says no, that he had promise
with C
Agreement to lessen payment
Goes into receivership. The
receiver wants the difference
between the lower rate and
standard rate for the last ____
years. And wants the
standard rate going forward.
Arguments
The Respondent makes
reference to common law
estopple
Example of this, someone can
be estopped from that
representation.

ISSUE: Was there an


official agreement to
permanently lower the
rent?
HELD Court said there
was no agreement b/c
there was no
consideration. Court
exercised its EQUITABLE
OPTION and ruled that
this was a case of
promissory estoppel.
Held to that promise until
he gives notice. And then
at a point in time, can go
back to regular rates.
Notice is when war time
period ends.
RATIO: an estoppel
can extinguish preexisting obligations
and suspend rights.

REASONS:
a promise intended to be binding,
intended to be acted on and in fact
acted on, is binding so far as its terms
properly apply.
Lord Denning: once youve made the
promise that you will accept the lower
rent, it is binding so long as the terms
apply. The term that had to be met here
is : once the flats are fully rented, at
which point the lower rent ends.
Ps promise was restricted to time period
of economic recovery and thus was only
a TEMPORARY relinquishment of Ps
rights. P was entitled to full amount
once economic depression ended.
NOTE: when circumstances that give
rise to promissory estoppel end, so does
the estoppel!
The cprts have not gone so far as to
give a cause of action in damages for
the breach of such a promise, but they
have refused to allow the party making
it to act inconsistently with it.
(Shield not a sword)

Promissory Estoppel

Where one party has by his her conduct or words made to the other party a promise or
assurance which has intended to affect the legal relations between the parties, then,
once it is acted upon by the other party, the party making the promise or giving the
assurance cannot afterwards revert to the previous legal position without notice as if
no promise or assurance had been made.

Is a practical response to foakes v. Beer


63

a) The Nature of the Representation


Intention to be bound

John Burrows Ltd. v. Subsurface Surveys Ltd 1968, S.C.


FACTS
Purchase and sale agreements
secured by promissory notes.
Clause: if payments are more than
10 days late creditor can go after
the full amount. For 18 months
payments were consistently 10
days late, but nevermore than 35
days. After a disagreement, the
creditor went after the full
amount.
S = defendant
J = plaintiff

ISSUE P estopped from


claiming the debt since he
never invoked the clause
in the previous 18
months? Is promissory
estoppel available?
HELD: Court said no, S is
not estopped b/c there
was no intent to waive
rights for J under original
K. Appeal allowed. Full
amount is payable.

Indulges are not promises and


therefore promissory estoppel
does not apply. For PE there
must be a promise, either by
words or conduct, and its effect
must be clear and unambiguous.
3) estoppel can only be invoked
when the parties have entered into
a contract and afterward by their
own consent or act enter into a
course of negotiations which has
the affect of leading one of the
parties to suppose that the strict
rights arising under the contract will
not be enforced
one party cannot take advantage of
friendly indulgences and then claim
promissory estoppel
Must be an express promise to
constitute promissory estopple.

B) The Equities
D. & C. Builders Ltd. v. Rees 1966, Q.B.
FACTS
Outstanding debt for work
done
R forces D to accept lesser
amount (wife of man-man
sick in bed)
D about to go bankrupt and
accepts offer under duress
We have no choice
but to accept
R argues PE as defense

Issue: is D estopped
from claiming the
whole debt?
Holding: no true
accord there was
intimidation and
undue pressure D
is not estopped

RATIO:
- In cases of promissory estoppel, he who seeks
equity must do equity

- PE is not applicable where the


promise is extracted by intimidation
Lacked a balance of equities. where there
has been a TRUE ACCORD, under which
the creditor voluntarily agrees to accept a
lesser sum in satisfaction and the debtor
acts on that accord by paying the lesser
sum and the creditor accepts it, then it is
inequitable for the creditor afterwards to
insist on the balance. But he is NOT
BOUND unless thee has been truly an
ACCORD between them.
if faced with undue pressure, and
intimidation, there is no true accord no
person can insist on a settlement
procured by intimidation!
- a cheque is not good consideration (but
remember, Foot v. Rawlings is a S.C.C. case,
and this case is British)
DENNING: PE not applicable where the promise
has been extracted by intimidation.
64

In Ontario, Mercantile Amendment Act could


give another argument to the Defendant.
Would force the plaintiff to show duress.
Pinnels Case and Foakes v. Beer establish that partial payment is not satisfaction of the whole the
creditor is not bound by an acceptance of a partial payment. The rule of equity however, has intervened
and allowed the acceptance of the partial payment under certain conditions (see Central London Property
Trust case above). The creditor is barred from his legal rights only when it would be inequitable
for him/her to insist on them. Where there has been a true accord, under which the creditor
voluntarily agrees to accept a lesser sum in satisfaction, and the debtor acts on that accord by
paying the lesser sum and the creditor accepts it, then it is inequitable for the creditor
afterwards to insist on the balance.
WAIVER: (a kind of Promissory Estoppel) = occurs when one party to a contract or to proceedings takes
steps which amount to foregoing reliance on some known right or defect in the performance of the other
party. Waiver will be found when evidence demonstrates the party waiving had (1) a full knowledge of
rights; and (2) an unequivocal and conscious intention to abandon them. = a very stringent test, because
an overly broad interpretation of waiver would undermine the requirement of contractual consideration
Test for Waiver: waiver is found only where the evidence demonstrates that the party waiving has
1. full knowledge of their rights and;
2. has unequivocal and conscious intention to abandon them
- The test is stringent b/c no consideration moves from the party in whose favour a waiver operates
Difference Between PE and Waiver:
Generally you have a waiver when one party is in breach of the contract (usually missed deadline)
and the other party does not object
PE is when one party makes a promise and then reneged (breaks that promise) on it
Variation
A change made by mutual agreement for the benefit and convenience of both parties to an
original agreement- usually expressed explicitly either orally or by writing

C) NOTICE Reverting back to original legal rights

Saskatchewan River Bungalows Ltd. v. Maritime Life Assurance Co. 1994,


S.C.
FACTS
S made late payment
but M said they would
not cancel insurance
plan if pay immediately
S sends money but it
never reaches M so M
gives notice (waiver)
that policy will lapse,
allows time for
payment
S sends money again 3
months later then
makes subsequent
claim on policy
M claims that policy
has lapsed

Issue: Did M waive legal


rights?
- was appropriate notice
given?
- was it acted upon within
a reasonable time?
Holding: M waived rights,
but waiver elapsed. S did
not respond in reasonable
time. Appeal allowed,
judgment for M.
Reasons:
M: policy issued to S
lapsed after expiry of
grace period for payment
of 1984 premium.
S: M, through its conduct,
waived its right to compel
timely payment under
policy. None of Ms acts
sufficient to retract its
waiver of time and thus
the policy was still in force.

RATIO:

waiver will be found only


where the evidence demonstrates that
the party waiving had a) full knowledge
of the rights they were alleged to have
waived b) an unequivocal and
conscious intention to abandon

a person who waives a right


can restore that right by giving
reasonable notice: this reasonable
notice requirement has the effect of
protecting reliance by the person in
whose favour waiver operates

However, a notice
requirement should not be imposed
where reliance is not an issue
- the underlying principle is that a
person should not be allowed to go
back on a choice when it would be
unfair to the other party to do so
Waiver and promissory estoppel are
treated the same way by the Supreme
Court
65

The respondents were


not aware of Maritimes
waiver until they
received the November
letter, along with the
lapse notice and late
payment offer.
Therefore, Maritime
was not required to
give any notice of its
intention to lapse the
policy.

Waiver has been around for a while.


Note: KEY- SCC said that the company
waived its rights BUT by April 2nd the SRB
was on notice MLI wanted to reinstate their
rights, the fact that the respondents took 3
months to pay results in MLI being off the
hook- courts tend to force insurance co. pay

International Knitwear Architects Inc. v Kabob Investments


LTD.

Facts

Plaintiff tenant leased commercial premises from the defendant


landlord for 5 year term
Tenant began to experience difficulties and landlord agreed to
reduce the rent
Tenant missed Dec. 1 payment so landlord distrained for arrears of
basic rent throughout the term of the lease and demanded arrears,
interest and payment in full
Tenant sued for damages for illegal distress; landlord
counterclaimed for rent and other moneys owing under the lease for
the whole term
Is defendant tenant liable to pay rent back to landlord? Was reasonable
notice given?
Appeal allowed in part.

Issue
Decisi
on
Reaso
ns

Where notice must be given to effect a purpose, at least 2 questions arise:


1. Must the notice be for a reasonable period?
2. If so, must notice specify the period correctly?
A so called dated notice?
Reasonable notice can invoke a waiver.
With proper notice, the contractual provision, which would
otherwise have been estopped CAN gain back original right in the
original contract upon reasonable notice)

Ratio

Notes

When one party to a contract in the absence of fresh consideration agrees not to
enforce his rights an equity will be raised in favour of the other party. This equity is,
however, subject to the qualification
(a) that the other party has altered his position
(b) that the promisor can resile from his promise on giving reasonable notice,
which need not be a formal notice, giving the promise reasonable opportunity of
resuming his position
(c) the promise only becomes final and irrevocable if the promise cannot resume
his position

d) THE RELIANCE

66

deals with the issue of the degree to which you have to have reliance for the doctrine to apply
(expressed in High Trees) : where somebody makes a promise intended to be binding and acted upon
and which is in fact acted upon the promise should be considered as binding
here we are dealing with issue of reliance- extent to which the promise needs to be relied upon
whether or not the reliance must be detrimental to the person intending to revoke
Letter of credit we will extend our credit to the buyer. The letter will be sent to the seller.
The seller will need to provide documentation bill of laden (title of the goods), bill of
sale (contract between buyer and seller), and an insurance document (if the documents
are damaged while being shipped). Protects both parties. Irrevocable Letter of credit.
Will not act on the clients instructions to revoke payment.

W.J. Alan & Co. v. El Nasr Export & Import Co. 1972 QB CA
FACTS
seller stipulates payment
for coffee to be in Kenyan
shillings
buyer pays in sterling and
seller does not object and
continues more
transactions
sterling eventually
depreciates
seller sues for difference
Seller accepted the letter
of credit in Kenyon shilling

ISSUE: Did the sellers waive


their right to receiving in Kenyon
Shillings
HELD: Yes! Appeal allowed.
Note: this is not estoppel- this is
variation; this was a new offer,
but even if it was not variation it
would still succeed on waiver;
letter of credit in sterling was a
counter offer, which was
accepted, know how to spot the
issue!
What is the new consideration:
the risk that the currency could
go up or down. Could have done
the opposite.
REASONS:
Once credit is established and
accepted it is unalterable,
except with the consent of all
the parties concerned, all of
whose legal rights and liabilities
have necessarily been affected
by the establishment of the
credit.

RATIO:

You dont have to have


detrimental reliance in order for
the principle of promissory
estoppels to be invoked

Just need change of


position ^

As long as there is a
change of position, that is
sufficient.

If one party by his


conduct, leads another to
believe that the strict rights
arising under the contract will
not be insisted on, intending
the other should act on that
belief and he does act on it,
then the first party will not
afterwards be allowed to insist
on the strict legal rights when it
would be inequitable for him to
do so.
Ratio for variation side; if a letter
of credit (or other negotiable
instrument) is non-conforming and
thus introduces a variation of the
original contract, then acceptance of
such a letter is a binding acceptance
of the variation.
Warranty: His strict rights are at any
rate suspended so long as the
waiver lasts. He may on occasion be
able to revert to his strict legal
rights for the future by giving
reasonable notice in that behalf, or
otherwise making it plain by his
conduct that he will thereafter insist
on them
BUT, there are cases where no
withdrawal is possible. It may be too
late to withdraw; or it cannot be
done without injustice to the other
party. In that event he will be bound
by his waiver
67

Societe Italo-belge Pour le Commerce et LIndustrie S. A. v. Palm and


Vegetable Oils (Malaysia) SDN BHD; The Post Chaser.. 1982 QB
FACTS
P sells palm oil to D
D sells to Subbuyer
P was suppose to
make declaration
of ship sailing
P did it late
Buyers dont
protest
SB protest
B requested P to
hand over
documents
SB rejected D
D subsequently
rejected P

Notes

ISSUE: Have the defendants


waived their rights to reject the
plaintiffs tender of documents? Was
there sufficient reliance by S?

HELD: Court found that


there must be something in fact
situation that renders it inequitable for
the alleged promisor to back away from
the promise. Found for B

Court finds in favour of


defendants; nothing inequitable about
rejecting documents within two days;
there was no detrimental reliance.

Doctrine of PE did not apply

RATIO:

(1) if one party by its


conduct leads another party to
believe that the strict rights arising
of the contract will not be insisted
upon then they will not be allowed
afterwards to insist on those legal
rights as long as it would be
inequitable for them to do so;

(2) in order to establish


inequity not necessary to show
detriment, indeed the recipient of
the waiver may benefit and yet it
may be inequitable for the purpose
who made the representation to
enforce the rights
Fundamental principle: the
representor will not be allowed
to enforce his rights where it
would be inequitable having
regard to the dealings which
have thus taken place between
the parties. To establish such
inequity, it is not necessary to
show detriment, indeed the
representee may have
benefited from the
representation and yet it may
be inequitable, at least without
reasonable notice, for the
representor to enforce his legal
rights.

It is not necessary to show detriment in order for a representee to


establish the inequity required to attract the doctrine of promissory
estoppel
Ryan v Moore: Bastarache J detrimental reliance encompasses 2 distinct,
but interrelated, concepts: reliance and detriment. The former requires
that the party seeking to establish the estoppel changed his course of
conduct by acting or abstaining from acting in reliance upon the
assumption, thereby altering his legal position. If the 1 st step is met, the 2nd
requires a finding that, should the other party be allowed to abandon the
assumption, detriment will be suffered by the estoppel raiser because of
the change from his assumed position.

E) SWORD OR SHIELD?
68

Using waiver or PE as a sword v. a shield


Using PE and waiver as a defense (SHIELD)Question whether or not you can use PE or Waiver
as a sword- as a plaintiff in order to bring action forward
Traditional and current Canadian position on this issue is that PE may only be employed defensively
The best example where a party has alleged to breach a contract and try to rely on PE or waiver to say
that the other party had waived their right to strict enforcement of contract
we know that in order to have contract you need consideration- absence of consideration no agreementcan create inequities

Combe v. Combe 1951 C.A. SHOULD BE FIRST CASE

FACTS
Husband on courtroom steps
makes promise to pay 100
pounds a year. He never pays
and she sues after 7 years
In contracts the Statutes of
Limitations says she can only
claim for 6 years
The Wife can only enforce the
promise if there was consideration
for it.

ISSUE: Can Promissory


Estoppel be used as a
sword?

- she is using Promissory Estoppel


as a sword, and an original cause
of action Cant do this.

HELD:: There was no


consideration for the
promise and she is the
plaintiff and she cannot
use promissory estoppel
as a sword

Estoppel can be used to


supplement an action, but not as
an action itself
RATIO: the doctrine
of PE can only be used as
shield and not as sword
Cant sue on a promise based on
promissory estoppel.

Cant sue for spousal support


because she makes more. So she
sues on the promise? Is there
consideration? No. So she brings
up promissory estoppel.

Can you use promissory estoppel as a sword?


NO
Petrides v. Shabinsky 1982, Ont. H.C.

P supposed to give notice in


writing in exercising option
clause to renew rental
agreement by Dec
Negotiations took place after
New Year but could never agree
on rent
S suddenly decided to revert to
its strict legal rights and request
the tenant to vacate
P argues waiver problem with
arguing promissory estoppel is
that estoppel cannot be used as
a sword

Can you use promissory


estoppel as a sword? (to
commence a claim?)

Holding: for plaintiff


renew lease (5 years), go
to arbitration about rent
- Not a case of PE,
landlord waived right
to strict legal
obligations, by
negotiating past due
date, and not
terminating lease.
- Counter Case is Robichaud (below)

- estoppel cannot be used as a


sword
- can only apply Promissory
Estoppel when a legal
relationship exists
RATIO:
the doctrine of waiver in
appropriate circumstances may be
able to be used as a sword.

YES
Robichaud c.Caisse Populaire De Pokemouche Ltee 1990, N.B. C.A

FACTS
R owed $ to Royal
Bank & Cassie
Populaire

ISSUE: Can he use PE as a sword?


HELD: YES
judge quotes from Waddams, a well known
contracts expert = it seems irrational to make

RATIO:

-PE can
be used as sword if
equity permits it69

RB and CP both
agreed to accept less
in exchange for
dropping judgment
Board of Directors
refused to accept deal
R sued CP for
rengagning on their
promise

enforceability depend on the chance of whether


the promisee is plaintiff or defendantif the
reason for enforcement is the promisees reliance,
such reliance may be just as strong whether the
promisee appears as plaintiff or defendant.
Judge concludes if the principle of promissory
estoppel could be invoked successfully as grounds
of defencethen..to refuse its application on the
pretext that it is not invoked as grounds of
defence is, in my opinion, untenable and contrary
to the principles of equity on which the doctrine is
based.
Judge suggests promissory estoppel should
be able to be used as sword

( in this case equity


came from fact
that plaintiff had
relied on his
promise to his
detriment )
Note: it goes
against all other
case law
Perhaps we should
rethink this notion
of sword shield
dichotomy. (this
hasnt been
followed)

Not going to be examined on Walton Stores.

Waltons Stores (Interstate) Pty. Ltd. v. Maher (1988) H.C. of Australia


FACTS
W wants to lease Ms
land and they reach
agreement
M begins demolition of
building at Ws request
W finds out about
demolition but says
nothing then claims no
K

no legal
relationship!
- Australian Case not
binding
-

ISSUE: whether the


appellant can be
estopped from denying
the existence of a
binding contract
HELD: appellants were
estopped from denying
that an exchange had
occurred
- M relied on exchange,
to their detriment.
- Ws activity was
unconsciousable

- Notion of promissory estoppel is


flexible enough to say; The promise is
enforced in circumstances where
departure from it is unconsciousable
Even though there was no legal relationship.
- This case brings promissory estoppel away
from consideration towards reliance. - - There was no consideration here, no
agreement.
- Court applied PE even though there
was No Legal Relationship, no
Consideration.
RATIO:
-Doctrine of promissory estoppels may
extend to enforcement of voluntary
promises where it would be
unconscionable not to enforce such
promises

NO Sword

M.(n) v. A (A.T.) 2003, (BCCA) P254


FACTS
- man went to Britain, met a
women, told her to quit her job,
come to Canada, hed pay her
mortgage. She did.
Relationship ended, he kicked
her out. Did not pay mortgage.
She is arguing Waltons that
he should be estopped from
going back on his promise

ISSUE: Did the trial judge err in


refusing to enforce the promise on
which Mrs. A relied to her
detriment? Can Mrs. A use PE as a
sword?
HELD: - NO!, NO! exchange, to
their detriment.
- Ws activity was
unconsciousable

RATIO

minimum
element required to give
rise to an estoppel and
thus of an
unconscionable conduct
include and assumption
or expectation as to a
legal relationship
between the promisor
and promisee

- B.C. court says this is not an appropriate case to investigate the principle, but kind of leaves the door ajar
for a future court to investigate. Pg.256-257 talks about Waltons notion that there has to be very
INJURIOUS reliance/unconsciousable not to enforce promise.
70

Principles of Promissory Estoppel


(1) must be contractual relationship at time the promise was made
(2) there must be a clear and unambiguous promise by words or conduct, establishing the promisors
intent (Burrows Case)
(3) there must have been sufficient reliance by the party raising the estoppel upon the statement or
conduct of the promisor
(4) A party to whom the representation was made must of acted upon it to his or her detriment? (this issue
still has to be resolved)
(5) the person to whom the promise was made must act equitably.
(6) Promissory Estoppel cannot be a cause of action, but maybe a part of a cause of action (Coombe)

INTENTION TO CREATE LEGAL RELATIONS


Intention is an element of contract formation. Common law has rejected intention as the criterion for
the enforceability of
promises, in favour of consideration. The traditional view is that the presence of consideration gives
rise to a presumption
that the parties intended to be legally bound, but this presumption is rebuttable. In certain situations,
commonly those dealing
with social and domestic arrangements, the opposite presumption arises that the parties are not
presumed to intend
legal relations (in the absence of clear evidence to the contrary).
No presumption of intention to create legal relations in domestic and social contracts!

B) FAMILY ARRANGEMENTS
Balfour v. Balfour 1919, Eng.C.A.
FACTS
Couple who while they are
married have a domestic
arrangement that husband will
pay wife 30 pounds a month for
maintenance
Then husband takes off and
decides not to come back and
she sues
Husband argument: entered
into no contract with his wife:
there are agreements between
parties which do no result in
contracts within the meaning of
that term in our law.

ISSUE: was there an intent to


create legal relations?
HELD: no- not enforceable/ no
binding obligation. Policy
reason is that state should not
be involved in family
arrangements.
To my mind those
agreements, or many of
them, do not result in
contracts at all, and they
do not result in contracts
even though there may be
what as between other
parties would constitute
consideration for the
agreement.

RATIO: closely
associated parties such as
spouses would be presumed not
to create legal relations in the
absence of clear evidence to the
contrary

There is a presumption
in family relationships that the
parties do not intend to have
contractual relationships.

If we were to allow this


type of argument, then we
would have a flood of people
coming forward

Up to the other side to


rebut the assumption.

Onus on the plaintiff to


prove contract here.

Reasoning:
Not contracts because the
parties did not intend that
they should be attended by
legal consequences.
It would mean that when
the husband makes his wife
a promise to give her an
allowance of 30s. a week,
71

whatever he can afford, for


the maintenance of the
household and children,
and she promises so to
apply it, not only could she
sue him for his failure in
any week to supply the
allowance, but he could sue
her for the non
performance of the
obligation, express or
implied, which she had
undertaken upon her part.
- Note The concept of family underlying Balfour is of course fluid i.e. the Family Maintenance Act of
Manitoba says a spouse has a right to reasonable amounts for clothing etc.

c) COMMERCIAL ARRANGEMENTS
Rose and Frank Co. v. J.R. Crompton and Bros. Ltd. 1923, K.B. CA
FACTS
R and F were sole
distributors of JRs paper
products
Signed new agreement with
clause, containing an
honourable pledge claim (no
legal remedy agreement
only an expression of intent,
not legal K)
JR refused to fulfill orders
one day

ISSUE: was there


intention to create
legal relations which
are binding?---NO

Intent will always be inferred in a concluded


contract, unless there is clear language
expressing otherwise
- any honourable pledge clause will be
honoured in court
HELD: NO! J.R. wins
RATIO:
clause is upheld.

Honor bound clause

although it is generally presumed


that the intention to create commercial
relations is present in the making of
commercial arrangements, where parties
wish to be bound in honor only, and so
express themselves the absence of intent is
established
note same thing that happened in Blair - employee promised benefits upon retirement, company
reneges in the contract, it explicitly said this is not intended to be legally binding, and
therefore it is not

Toronto Dominion Bank v. Leigh Instruments Ltd. (Trustee of) Ont.CA, leave to
appeal refused (2000)
FACTS
- TD has advanced
financing to the
instruments based on a
letter of comfort
- L issues comfort letters
in connection with a
series of loans from TD,
expressing intent that
they not be liable for
subsidiary company who
took out the loan.
Two arguments
Letter of comfort
established a contract
Also a tort action by
saying it was a negligent
misrepresentation.

ISSUE: : did the parties


intend that paragraph 3 of
the letter of comfort be
legally binding?
HELD: NO!, for Leigh
On either argument: how do
we legally interpret the
comfort letter.
Based on the interpretation
that the interpretation
wasnt the way the bank
wanted it interpreted. Even
by the nature of a comfort
letter, what does the
comfort letter mean. It has
to have legal significance. It
has commercial significance

RATIO:

in general, letters of comfort


do not impose binding
contractual obligations between
the lender and the provider of
the letter of comfort, as there is
no intention to create binding
legal relations

Reasoning: The bank knew full well that the


letter of comfort was not security in the
traditional sense and that its commercial value
depended very much on the relationship which
existed between the lender and the provider of
the letter of comfort.
It was well known that Plessey would not
provide any guarantee on loans made to Leigh
by the Bank. The letter was crafted in such a
way to avoid any suggestion that Plessey had
any legal responsibility for the loans.
72

even if it doesnt have legal


significance.
There was a reason why TD
entered into the agreement.

The interpretation of para 3 now advanced by


the Bank would effectively put Plessey in the
position of a guarantor subject to Plesseys
ability, on notice to the Bank, to change its
policy.
The interpretation urged by the Bank would give
it alost exactly the security which it knew full
well was not available to it when it chose to
proceed with the loan in the hopes of doing
more business with Plessey and its subsidiaries.

FORMALITY Promises under seal


d) PROMISES UNDER SEAL
Promise can be made binding with a deed (a seal).
Royal Bank v. Kiska 1967, Ont. C.A
FACTS
Kiska guarantees his
brothers debt, then is
called on to pay by the
bank
Bank failed to put real
seal on doc.
Bank could have called
in guarantor earlier but
didnt so there was
forbearance

ISSUE: Do the words


Promises under seal are a way around
sealed replace the
consideration
need for a seal? Does the - Must have a real seal, not just the word seal
accused have to pay b/c
- - signature and stamp is not good considerationthere was consideration
need a formal seal!
HELD: yes. K enforced.
RATIO
Guarantee binding
1. Promises made in a sealed document are
because supported by
enforceable not withstanding the absence of
consideration
consideration
Because there was
2. The presence of words such as given
forbearance the court
under seal and signed sealed and
found consideration
delivered in the presence of are merely
Actually have to have it
anticipatory of a formality which must be
sealed.
observed and are not a substitute for it. The
Note: not very sig case!
operative act is the affixing or adoption of a
seal.
a way around the doctrine of consideration put a seal on document, which says it
is a legally binding document

FORMALITY: The Requirement of Writing


-s 4 and 17, section 4: certain contracts have to be evidenced in writing to be enforceable
section 17 imposes writing requirement on contracts for the sale of goods, wares and
merchandise for a price of 10 pounds or more.
s. 4 requires that the tenure of land or leases lasting longer than two years must be in
writing.
cant bring action for a promise or agreement that is not in writing
Operation of the Statute
o If an agreement for the sale or transfer of an interest in land is not in writing
o The contract is valid but unenforceable by a court. Therefore, no action shall be brought
up. However, the oral contract may sometimes be used as a defence
o Evidence to prove contract may arise
In acts of part performance
According to the equities of the situation
- Requirement of the Statute of Frauds
Document which was against the use of oral executor contracts.
Was a way to cut down the use of oral contracts.
Where there must be in writing:
(1) Anything to do with land (leases, easements, licenses)
73

(2) Guarantee or indemnity


Guarantee is where you guarantee the obligations of another when the debtor doesnt pay, the
creditor can pursue the guarantor.
There must be a memorandum in writing, which must be signed by the party in charge; form of note
or memo (any document or written note which has the required details is sufficient. The motive for
the document is irrelevant; the document may be very informal, a note, receipt or scarp of paper).
The memo should describe the parties, property, and price
Doesnt require the signature of both parties. Doesnt create the contract. Just must have the
signature of the defendant.
The memo doesnt have to be created with contemporaneously with the creation of the contract.
Cheque could constitute memo in writing. Or letters years apart.

WHENEVER THERE IS A PROPERTY ISSUE> ALWAYS MAKE SURE


THERE ISNT A STATUTE OF FRAUDS ISSUE (EXAM)
o Is there a written agreement, if not, is there part
performance, are the actions unequivocal
-

The signature: may be initial, printed, may be authorized by an agent, may be other than at
the bottom of the page
o Must have been placed with the intention of authenticating the document
- Joined Documents: In order to find a contract, may be able to bring two documents together Must be
something on basis of the two documents to conclude that they are related to the same project; if they
reference each other, if they obviously relate to one another or if you can imply the relationship; If the
connection between the two documents can only be established by parole evidence (actual oral testimony)
it cant be joined.
4. Contracts for sale of interest in land still valid!!!!!!!!! after enacting of a
statute that required contracts of sale of land to be in writing, courts of equity
decided to enforce contracts that were not in writing- but certain
circumstances had to be proved in order for court of equity to do so---doctrine of
part performance!
Where one party to the oral agreement partially performed his or her obligation,
the oral agreement may be enforced in order to avoid injustice to the party
conferring value- (so if provided services, the service provider will seek equitable
remedy of performance in a transaction in relating to land (transfer land to me) requires
other party to transfer that land because they have partially performed their obligations
under contract---doctrine of part performance)
There are three types of contract invalidities
(1) Void (ab initio or de futoro) there never was (is) a contract
(2) Voidable the plaintiff can avoid the contract but it is a good contract until brought to an end by a court
(3) Unenforceable no action can be brought upon the contract for its enforcement, but, it is valid as long
as the parties wish to proceed under it.
Part Performance:

The intervention by equity and the circumstances that must be proved to allow equity to dispense
with the writing requirement has become known as the doctrine of part performance.

A defense to this is to say that this is a dealing of real property and the contract needs to be in
writing.

Dynamic Transport Ltd v O.K Detailing Ltd 1978 SCC


Plaintiff was to buy land from
respondent dispute over boundary of
land for sale. Respondent (seller) trying
to argue that contract was void for lack
of clarity of description for actual
property--Court held, however, that the document
was vague for the description of the
land, but the court can permit oral

Issue: whether the contract lacked


clarity as related to description of
property?

Held: Court held, however, that


the document was vague for the
description of the land, but the
court can permit oral evidence to

Ratio:
RATIO: the courts will
strain to find sufficient
writing to satisfy the
statute of frauds and in
certain cases oral
evidence will be permitted
for the purposes of
74

evidence to demonstrate what the


missing details of the description were
(not adding terms--)

demonstrate what the missing


details of the description were (not
adding terms--)

explanation.
MIGHT BE WRONG
Bad case- Berryman

The provincial electronic commerce


statutes provide that a legal
writing requirement will be
satisfied electronically if the
electronic record endures and
remains accessible.
There is no difficulty in principle
with a signature in electronic form.
As long as the relevant mark was
inserted with the evident intention
of authenticating the entire
electronic writing, it will count as a
signature.

Deglman v. Guaranty Trust Co. S.C. 1954 ONE OF THE MOST IMPORTANT IN
CANADIAN LAW
FACTS
Nephew of deceased was
promised that if he did
services from time to time,
his aunt would leave him
one of the two houses
(adjoining lot and house on
it)
She dies without putting
nephew in her will
Nephews services only lasted
for a short period of time
Nephew doesnt have a
transfer of land in writing so
this case comes up against s.4
of the Statute of Frauds
Nephews acts of part
performance should trigger the
relief of equity
Nephew took her on rides and
did chores on both houses

ISSUE: what is the nature of part


performance which will enable the court to
order specific performance of a contract?
HELD: No transfer of land, but nephew
received some money from estate.
(restitution)
Restitution on the basis of quantum
meruit.
Reasoning: In relation to specific
performance, strict pleading would seem
to require a demonstrated connection
between the acts of performance and a
dealing with the land before evidence of
the terms of any agreement is admissible.
Rand v. Cartwright Where Rand required
the conduct to be referable only to the
specific contract that had been alleged,
Cartwright held that it was sufficient if the
acts were unequivocally referable in their
own nature to some dealing wit the land.

- RATIO: In
order to invoke the
doctrine of part
performance, the acts
must be unequivocal in
the sense that they
must have relation to
one agreement and to
no other.

Acts should
demonstrate that they are
directly related to the
promise

- this case
introduced doctrine of
unjust enrichment
Referred to Madison v
Alderson where that in
order to invoke the
doctrine of part
performance, the acts
must be unequivocal,
must have relation to
one agreement relied
upon and to no other.
- High level of standard

Thompson v. Guaranty Trust Co. 1974, S.C


FACTS
T lived with
deceased, who said
if you stay and work
farm, I will give it to
you
- Will is stolen (no

ISSUE: Despite the lacking


of memo in writing there have been
sufficient acts of part performance on
the part of the plaintiff to take this
case out of s.4 of statue of fraud?
HELD: YES! Court found for T who
worked on the farm.

RATIO: Acts of part


performance that are unequivocally
referable to some interest (will) or
contract in land render the contract
enforceable despite the statute of
frauds
Note: Illustration of a situation where
75

written
document)
Wrong
documentation.
Supposed to
have a will
written up, but it
ends up being a
power of
attorney

Court said T actually improved the


land built up a farm operation, rebuilt the house, put new floors, tore
down house

actions do meet the test of part


performance set out in Alderson

Distinction from Deglman


- Oral evidence from Dick, saying he
wants Gus to have everything
- A longer period of performance on
the farm.
- Both parties thought there was a
will, ended up being a POA
- Didnt go to war in order to look
after the farm.
Iris Murdoch? Facts: wife and husband lived and worked on ranch. Wife worked hard on ranch for
years. When they divorced, ranch was in husbands name. Question arises, is she entitled to half
of land because of her performance? At the end of the day, the court says no, she only did what
any ranchers wife would do. She got some money, he got ranch.

Lensen v. Lensen 1984, Sask. CA


FACTS
-Son suing his father for the
farm. Says his father promised
his son could buy the farm for
X$.
- son relied on the promise,
and passed on opportunity to
buy other land.
- Son suing for specific
performance.
- Son argues he renovated the
house, worked on farm and
improved it for over 20 years.

Issu
e: Whether the
acts of part
performance
override the
statute of
frauds?

HELD: YES! for


son

RATIO: In the case of part


performance if the acts relied upon are
unequivocally referable in their own nature to
some dealing with the land then the
requisite test of part performance is met
(nuance from previous judgement)(trying to
accommodate steadman, but not able to
overrule the SCC decision)
. Loosened Alderson test.
- Even the payment of money was a sufficient act of
Part Performance to get around the Statute of Frauds
This decision was reversed on other grounds by S.C.
which did not comment on the loosening of the test
or approval of the Steadman test which means they
probably approved of the test.- now we have a more
lenient test.

Contracts for the Sale of an Interest in Land (most important one)


VERY IMPORTANT- why we study statute of frauds
Contracts dealing with land that has to be in writing
Courts did not enforce oral contracts dealing with land
Courts of equity decided that they were going to start enforcing oral contracts dealing with land
and in doing so they developed doctrine of part performance
Certain circumstances must be proven to establish above doctrine
Where one party to an oral agreement has partially performed the contract (their undertaking) the
oral agreement may be enforced to avoid injustice
Why? Because courts of equity created this
Person who provided the services seeks the equitable remedy of specific performance claiming
they have partly performed the contract (the oral contract)
Seek it under the ORAL AGREEMENT
Specific performance: is instead of getting money to satisfy you for the contract breached you
are actually asking for the contract to be performed
Its more just to give someone what they wanted than to give them substitute for cash
Ways around this requirement of writing:
STRICT TEST Sometimes we can relax this law to have everything in writing if the person who is
getting the promise enforce can prove that they did acts that were unequivocally referable for a
contract for land, then they might be able to get that contract in force (ex. working the land or taking
care, but not washing windows) (Maddison)
76

Maddison v. Alderson (1883)- acts must suggest that you were expecting to get the land (NO
LONGER APPLICABLE IN CANADA)
REQUISITE TEST BUT, as long as there is some part performance that is not inconsistent with the
oral agreement, then the agreement would be enforced (Stedman v. Stedman)

o Lensen = the Canadian version of Stedman


NOTE: An e-mail for the sale of land meets the requirement of being in writing

CHAPTER 5: PRIVITY OF A CONTRACT


Introduction: The Doctrine of Privity applies in Canada to prevent two types of people from enforcing a
contract; (1) a person who is a complete stranger to the contract has no legal right to enforce the promise
of any party to that contract(2) the third party beneficiary, the person identified and intended by the
promisor and promisee to receive all or part of the benefit of the agreed upon performance. the second
aspect is controversial
the doctrine of Privity means that a person cannot acquire rights , or be subject to liabilities arising from a
contract to which he/she is not a part of. It does not mean that a contract between A and B cannot affect
the legal rights of C indirectly.
Issue: in the cases below, is the third party denied rights because of Doctrine of Privity or lack of
consideration?

Provender v. Wood in 1600s a third party COULD sue RATIO: party to whom
the benefit of the promise is the one who brings the action

Tweddle v. Atkinson 1861 Q.B.


FACTS
plaintiff married
- his parents and brides
parents agreed to give plaintiff
a marriage portion. They put
the agreement in writing.
- Wifes father has not paid
agreed sum. - -- Plaintiff sues
his executor.

ISSUE: could
the plaintiff (the groom,
son of the decased, father)
sue on the contract which
was made for the benefit?
HELD: NO! son cant sue:
he wasnt privy to the K.
The son have not provided
consideration, therefore,
does not have privity of
contract

RATIO: No stranger to
the consideration of a contract
can take advantage of the
contract even if it has been
made for his/her benefit

It would seem wrong


that the son could sue for the
non performance of the
contract, but Atkinson couldnt
sue the son for the nonperformance of his father

^^ Benefit and burden


should go together

Natural love and


affection is not a sufficient
consideration for a promise
upon which an action may be
maintained;
- rule originally created to protect
estates from lawsuits.
-Consideration must move from
the promisee (the person
entitled to sue)

Dunlop Pneumatic Tyre Co. Ltd. v. Selfridge & Co. Ltd. 1915, H.L

FACTS
A price maintenance
scheme

ISSUE: Can Dunlop sue Selfridge for selling


below the amount even though contract
was with Dew?

RATIO:

- No
stranger to a
77

Dunlop sells tires to Dew


consideration can
HELD: NO! Only a person party to a
take advantage for
on the promise that Dew
contract can sue on it. There appeared to
the contract even if
not sell them below list
be no privity of contract between Dunlop
it is made for their
price
and Selfridge. Because Dunlop didnt
benefit
Penalty of 5 pounds per
furnish
any
consideration
for
Selfridges

- Privity
tire in liquidated damages
promise not to sell below the list price.
is a fundamental
Dunlop sells tires to
(There is also no consideration between the principle of
Selfridge and tells them
two parties) and no agency, which would
English law
to promise not to sell
permit
it

- If an
below list price
Justice: Appeal ought to fail In the law of
Selfridge gets tires from
agency can be legit
Dew and sells them below England certain principles are fundamental. established then the
1. A person who is a party to a contract suit may
list price
can sue on it
successfully be
Dunlop sues Selfridge for
2.
If
a
person
with
whom
a
contract
not
argued by the
breach of an agreement
under seal has been made is to be
principle against the
with Dew to not sell below
able to enforce it consideration must person with whom
list price
have been given by him to the
the agent
Selfridge is the third
promisor or to some other person at
contracted with.
party
the promisors request
Dunlop says that Dew
3. A principal not named in the
was acting as their
contract may sue upon it if the
agent, while they were
promise really contracted as his
the principle
agent. But, in order to entitle him so
However, no evidence of
to sue, he must have given
this relationship
consideration either personally or
If agency, what did
through the promise, acting as his
Dunlop do, or forbear
agent in giving it.
to do, in a question
with Selfridge?
Liquidated damages are a genuine pre estimated cost of not fulfilling the contract
Puts the burden on the defendant instead of the plaintiff in proving damages
A Third Party may Acquire the Benefit through:
(When the Privity rule does not apply)
a) Statute: Insurance Legislation claiming insurance money owed to you (contract was
between the insured person and the insurance company) Insurance Act R.S.O. 1990, c. I.8
- Survivor has the right to sue the insurance company for the compensation!
Also with consumer protection legislation (Not in Ontario, they have another way of enforcing
contacts on manufacturers. This is done through multi level contracts)
b) Specific Performance:
- the common law did not specifically enforce contractual obligations except those to pay money
Specific Performance of other contractual obligations was available only through equity.

Beswick v. Beswick 1966, C.A. Eng


FACTS
-Uncles coal business
transferred to nephew
(written agreement) on
condition that after death
of uncle, nephew pays
widow a salary.
- Uncle dies, nephew stops
paying widow
- She brings action against
nephew, in the capacity of
administratrix of her

ISSUE: Can widow sue, even though


she is not privity to the K?
HELD: YES! Widow can sue in her
role as administratrix of husbands
estate, but not in her own personal
capacity.
- HLs says it is up to Parliament to
change law, allow widow to sue
on her own behalf (end doctrine
of privity)

RATIO:

-The remedy of
specific performance may be
useful for circumventing the
doctrine of privity

Where a third party


can show sufficient interest in
a contract such as being the
admin then the party can sue
on that capacity and seek
remedy of specific
78

husbands estate, and in


her own personal capacity
also sued for arrears.

Distinguishes from Dunlop: It is


different when a third person has no
legitimate interest, as when he is
seeking to enforce the maintenance
of prices to the public disadvantage.
Or, when he is seeking to rely, not
on any right given to him by the
contract, but on an exemption clause
seeking to exempt himself from his
just liability. (This no longer true in
subsequent case)

performance
- The strict law of Privity still
exists in both English and
Canadian law. However,
Dennings reasoning of
joining two parties to bring
an action (procedural
method) would probably
be tried in a Canadian
court.
- Mutuality is a ground in
favour of specific
performance.

Note Subsequent Case focuses


on specific performance and
rights as administratrix: Why
should the estate be barred
from exercising its full
contractual rights merely
because in doing so it secures
justice for the widow who, by a
mechanical defect of our law, is
unable to assert her own rights?
Such a principle would be
repugnant to justice and fulfill
no other object than that of
aiding the wrongdoer. With the
wife stepping in as
administratrix, it is no longer an
issue of the doctrine of privity.

Note: Lord Denning she can do it both ways- where the contract is made for the
benefit of third person, who has legit interest in enforcing it , it can be enforced by the third party in the
name o the contracting party or jointly with him. Lord Reed- that third party can bring action is not
consistent with common held view- Lord Denning was wrong to say that because she stands to personally
benefit she has right to sue. The only way she can sue is as an admin to the estate.

IMPT: Specific performance is an acceptable remedy of this case and is only way in
which the estate can enforce

c): Trust: Arise in circumstances where property is held by a person (trustee) to deal
with property for benefit of third person (beneficiaries). Once the trust is established the
beneficiaries is entitled to enforce the trust or obligations directly. Express words of a trust are not
required but there has to be some type of evidence that establishes an intention to create a trust.
A promise to benefit a third person is some type of evidence that a trust is in existence.

VANDERPITTE V PREFERRED ACCIDENT INSURANCE CO. 1933 PC


FACTS

- Analysis in this case falls on the


provision in the fathers car insurance policy that
extends protection to persons (his minor
daughter) who were driving his car with his
permission

- S. 24 of the INS act of BC at the


time provided that where an insured person is in
default on a judgment awarding damages you
can go after the insurance company to claim
the judgment

-The appellant is arguing that she is


an insured company under this provision and
s.24 says we can go after ins company if that

ISSUE:
driving the car with
permission held in
trust by the father for
the benefit of the
daughter?
HELD: NO! 1) she is
not party in the
contract btwn the
insurance company
and herself 2) no
evidence that the
father had any
intention to create a
beneficiary interest for

RATIO:
- A trust will only arise
to benefit a third party
beneficiary in
circumstances where it
is clear that the parties
actually intended to
create a trust
relationship

79

judgment is in default- this is what they did

his father (acting as


agent)

This was later overruled by legislation ^


Agency
McCannell v Mabee McLaren Motors LTD 1926
Manufacturer of cars and sent
these cars to diff territories where
they had dealers; dealers granted
right to purchase auto on terms
found in contract.standard form
contract and gave the retailers
permission to sell in their
respective territory.there was a
clause in contract that stated that
the dealer would not sell or trade
any cars to people who were
residing outside of
territory.wanted to create an
atmosphere where dealers were
not competing with each other
(exclusive deal in territory);
provision in contract stipulated
that it is agreed that the
paragraph shall be construed
between dealers in themselves

Issue: is there privity of contract between


dealers who each signed similar agreement
with company
Holding: yes privity between each of
dealers
- Court said that student baker as
manufactuer acted as agent on behalf of
each dealer in contracting with another
dealer such that each dealer contracted with
one another
- Because acting as agent of other dealers,
the privity was between dealers
themselves.each dealer in privity with
other dealer
- Comparison---dealer a agree with dealer b
to cary out terms of clause.Consideration
is htat each dealer would not sell in the
others territory

if promisee is
contracting as
agent on behalf
of third party,
doctrine of
privity has no
application
between the
third party and
the promisor

New Zealand Shipping CO LTD v AM Satterthwaite & CO LTD 1975 PC


- Shipping of expensive drilling
machine---persuant to terms
- Shipper was the manufacturer of drill
(ajax machine)
- No servant or agent of the carrier
(ship) including independent
contractors shall with any
circumstances be under any liability for
loss or damages caused to the machine
being shipped
- The purchaser (consignee) was known
as AM Satterrecited the machine and
it becomes apparent that it was
damaged ---by stevedores (new
Zealand) hired by carrier to unload the
machineconsignee said yah were

Issue: was the carrier acting as agent


for new Zealand shipping co when
contracting for the shipment of the drill
such that nZ can take the benefit of the
limitation provision?
Held: yes! They were acting as their
agent ---NZ could take benefit of limited
liability clause
4 requirements were met!!!!:
1.The bill of landing was clear that NZ
was intended to be protected by the
limitation provision
2.the bill of landing made it clear that
the carrier was not only contracting on
its own behalf but also as agent for NZ

the success of
agency
argument will
depend on
whether:
Four point test
to show
agency. If
agency, no
longer a privity
problem.
1. THE THIRD
PARTY WAS
INTENDED TO
RECEIVE
BENEFIT
80

bound by terms of billing (could not


have claim against carrier) but instead
sued the company who were to unload
(New Zealand)---they said they are
third party beneficiaries and cannot
claim benefit of that liability clause
because they are not privy to the
contract
- New Zealand said they can benefit
from the limited liability clause because
they were servant agent for
independent contractor hired by the
carrier.
- Bill of landing also stipulated the
carrier was also acting as agent?
- Carriage of good by sea act---carrier
was discharged from all liability for
damages unless lawsuit brought about
within one year of delivery---which it
wasnt---the carrier was off the hook for
more than one reason ---this is why
they sue new Zealand

3.The carrier must have authority form


NZ shipping to act on their behalf or
ratification after the fact for them to do
so
4.Difficulties about consideration must
be overcome
- Carrier in actual contract contracted for
limitation of liability on their behalf but
did so for agents etcit was clear.NZ
had regular type of business relationship
with carrier so carrier acted on its
behalf to protect. ---the performance of
the services for the benefit of the shipper
(NZ performing the benefit of unloading
the ship was the consideration and
therefore NZ shipping should be allowed
to have benefit from the contract)

UNDER THE
CONTRACT
2. THE
PROMISEE IS
ACTING AS
AGENT FOR
THE THIRD
PARTY (I.E.
RELATIONSHIP
AND HOW
DONE IN PAST)
3. PROMISEE
HAS
AUTHORITY
FROM THIRD
PARTY TO ACT
AS ITS AGENT
4.
CONSIDERATIO
N MUST MOVE
FROM THE
THIRD PARTY
(NZ PROVIDED
BENEFIT TO
PURCHASERS)

Employment
True exception to the doctrine of privity
London Drugs LTD v Kuehne & Nagel International LTD 1992 SCC
FACTS
London storing
transformer in
warehouse
Movers damaged
transformer when
moving it - $33500 in
damages
London makes claim to
warehouse
Warehouse says that
they only took limited
liability of $40, thats
all they get if dont
exercise reasonable
care and diligence
London sues two
employees for
negligence
Employees argue they
should be cut some
slack and should come
under limited liability
coverage of owner
London argues wording
of contract
warehouseman no

ISSUE: Can employees be privy to their


employers K limitation of liability?
HELD: YES! for employees. They owed a
duty of care to the appellants, but they are
protected under limitation of liability clause.
Employees can use as a defence but no right
to sue on the contract between the employer
and London Drugs. Nothing changes the rule
in NZ Steam Ships.
Policy reason:
- Doctrine of Privity relaxed in cases of
employee-employer relations- appellant had
option for insurance coverage- also idneitfy
of interest btwn employee and employerand employer performs contractual
obligation through employees, didnt make
sense to interpret the term warehouse man
as to mean anything but the employees
-

court says they can have power and duty


to make incremental changes to prevent
injustice and adapt to changing views of
society- find common interest btwn

- S.C. recognizes the


Doctrine of Privity prevents
third party beneficiaries
from enforcing a contract
they were not a part of.
The court makes an
outright exception in the
case of employees where;
(pg.326) TWO PRONG
TEST
1. The limitation clause
expressly or implicitly
extends benefits to
employee(s) who seek to
rely on the limitation
clause AND;
2. The employees seeking
to benefit from the
limitation clause were
acting in the course of
their employment and
must have been
performing the very
services provided for in
the contract between the
employer and the plaintiff
where the loss occurred
81

indication that it
includes employees of
warehouse
-

employee and employer- an identify of


interest- doctrine of privity did not
appreciate special relationship between
employee and employer
ought to have known that its the
employees who were to perform the
services

- employees who qualify


as third party beneficiaries
can use their employers
limitation of liability clause
as shields (as a defence to
an action)

Mention Edgeworth (below) as countercase/exception

Edgeworth Construction LTD v ND Lea & Associates LTD 1993 SCC


FACTS
E enters into contract with
BC to construct portion of
the highway
During course of
construction project E
claims they lost money
due to errors in
specification and
construction drawings
Contract btwn E and BC
stipulates that any
representation in the
tender documents were
furnished for general
information of bidders
not in any way warranted
or guaranteed on part of
the province
E sues ND and engineers
who actually produced
those specifications and
who fixed those
signatures on them
ND hired by BC to draw up
drawings- ND wants to
rely on clause

ISSUE
RATIO
The employee
Whether the contract btwn E and the province
exception made in
negated the duty of care owned by the
London drugs is
engineers to E which would otherwise had
dependent on
arisen on the facts pleaded?
intention of the
Whether the Engineers not being parties to the
parties and not just
contract could claim the benefit of its
as their status as
exclusion of liability for the representation of
employees
the tender documents.
HELD
Where 3rd party is an
No ND cant rely on the clause!
independent
London drugs doesnt apply because the clause
contractor or
wasnt intended to protect the engineers
professional body
They have their own capacity to get their own
where it can be
protection.
assumed they
REASONS
insure themselves,
Contractor E was relying on the accuracy of the
it cant come
engineers ND design just as much as after it
under insurance of
entered into contract with BC and before
contracting body
Neither the ministry of contractor ever
Exception to
assumed the risk of errors
employee rule
Clause cited in contract absolved BC from any
from London
liability- does not purport to protect
Drug
engineers from liability
In order for third party to seek protection under
exclusion clause must demonstrate that the
contract clause provides protection or should
be implication (i.e. like warehouse man where
we implied employees) provide protection for
the very persons that are claiming the benefit
Limited clause did not seek to protect
engineers- did not give rise to inference that
ND should have the benefit of the exclusion
clause

Subrogation
Fraser River Pile & Dredge LTD v Can-Dive Services LTD 1993 SCC
FACTS
2 parties: Fraser River and
insurance company
Fraser provides ships charter

ISSUE: Can candive claim the


protection of clause
even though it was a

Exception to the doctrine of privity:


If you are going to modify an
insurance K to the detriment of a 3rd
party, you must give 3rd party notice
82

ships to Can-Dive
Can-Dive sinks a ship that
belongs to Fraser
Clause with insurance company
waived right of subrogation
against any charterer cant sue
charterer waived right to sue
(aka if loss is caused by
charterer we will not step in your
shoes)
Insurance company and Fraser
get together and modify contract
waive right to waive right to
sue- meaning Ins will step in for
FR
Insurance contracts can always
be modified
Modified contract has nothing to do
with Can-Dive (3rd party)- Can dive
being sued but want to rely on this
clause that says I cant be suedINS company you waived that rightcant suspend the waiver of that
right in side agr- but are they privy
to this contract such that they can
rely on this clause?

stranger to the
contract on the basis
of principled
exception to the
doctrine of privity
established in lond.
HELD: YES! for CanDive. Cant change
after the event (the
sinking of ship).
Could of altered it
before.- cant revoke
once the benefit is
developed.
Subrogation: i.e.
insurance company
gives you the money,
but stands in your
shoes to sue for the
damages done to you
A principled
exception to privity

(especially in a commercial K such as


this one)
Where Doctrine of Privity will create
wrongdoings, the courts can make
incremental changes to the doctrine
as they see fit (i.e. where it is
necessary)
The defendant must be aware of
the waiver of subrogation, and act
on this to his detriment
C was unaware of the waiver and
therefore did not rely on it to his
detriment
Note: this is an expansion of London
Drug- SCC says SCC did not extend for
their exception to be limited to just
employees
Modified principles from London Drugs
RATIO:
What London doctrine stands for that a
principle of exception for privity can be
established where:
a) the parties intended to extend a
contractual benefit to the third party and
where
b) the third party was acting in the
manner set out in the contract to which
it is not a party

Principled Exception Extension Brown v. Belleville


- Principled exception extended to allow for a suit by a third part, a subsequent
owner of the land, on a contract made by the municipality to build and
maintain a storm sewer with the orginal land owner in return for being allowed
to build the storm sewer on the owners land. The third part brought suit to
enforce the municipalitys obligation to maintain the sewer.
- Third party using principled exception as a sword not a defensive shield.
- Crossed the threshold of only using it as a shield.
Contingent Agreements Next Week

Contingent Agreements
November 26, 2015

If we have a condition precedent, should there be obligations on one party


for best efforts to apply for the condition to be resolved.
Plenty of examples of conditions precedent
o Sale of business subject to disclosure of financial records.
Wiebe v. Bobsien
o Outlines the two forms of condition precedent
(1)Circumstances where a condition precedent prevents the
formation of a contract
Why is there no obligation for either party
o Because there isnt a contract.
83

How do you bind the vendor


Create an option to turn it into a contract
(2)Condition precedent suspends the performance of the
contract
further obligations are suspended until performance of the
condition.
Binding agreement.

o Always resort back to this case for a description of


both these forms of condition precedent

In Canada, form of conveyancing is to enter into a purchase and sale


agreement, it creates obligations on both parties, therefore is binding.
o In the UK, the contract doesnt become binding until there is an
exchange of documents. Until them, no obligations.
o There is often a failure to understand this difference.
Binding or non-binding often comes down to whether the satisfaction of the
condition is subjective or objective
o Something is objective when it defers it to a third party
As soon as there is an obligation of best efforts. Then it is a promissory
conditions precedent
o If not, non promissory condition precedent
Wiebe v. Bobsien (Appeal Court -Dissent)
o Says that the nature of the condition was uncertain because it was
expressed as the purchaser has to sell his house
o Says a purchaser has to get a certain amount, which creates this
uncertainty.
o Says it was a poor construction of the clause. Didnt specify price.
Dynamic Transport v. O.K. Detailing
o Vendor said non-promissory.
o Vendor had done nothing to get the approval for subdivision
o Court says that this is a promissory condition precedent because it
placed an obligation on the vendor because when you are subdividing,
the legislation says that the vendor must seek this approval.
o If you dont use best efforts, then you are in breach in contract.
Remedies
o Specific Performance
Require someone to go ahead seeking approval and see what
happens
o Compensation through Damages
Through third party
Factor the loss of a chance in the damages
o Or can ask to convey the property without the subdivision or the
condition
Purchaser takes the risk of seeking sub dividing after. But asks
for an abatement on the selling price.
Eastwalsh Homes LTD v. Anatal Developments

84

o Trial Judge said that they were entitled to 50 percent of damages


because they were 50/50 of getting approval if they gave there best
efforts
o CA disagreed with the Trial judges conclusion on the facts
Said that the percent chance was much lower of getting
approval
There has been a breach, but not loss.
Unilateral Waiver
o What waiver means, the purchaser has the right to give up protection
that the condition provides.
o Turney v. Zhilka
Purchaser purports to waive the clause without the subdivision
approval.
The vendor says no right to unilateral waiver because it is up to
third party
Court says that there is no unilateral waiver because this would
result in a new contract.
The court interprets the clause as a non-promissory contract.
Judge calls this a true condition precedent non-promissory
condition precedent
When there is a third party component, no right to unilateral
waiver
o Barnett v. Harrison
Purchaser is a developer but there is an approval required by
the municipality for a zoning change
Purchaser wants to waive this zoning change approval and will
take the risk of future of approval
The vendor says that the clause has not been met, so the
contract has not come an end. There has been a subsequent
buyer wanting to offer more money
J. Dickson comes with 4 points why this is a true condition
precedent
(1) Distinction between a right of A to waive a default of B
in the performance of a clause for As benefit, and the
attempt y a A waive his owner default or the default of C a
third party whose consent is needed. It appears that A is
able to benefit from his own default
(2) Where contract is drafted by legal advisors it would
ride rough shod over contract terms and amount to
rewriting the parties agreement (when lawyers are
involved, there must be a purpose behind the clause)
(3) May turn a conditional agreement into an option if
purchaser given right to waive particularly where waiting
for planning consent over a long period
(4) That adherence to the Turney reasoning avoids the
issue of determining in whose favour the condition is
expressed
(5) Turney v. Zhilka is a rule of long standing and is
predictable and certain.
85

When there is a condition subject to third party, the SCC will not
provide a unilateral waiver.
Effect is for the parties to provide a clause in the contract
to mention that something is in the interest of the
purchaser and it can be waived.

EXAM:
o 2 hour closed book examination
o 1 hour Canadian and hour American
o The importance of time management***
Common mistake of spending money on 1 question.
o Easier to get first 60 percent of a question that the last 40%
o 1 or 2 hypothetical questions. 1 or 2 short answer
o Dont try to pick subjects. Exam favours knowledge of everything
o Will get Table of Contents from the casebook.
o Importance on SCC and Ontario Court of Appeals Cases.
o Hypotheticals
Identify issues, and render advise!
Common mistake is to recite the facts. Not good idea.
However, identify the LRFs
Answer: This is a question regarding! Is it this, if so, these
consequences flow. If not, these consequences flow. It is more
likely that it is this (conclusion)!
o Have confidence in yourself!
Representations next class Up to to page 390 is going to be examinable

Representations
Rescission
1. It is commonly used to denote the setting aside of a contract because of
some defect affecting its formation, such as misrepresentation, duress or
undue influence
2. It is also used to describe the discharge of an existing contract by
subsequent agreement of the parties.
3. It is incorrectly, but commonly, used to refer to the situation in which an
innocent party is discharged from having to carry out his or her obligations
under the contract because of the other partys serious breach of contract or
failure to perform. Here, the contract is not wiped out but, on the contrary,
the innocent party is entitled to be compensated by virtue of its previous
existence to the extent necessary to put him or her in the position he or she
would have been in had the contract been performed
The effect of a suit for rescission, on the other hand, is to determine that the
contract is one that ought not to be enforced. Hence, any monetary award
made upon rescission should have as its object the restoration of the parties
to their pre-contract positions

86

Chapter 7: Representations and Terms: Classifications


and Consequences
Misrepresentation and Rescission
Representation in Terms
- Will statements made prior to formation be deemed enforceable as part of a contract?
- 3 categories of statements
(1) Sales Talk - not terms of the contract because they are made without contractual intention (mere fluff)
therefore no liability attached
(2) Misrepresentations : Prior to contract being finalized, Not terms of the contract, can lead to contractual
obligations
(3) Terms: More serious liability if these statements are broken
a. 2 categories (below)
(1) Conditions more important terms of a contract
(2) Warrantees less important terms
- Depending on the type of statement, different remedies are available (damages, rescission)
- If a statement is deemed a Term subcategories
o The remedies to follow are set out in common law
Misrepresentation = Rescission
3 issues:
(1) type of contractual statements which if proven to be false will give rise to a misrepresentation (and therefore rescission)
(2) Nature of the remedy of rescission
(3) Limitations imposed on ones ability to rescind for misrepresentation
Distinction between Rescission and other remedies
- Rescission (3 ways the term is used)
o (1) Commonly used to denote the setting aside of a contract because of some defect affecting is formation
(such as misrepresentation, undue influence, and duress) this is the proper definition
o (2) Discharge of an existing contract by subsequent agreement of the parties
o (3) It is incorrectly, but commonly, used to refer to the situation in which an innocent party is discharged from
having to carry out his or her obligations under the contract because of the other partys serious breach of a
contract or failure to perform.
- Claim for Damages vs. Rescission
o The action for damages is an action to enforce the agreement and thus has as its objection, the substitution of
money damages for the performance, which should have been rendered. The affect of a suit for rescission, on
the other hand, is to determine that the contract is one that ought not to be enforced.
Innocent Misrepresentation- you go back to your original position as if the contract was never there- recession
Negligent Misrepresentation- you continue as if the contract went through- you get what you would have in the contract- this is essentially tort- you are awarded damages
Fraud- everything- this is difficult because you have to prove the intention of the other party to fraud. When you see deceit
you know there is fraud. There is a sense that something is deceitful.

Redgrave v Hurd 1881 CA


Innocent misrepresentation gives rise to rescission
Facts

Issue
Reasoni
ng

Solicitor negotiates to buy part of practice and a house. R says the firm earns 300-400 / year, but documents show 200.
Further documents provided but not looked at apparently show a further income. Buys the home (precondition of
contract) and joins the practice. Realizes practice is useless. Seller is suing for specific performance, counter-claim for
rescission and damages. Argues he was induced by misrepresentation about the finances.
CA: could the defendant rescind the contract as a result of the alleged misrepresentation by the plaintiff?
Trial: for plaintiff.
CA: Yes- for the defendant, counter-claim held.
Trial: because he didnt review the documents that were false he could not have relied on them and therefore was not
induced by them.
CA: when a person makes a material misrepresentation to another inducing him to entire into a contract and as a result
the other person enters into the contract, the law infers that he was induced by the representation to enter into the
contract.
87

In order to take away the remedy of rescission from the aggrieved party you must demonstrate either (1)
aggrieved person first had knowledge of the facts contrary to representation; (2) states or showed by his conduct that he
did not rely on representation.
Ratio

In this case there was no evidence that he didnt rely on the inducement to enter in the contract.
When a person makes a material misrepresentation to another to induce the other to enter into the contract it is
an inference of the law that he was induced by the misrepresentation to enter into it and the representation being
untrue is a sufficient ground for rescinding the contract.(2 exceptions that arise in fact scenario)
Note: misrepresentation must relate to a material matter
Material misrep. must be considered by any reasonable person to be relevant to the decision to enter into the contract.

** If it is a material representation calculated to induce him to enter into a contract, it is an


inference of law that he was induced by the representation to enter into it, and in order to take
away his title to be relived from the contract on the ground that the representation was
untrue, it must be shewn either that he had knowledge of the facts contrary to the
representation, or that he stated in terms, or shewed clearly by his conduct, that he did not
rely on the representation
**Where you have neither evidence that he knew facts to shew that the statement was untrue,
or that he said or did anything to shew that he did not actually rely upon the statement, the
inference remains that he did so rely, and the statement being a material statement, its being
untrue is a sufficient ground for rescinding the contract.
Smith v Land and House Property Corp. 1884 CA
difference between misrepresentation and an opinion
Facts

Issue
Reasoni
ng
Ratio

Offer of sale of a house, current tenant was described as a most desirable tenant. Defendants agree to buy the hotel.
Tenant then goes bankrupt. Defendants refuse to complete transaction. Plaintiff bring suit for spec. Performance.
Defendants claim the description of the tenants virtues amounted was a misrepresentation. Plaintiffs argue the reference
to the tenant was a mere opinion and not a statement of fact.
Can a statement of opinion serve as a basis for rescission due to misrepresentation?
H: Yes.
-if the facts are not equally known to both sides, then a statement of opinion by one person who is in the position to best
know the facts, the opinion is deemed to be a statement of fact, which justifies the opinion.
In applying the above rationale to the case, the landlord knew all the relations, which was not conferred to the seller,
such as when the tenant did not pay his rent perfectly.
A statement of opinion can serve as a basis for rescission

If the facts are not equally known to both sides, then a statement of opinion by the one who
knows the facts best involves very often a statement of a material fact, for he impliedly states
that he knows facts which justify his opinion.
Bank of British Columbia v Wren Developments LTD 1973 BC SC
Facts

Issue
Reasoni
ng

P claims for $25K against Allan and Wren, as they have signed written guarantees for a loan given by the bank. Alan is
a director for the company so he signs the guarantees. Wren defaults on the loan payments. There are a series of
negotiations and guarantees in which Wren deposits shares in the bank as collateral on the loan. Smith (another
director) asks for release of some shares without Alan knowing. Loan comes up for renegotiation; Alan gives new
guarantee. Alan finds out that Smith had made satisfactory arrangements for the payment of the loan and signed a new
guarantee, only routine procedure for Alan to sign. Company defaults on the loan and they go after Allan as a guarantor.
I: Is the defendant, Allan, liable for the balance of the amount claimed under and by virtue of
his written guarantees?
H: NO
When Allan signs the second guarantee, he was misled by words, acts, and conduct of the bank into believing there had
been NO change in the security (shares) held by the plaintiff, otherwise he wouldnt have signed the second guarantee.
The bank was seen as negligent which was the misrepresentation.
When Allen signed the second guarantee, he was misled by the actions and conduct of plaintiff;
88

mislead into believing that there was no change in the collateral of securities, otherwise they
come to conclusion that he would not sign guarantee; unilateral mistake on Allens behalf,
induced by the P in a failure to disclose the relevant facts to him. Allan is not liable on the
second collateral agreement b/c there was a defect in the formation of the contract due to
misrepresentation; bank was negligent b/c shares were not released properly Smith acting a
lone
Misrepresentation of terms can result from a combination of words, acts, and conduct on the part of the party
making them

Ratio

Kupchak v Dayson Holding Ltd. 1965 BC CA


Facts

K purchases shares of a hotel from D (gave up two of their properties and mortgages moved in and worked there) K
learns misrep of the past earnings of the hotel stop making mortgage payments. D sell interest in one of the pieces of
land and tear down existing building. K claims fraud.
Doctrine of Laches: an equitable doctrine which is a defense that may be raised in a case in which the defendants
position has been prejudiced by the plaintiffs delay in taking action.
Election: In such cases the question is, has the person on whom the fraud was practiced, having notice of the fraud,
elected not to avoid the contract?
Could the court order rescission even though one of the properties could not be given back to
the appellant in the original state?

Issue

Reasoni
ng

Ratio

Did the appellant by conduct, word, or silence, 1) elect to affirm the exchange? And 2) whether
their claim is barred by latches?
H: Damages awarded to the appellants (rescission not possible)
(1) property which they acquired by fraud should not bar their action by rescission unless its impractical or unjust to the
R that it ought not to be imposed on a guilty party.
- Rescission is an equitable remedy - court has power to order that the aggrieved party pay money compensation in
order to affect substantial restitution under a decree for rescission.
(2) If the party had the right to avoid the contract by their own words, conduct and lapse of time (it took them a year to
bring it forward!) elect to adhere to it; the other party has a defense against a suit or rescission.
- Obligation existed to continue to manage the hotel and could not have transferred it back prior to a court order.
Barred by laches? Letter to D put them on notice for a proposed action against them. - No prejudice because they were
put on notice.
1) Rescissions is an equitable remedy and the courts have the power to order a
power to pay money compensation to the other party, in order to effect substantial
restitution under a decree for rescission.
2) If the party who had a right to avoid a contract by word or actions, definitively
elected to adhere to it, the other party has a complete defence to any proceedings
for rescission.
3) If the party has done with regard to lapse of time or delay put the other party in a
situation in which it would be unjust to give a remedy, the doctrine of latches will
apply.

REPRESENATION AND TERMS


Ex. Seller says, its a good little bus, you will have no trouble with it the buyer considers this to be a term of the
contract. The seller considers this mere sales talk. Issue is what actually forms the terms of the contract?

Heilbut, Symons & CO v Buckleton 1913 UK


Innocent misrepresentation
Facts

Issue

-H (buyer) are rubber merchants, underwrote a large number of shares in Rubber company.
-any prospectus for the company? which there is not. suggests they are bringing it out - good enough for the B.
-Value of the shares drop B brings action against appellant for fraudulent misrepresentation and in
the alternative for damages for breach of warrantee.
Was the statement that the company was a rubber company a collateral contract (was it a
warrantee) or was it merely an innocent misrepresentation?
Can you claim damages for innocent misrepresentation?
It was an innocent misrepresentation and you cannot claim damages for innocent
89

Reasoni
ng

Ratio

misrepresentations?
Not contested that the A said it was a rubber company. A have to show a warrantee, a contract
collateral to the main contract to take shares. The main contract here was for the sale of the
shares, it entailed application for shares in the rubber company. Collateral contract was the
guarantee that this was a rubber company. In consideration of the other company purchasing
shares, the rubber company was warranting that they were a rubber company.
- not contested that the company was presented as a rubber company.
- H have to show a warranty (a collateral contract to the main contract to take shares whereby the appellants in
consideration of the respondents in taking the shares promise that the company itself was a rubber company).
A warranty exists when there is an affirmation at the time of the sale provided it appears on evidence to be so intended.
In this case there is no evidence to support the existence of this contract.
- statement in saying that the company was rubber company was simply an answer to a question, a statement of fact. No
intention for a warranty to develop.
A person is not liable in damages in any way for an innocent misrepresentation. In order to establish a cause for
damages, the statement must be fraudulent or made recklessly without care (negligently). Must have evidence of
intention between parties to create a warranty
--As a proposition of law that the affirmation of the time of sale is a warrantee provided that it
appears on the evidence to be so implied. The present case is an absence of evidence to
support a collateral contract. There statement in response to the inquiry by R was a mere
statement of fact. The statement in responses to the inquiry by R was a mere statement of
fact, in response to a question of information, it was not meant to be a warrantee. Statement
was innocently made which gives no right to damages in law. Absolutely no evidence of an
intention on either or both of the parties that there should be contractual liability in regards to
the statement regarding the rubber company.
Principle of law a person is not liable in damages for innocent misrepresentation. In order to
establish a cause of action for damages the statement must be fraudulent or recklessly made.
An affirmation at the time of sale is a warranty provided it appears on the evidence
to be so intended.
Principle of law a person is not liable in damages for innocent misrepresentation no
matter in what way or in what form it is made.

Dick Bentley Productions LTD v Harold Smith (Motors) LTD 1965 UK


Facts
Issue

Reasoni
ng

Ratio

B buying car from S


S tells him gearbox replaced only driven X amount of times etc.
B takes wife to see car, says to her in presence of S what S said about car
Was statement a mere innocent misrepresentation, giving rise to no damages or was it a
warranty?
TJ- warranty
awarded damages.
Held: Warranty
-Affirmation at time of sale is a warranty provided it appears on the evidence to be so intended.
- Intended gives rise to many difficulties. If a representation is made in the course of dealings between 2 parties and that
representation is for the very purpose in inducing the other party in acting on it and actually does so, that is prima facie
ground for inferring that the representation was intended as a warrantee.
- It is not necessary to speak of it as being collateral, it suffices to say that the representation intended to be acted upon
and was so.
- Presumption rebutted by the maker of the promise if he / she can show it really was an innocent misrepresentation and
was innocent in fact in making it and it would not be reasonable in the circumstances to be bound by it.
Court held it was warrantee b/c affirmation at time of sale is a warrantee provided it appear on
the evidence it was so intended. Affirmation of good quality of car (i.e. good condition) was a
warrantee b/c there was an intention there that there would be binding liability. If a
representation is made in a course of dealing for a contract and the very purpose of that
misrepresentation is for inducing the other party and it actually induces it him, it is prima facie
ground, that representation was intended as a warranty. Not necessary to speak of it as a
collateral. This inference can be rebutted by the person who made it if they can prove it was
innocent (ex. Innocent in making a statement when looking at the docs with the ar and on them
it said that the car had been refitted. Relied on that, but the documents had a computer error =
innocent).
1) If a representation is made with the very purpose of inducing the other party to
90

act on it, and it induces him to act on it by entering into the contract, that is prima
facie grounds for inferring that the representation is warranty.
2) The maker of the misrepresentation can rebut this inference, if he can show that
it was really an innocent misrepresentation that he was innocent in fault in making
it and it would not be reasonable in these circumstance for him to be bound by it.

Leaf v International Galleries 1950 UK


Facts
Issue
Reasoni
ng

Ratio

Buyer buys oil painting from seller and seller says it was painted by Constable and is on label on pack of picture and on
bill of lading. 5 years pass and b advised it wasnt constable and brings claim for rescission.
I: Whether the buyer is entitled to rescind the contract after he accepted the painting and had
it for 5 years?
H: NO
Court emphasized the action was for rescission and not damages because damages werent claimed by plaintiff.
Term here was a condition - if he came in proper time he could have rejected the goods (SofG Act), but the right to
reject for breach of condition is limited by rule that once buyer has accepted or deemed to accept the goods in
performance of contract, he cannot thereafter reject the goods and is confined in his claim for damages.
Condition = term of contract of the most material nature and a claim to reject for breach of condition is barred then a
claim for rescission on the ground of innocent misrepresentation is also barred.
You are deemed to accept the goods and you retain them and you do not intimate to the seller that you are rejecting
them.
If the term of the contract was a condition, the buyer could reject the picture for breach of the condition at any
time before he accepted it or was deemed to have accepted it, whereas, if it was only a warranty, he could not
reject it but was confined to a claim for damages.
On a contract for sale of goods, an innocent misrepresentation may in proper case be a ground for rescission even
though the contract has been executed but once the buyer is deemed to have accepted the goods, the claim is
barred.

91

Вам также может понравиться